You are on page 1of 186

Tủ sách Tổ kiến

LƯU HÀNH NỘI BỘ

TUYỂN TẬP ĐỀ THI TUYỂN SINH


VÀO LỚP 10 CHUYÊN T OÁN, T IN

Năm 2018 - 2023


BIÊN TẬP: NGUYỄN TIẾN LÂM - TRỊNH HUY VŨ

Câu lạc bộ Toán A1


2 TUYỂN TẬP ĐỀ THI VÀO LỚP 10 CHUYÊN TOÁN

1
A
á n
To

cb
lạ
u

Mục lục

1 Đề thi 7
1.1 Đề thi tuyển sinh vào lớp 10 trường THPT chuyên Khoa học Tự nhiên năm 2018,
vòng 1 . . . . . . . . . . . . . . . . . . . . . . . . . . . . . . . . . . . . . . . 8

1
1.2 Đề thi tuyển sinh vào lớp 10 trường THPT chuyên Khoa học Tự nhiên năm 2018,

A
vòng 2 . . . . . . . . . . . . . . . . . . . . . . . . . . . . . . . . . . . . . . . 9
1.3 Đề thi tuyển sinh vào lớp 10 chuyên Toán Hà Nội, năm 2018 . . . . . . . . . . 10
1.4 Đề thi tuyển sinh vào lớp 10 trường THPT chuyên Khoa học Tự nhiên năm 2019,

n
vòng 1 . . . . . . . . . . . . . . . . . . . . . . . . . . . . . . . . . . . . . . . 11
á
1.5 Đề thi tuyển sinh vào lớp 10 trường THPT chuyên Khoa học Tự nhiên năm 2019,
To
vòng 2 . . . . . . . . . . . . . . . . . . . . . . . . . . . . . . . . . . . . . . . 12
1.6 Đề thi tuyển sinh vào lớp 10 trường THPT chuyên Sư phạm Hà Nội năm 2019,
vòng 2 . . . . . . . . . . . . . . . . . . . . . . . . . . . . . . . . . . . . . . . 13
1.7 Đề thi tuyển sinh vào lớp 10 chuyên Toán Hà Nội, năm 2019 . . . . . . . . . . 14

1.8 Đề thi tuyển sinh vào lớp 10 trường THPT chuyên Khoa học Tự nhiên năm 2020,
cb

vòng 1 . . . . . . . . . . . . . . . . . . . . . . . . . . . . . . . . . . . . . . . 15
1.9 Đề thi tuyển sinh vào lớp 10 trường THPT chuyên Khoa học Tự nhiên năm 2020,
vòng 2 . . . . . . . . . . . . . . . . . . . . . . . . . . . . . . . . . . . . . . . 16
lạ

1.10 Đề thi tuyển sinh vào lớp 10 trường THPT chuyên Sư phạm Hà Nội năm 2020,
vòng 2 . . . . . . . . . . . . . . . . . . . . . . . . . . . . . . . . . . . . . . . 17
1.11 Đề thi tuyển sinh vào lớp 10 chuyên Toán Hà Nội, năm 2020 . . . . . . . . . . 18
u

1.12 Đề thi tuyển sinh vào lớp 10 chuyên Tin Hà Nội, năm 2020 . . . . . . . . . . . 19

1.13 Đề thi tuyển sinh vào lớp 10 trường Phổ thông Năng Khiếu ĐHQG TPHCM
năm 2020, vòng 2 . . . . . . . . . . . . . . . . . . . . . . . . . . . . . . . . . 20
1.14 Đề thi tuyển sinh vào lớp 10 chuyên Toán Thành phố Hồ Chí Minh, năm 2020 . 21
1.15 Đề thi tuyển sinh vào lớp 10 trường THPT chuyên Khoa học Tự nhiên năm 2021,
vòng 1 . . . . . . . . . . . . . . . . . . . . . . . . . . . . . . . . . . . . . . . 22
1.16 Đề thi tuyển sinh vào lớp 10 trường THPT chuyên Khoa học Tự nhiên năm 2021,
vòng 2 . . . . . . . . . . . . . . . . . . . . . . . . . . . . . . . . . . . . . . . 23
1.17 Đề thi tuyển sinh vào lớp 10 trường THPT chuyên Sư phạm Hà Nội năm 2021,
vòng 2 . . . . . . . . . . . . . . . . . . . . . . . . . . . . . . . . . . . . . . . 24
1.18 Đề thi tuyển sinh vào lớp 10 chuyên Toán Hà Nội, năm 2021 . . . . . . . . . . 25
1.19 Đề thi tuyển sinh vào lớp 10 chuyên Tin Hà Nội, năm 2021 . . . . . . . . . . . 26
1.20 Đề thi tuyển sinh vào lớp 10 trường THPT chuyên Khoa học Tự nhiên năm 2022,
vòng 1 . . . . . . . . . . . . . . . . . . . . . . . . . . . . . . . . . . . . . . . 27
1.21 Đề thi tuyển sinh vào lớp 10 trường THPT chuyên Khoa học Tự nhiên năm 2022,
vòng 2 . . . . . . . . . . . . . . . . . . . . . . . . . . . . . . . . . . . . . . . 28

3
4 TUYỂN TẬP ĐỀ THI VÀO LỚP 10 CHUYÊN TOÁN

1.22 Đề thi tuyển sinh vào lớp 10 trường THPT chuyên Sư phạm Hà Nội năm 2022,
vòng 2 . . . . . . . . . . . . . . . . . . . . . . . . . . . . . . . . . . . . . . . 29
1.23 Đề thi tuyển sinh vào lớp 10 chuyên Toán Hà Nội, năm 2022 . . . . . . . . . . 30
1.24 Đề thi tuyển sinh vào lớp 10 chuyên Tin Hà Nội, năm 2022 . . . . . . . . . . . 31
1.25 Đề thi tuyển sinh vào lớp 10 chuyên Toán Thành phố Hồ Chí Minh, năm 2022 . 32
1.26 Đề thi tuyển sinh vào lớp 10 trường THPT chuyên Khoa học Tự nhiên năm 2023,
vòng 1 . . . . . . . . . . . . . . . . . . . . . . . . . . . . . . . . . . . . . . . 33
1.27 Đề thi tuyển sinh vào lớp 10 trường THPT chuyên Khoa học Tự nhiên năm 2023,
vòng 2 . . . . . . . . . . . . . . . . . . . . . . . . . . . . . . . . . . . . . . . 34
1.28 Đề thi tuyển sinh vào lớp 10 trường THPT chuyên Sư phạm Hà Nội năm 2023,
vòng 2 . . . . . . . . . . . . . . . . . . . . . . . . . . . . . . . . . . . . . . . 35
1.29 Đề thi tuyển sinh vào lớp 10 chuyên Toán Hà Nội, năm 2023 . . . . . . . . . . 36
1.30 Đề thi tuyển sinh vào lớp 10 chuyên Tin Hà Nội, năm 2023 . . . . . . . . . . . 37

1
A
2 Lời giải 39
2.1 Đề thi tuyển sinh vào lớp 10 trường THPT chuyên Khoa học Tự nhiên năm 2018,
vòng 1 . . . . . . . . . . . . . . . . . . . . . . . . . . . . . . . . . . . . . . . 40

n
2.2 Đề thi tuyển sinh vào lớp 10 trường THPT chuyên Khoa học Tự nhiên năm 2018,
á
vòng 2 . . . . . . . . . . . . . . . . . . . . . . . . . . . . . . . . . . . . . . . 45
To
2.3 Đề thi tuyển sinh vào lớp 10 chuyên Toán Hà Nội, năm 2018 . . . . . . . . . . 50
2.4 Đề thi tuyển sinh vào lớp 10 trường THPT chuyên Khoa học Tự nhiên năm 2019,
vòng 1 . . . . . . . . . . . . . . . . . . . . . . . . . . . . . . . . . . . . . . . 55
2.5 Đề thi tuyển sinh vào lớp 10 trường THPT chuyên Khoa học Tự nhiên năm 2019,

vòng 2 . . . . . . . . . . . . . . . . . . . . . . . . . . . . . . . . . . . . . . . 60
cb

2.6 Đề thi tuyển sinh vào lớp 10 trường THPT chuyên Sư phạm Hà Nội năm 2019,
vòng 2 . . . . . . . . . . . . . . . . . . . . . . . . . . . . . . . . . . . . . . . 64
2.7 Đề thi tuyển sinh vào lớp 10 chuyên Toán Hà Nội, năm 2019 . . . . . . . . . . 68
2.8 Đề thi tuyển sinh vào lớp 10 trường THPT chuyên Khoa học Tự nhiên năm 2020,
lạ

vòng 1 . . . . . . . . . . . . . . . . . . . . . . . . . . . . . . . . . . . . . . . 73
2.9 Đề thi tuyển sinh vào lớp 10 trường THPT chuyên Khoa học Tự nhiên năm 2020,
u

vòng 1 . . . . . . . . . . . . . . . . . . . . . . . . . . . . . . . . . . . . . . . 78
2.10 Đề thi tuyển sinh vào lớp 10 trường THPT chuyên Sư phạm Hà Nội năm 2020,

vòng 2 . . . . . . . . . . . . . . . . . . . . . . . . . . . . . . . . . . . . . . . 83
2.11 Đề thi tuyển sinh vào lớp 10 chuyên Toán Hà Nội, năm 2020 . . . . . . . . . . 87
2.12 Đề thi tuyển sinh vào lớp 10 chuyên Tin Hà Nội, năm 2020 . . . . . . . . . . . 94
2.13 Đề thi tuyển sinh vào lớp 10 trường Phổ thông Năng Khiếu ĐHQG TPHCM
năm 2020, vòng 2 . . . . . . . . . . . . . . . . . . . . . . . . . . . . . . . . . 99
2.14 Đề thi tuyển sinh vào lớp 10 chuyên Toán Thành phố Hồ Chí Minh, năm 2020 . 105
2.15 Đề thi tuyển sinh vào lớp 10 trường THPT chuyên Khoa học Tự nhiên năm 2021,
vòng 1 . . . . . . . . . . . . . . . . . . . . . . . . . . . . . . . . . . . . . . . 110
2.16 Đề thi tuyển sinh vào lớp 10 trường THPT chuyên Khoa học Tự nhiên năm 2021,
vòng 2 . . . . . . . . . . . . . . . . . . . . . . . . . . . . . . . . . . . . . . . 113
2.17 Đề thi tuyển sinh vào lớp 10 trường THPT chuyên Sư phạm Hà Nội năm 2021,
vòng 2 . . . . . . . . . . . . . . . . . . . . . . . . . . . . . . . . . . . . . . . 119
2.18 Đề thi tuyển sinh vào lớp 10 chuyên Toán Hà Nội, năm 2021 . . . . . . . . . . 122
2.19 Đề thi tuyển sinh vào lớp 10 chuyên Tin Hà Nội, năm 2021 . . . . . . . . . . . 127
CÂU LẠC BỘ TOÁN A1, HOTLINE: 034 761 1986 - 035 290 3286 5

2.20 Đề thi tuyển sinh vào lớp 10 trường THPT chuyên Khoa học Tự nhiên năm 2022,
vòng 1 . . . . . . . . . . . . . . . . . . . . . . . . . . . . . . . . . . . . . . . 131
2.21 Đề thi tuyển sinh vào lớp 10 trường THPT chuyên Khoa học Tự nhiên năm 2022,
vòng 2 . . . . . . . . . . . . . . . . . . . . . . . . . . . . . . . . . . . . . . . 135
2.22 Đề thi tuyển sinh vào lớp 10 trường THPT chuyên Sư phạm Hà Nội năm 2022,
vòng 2 . . . . . . . . . . . . . . . . . . . . . . . . . . . . . . . . . . . . . . . 139
2.23 Đề thi tuyển sinh vào lớp 10 chuyên Toán Hà Nội, năm 2022 . . . . . . . . . . 143
2.24 Đề thi tuyển sinh vào lớp 10 chuyên Tin Hà Nội, năm 2022 . . . . . . . . . . . 148
2.25 Đề thi tuyển sinh vào lớp 10 chuyên Toán Thành phố Hồ Chí Minh, năm 2022 . 153
2.26 Đề thi tuyển sinh vào lớp 10 trường THPT chuyên Khoa học Tự nhiên năm 2023,
vòng 1 . . . . . . . . . . . . . . . . . . . . . . . . . . . . . . . . . . . . . . . 158
2.27 Đề thi tuyển sinh vào lớp 10 trường THPT chuyên Khoa học Tự nhiên năm 2023,
vòng 2 . . . . . . . . . . . . . . . . . . . . . . . . . . . . . . . . . . . . . . . 164

1
2.28 Đề thi tuyển sinh vào lớp 10 trường THPT chuyên Sư phạm Hà Nội năm 2023,

A
vòng 2 . . . . . . . . . . . . . . . . . . . . . . . . . . . . . . . . . . . . . . . 171
2.29 Đề thi tuyển sinh vào lớp 10 chuyên Toán Hà Nội, năm 2023 . . . . . . . . . . 176
2.30 Đề thi tuyển sinh vào lớp 10 chuyên Tin Hà Nội, năm 2023 . . . . . . . . . . . 182

á n
To

cb
lạ
u


u
lạ
cb

To
án
A
1
Chương 1

Đề thi

1
A
á n
To

cb
lạ
u

7
8 TUYỂN TẬP ĐỀ THI VÀO LỚP 10 CHUYÊN TOÁN

1.1. ĐỀ THI TUYỂN SINH VÀO LỚP 10 TRƯỜNG THPT CHUYÊN


KHOA HỌC TỰ NHIÊN NĂM 2018, VÒNG 1
Câu 1.
1. Giải phương trình p √
x2 − x + 2 x3 + 1 = 2 x + 1.

2. Giải hệ phương trình (


xy + y2 = 1 + y
x2 + 2y2 + 2xy = 4 + x

Câu 2.

1
1. Tìm tất cả các cặp số nguyên (x, y) thỏa mãn

A
(x + y)(3x + 2y)2 = 2x + y − 1.

n
r
√ b
á
2. Với a, b là các số thực dương thỏa mãn a + 2b = 2 +
3
, tìm giá trị nhỏ nhất của biểu
To
thức
a b
M=√ +√ .
a + 2b b + 2a

Câu 3. Cho tam giác ABC có đường tròn nội tiếp (I) tiếp xúc với các cạnh BC,CA, AB lần lượt
tại các điểm D, E, F. Gọi K là hình chiếu vuông góc của B trên đường thẳng DE, M là trung
cb

điểm của đoạn thẳng DF.

1. Chứng minh rằng hai tam giác BKM và DEF đồng dạng.
lạ

2. Gọi L là hình chiếu vuông góc của C trên đường thẳng DF, N là trung điểm của đoạn
thẳng DE. Chứng minh rằng hai đường thẳng MK và NL song song.
u

3. Gọi J, X lần lượt là trung điểm của các đoạn thẳng KL và ID. Chứng minh rằng đường

thẳng JX vuông góc với đường thẳng EF.

Câu 4. Trên mặt phẳng cho hai điểm P, Q phân biệt. Xét 10 đường thẳng nằm trong mặt phẳng
trên thỏa mãn tính chất sau:

i) không có hai đường thẳng nào song song hoặc trùng nhau.

ii) mỗi đường thẳng đều đi qua P hoặc Q, không có đường thẳng nào đi qua cả P và Q.

Hỏi 10 đường thẳng trên có thể chia mặt phẳng thành tối đa bao nhiêu miền? Hãy giải thích.
CÂU LẠC BỘ TOÁN A1, HOTLINE: 034 761 1986 - 035 290 3286 9

1.2. ĐỀ THI TUYỂN SINH VÀO LỚP 10 TRƯỜNG THPT CHUYÊN


KHOA HỌC TỰ NHIÊN NĂM 2018, VÒNG 2
Câu 1.
1. Giải hệ phương trình
(
xy(x + y) = 2
x3 + y3 + x3 y3 + 7(x + 1)(y + 1) = 31

2. Giải phương trình p √ √


9 + 3 x(3 − 2x) = 7 x + 5 3 − 2x.

1
Câu 2.

A
1. Cho x, y là các số nguyên sao cho x2 − 2xy − y và xy − 2y2 − x đều chia hết cho 5. Chứng
minh rằng 2x2 + y2 + 2x + y cũng chia hết cho 5.

n
2. Cho a1 , a2 , · · · , a50 là các số nguyên thỏa mãn 1 ≤ a1 ≤ a2 ≤ · · · ≤ a50 ≤ 50 và a1 + a2 +
á
· · · + a50 = 100. Chứng minh rằng từ các số đã cho có thể chọn được một vài số có tổng
bằng 50.
To
Câu 3. Cho ngũ giác lồi ABCDE nội tiếp đường tròn (O) có CD song song với BE. Hai đường
chéo CE và BD cắt nhau tại P. Điểm M thuộc đoạn thẳng BE sao cho ∠MAB = ∠PAE. Điểm K

thuộc đường thẳng AC sao cho MK song song với AD, điểm L thuộc đường thẳng AD sao cho
ML song song với AC. Đường tròn ngoại tiếp tam giác KBC lần lượt cắt BD và CE tại Q, S (Q
cb

khác B, S khác C)

1. Chứng minh rằng ba điểm K, M, Q thẳng hàng.


lạ

2. Đường tròn ngoại tiếp tam giác LDE lần lượt cắt BD,CE tại T, R (T khác D, R khác E).
Chứng minh rằng năm điểm M, S, Q, R, T cùng thuộc một đường tròn.
u

3. Chứng minh rằng đường tròn ngoại tiếp của tam giác PQR tiếp xúc với đường tròn (O).

Câu 4. Cho a, b, c là các số thực dương. Chứng minh rằng


r r ! 
ab bc 1 1
+ √ +√ ≤ 2.
a+b b+c a+b b+c
10 TUYỂN TẬP ĐỀ THI VÀO LỚP 10 CHUYÊN TOÁN

1.3. ĐỀ THI TUYỂN SINH VÀO LỚP 10 CHUYÊN TOÁN HÀ


NỘI, NĂM 2018
Câu 1.

1. Giải phương trình: x2 + 3x + 8 = (x + 5) x2 + x + 2.
(
y2 − 2xy = 8x2 − 6x + 1
2. Giải hệ phương trình 2
y = x3 + 8x2 − x + 1

Câu 2.
1. Cho p, q là hai số nguyên tố lớn hơn 5. Chứng minh rằng p4 + 2019q4 chia hết cho 20.
√ √

1
2. Cho các số nguyên dương a, b, c, d thỏa mãn a < b ≤ c < d, ad = bc và d − a ≤ 1.

A
a. Chứng minh rằng a + d > b + c.
b. Chứng minh rằng a là một số chính phương.

n
Câu 3.
á
1. Với x, y, z là các số thực dương thỏa mãn xyz = 1. Chứng minh rằng
To
1 1 1
+ + = 1.
xy + x + 1 yz + y + 1 zx + z + 1

1 1 1
2. Với x, y, z là các số thự dương thay đổi và thỏa mãn + + = 3, tìm giá trị lớn nhất của
cb

x y z
biểu thức
1 1 1
P= p +p +√ .
2x2 + y2 + 3 2y2 + z2 + 3 2z2 + x2 + 3
lạ

Câu 4. Cho tứ giác ABCD (không có hai cạnh nào song song) nội tiếp đường tròn (O). Các tia
BA và CD cắt nhau tại điểm F. Gọi E là giao điểm của hai đường chéo AC và BD. Vẽ hình bình
u

hành AEDK.

1. Chứng minh rằng tam giác FKD đồng dạng với tam giác FEB.

2. Gọi M, N tương ứng là trung điểm của các cạnh AD, BC. Chứng minh rằng đường thẳng
MN đi qua trung điểm của đoạn thẳng EF.

3. Chứng minh rằng đường thẳng EF tiếp xúc với đường tròn ngoại tiếp của tam giác EMN.

Câu 5. Cho tập hợp S = {x ∈ Z | 1 ≤ x ≤ 50}. Xét A là một tập con bất kỳ của tập hợp S và
có tính chất: Không có ba phần tử nào của tập hợp A là số đo độ dài ba cạnh của một tam giác
vuông.
1. Tìm một tập hợp A có đúng 40 phần tử và thỏa mãn điều kiện đề bài.

2. Có hay không có một tập hợp A có đúng 41 phần tử và thỏa mãn điều kiện của đề bài?
Hãy giải thích câu trả lời.
CÂU LẠC BỘ TOÁN A1, HOTLINE: 034 761 1986 - 035 290 3286 11

1.4. ĐỀ THI TUYỂN SINH VÀO LỚP 10 TRƯỜNG THPT CHUYÊN


KHOA HỌC TỰ NHIÊN NĂM 2019, VÒNG 1
Câu 1.
1. Giải phương trình
26x + 5 √ p
√ + 2 26x + 5 = 3 x2 + 30.
x2 + 30
2. Giải hệ phương trình (
x2 + y2 = 2
(x + 2y)(2 + 3y2 + 4xy) = 27

1
Câu 2.

A
1. Tìm tất cả các cặp (x, y) nguyên thỏa mãn

(x2 − x + 1)(y2 + xy) = 3x − 1.

á n
2. Với x, y là các số thực thỏa mãn 1 ≤ y ≤ 2, xy + 2 ≥ 2y, tìm giá trị nhỏ nhất của biểu thức
To
x2 + 4
M= .
y2 + 1

Câu 3. Cho hình vuông ABCD, đường tròn (O) nội tiếp hình vuông ABCD tiếp xúc với các
cạnh AB, AD lần lượt tại các điểm E, F. Gọi giao điểm của CE và BF là G.
cb

1. Chứng minh rằng năm điểm A, F, O, G, E cùng nằm trên một đường tròn.

2. Gọi giao điểm của FB và đường tròn (O) là M (M 6= F). Chứng minh rằng M là trung
lạ

điểm của đoạn thẳng BG.

3. Chứng minh rằng trực tâm của tam giác GAF nằm trên đường tròn (O).
u

Câu 4. Cho x, y, z là các số thực dương thỏa mãn xy + yz + xz = 1. Chứng minh rằng:
!3
1 1 1 2 x y z
2
+ 2
+ 2
≥ √ +p +√ .
1+x 1+y 1+z 3 1 + x2 1 + y2 1 + z2
12 TUYỂN TẬP ĐỀ THI VÀO LỚP 10 CHUYÊN TOÁN

1.5. ĐỀ THI TUYỂN SINH VÀO LỚP 10 TRƯỜNG THPT CHUYÊN


KHOA HỌC TỰ NHIÊN NĂM 2019, VÒNG 2
Câu 1.
1. Giải hệ phương trình (
3x2 + y2 + 4xy = 8
(x + y)(x2 + xy + 2) = 8

2. Giải phương trình √ √


27 + x2 + x 27 + 2x
p = √ .
2 + 5 − (x2 + x) 2 + 5 − 2x

1
Câu 2.

A
1. Chứng minh rằng với mọi số nguyên dương n, ta luôn có
7 7 7
(27n + 5)7 + 10 + (10n + 27)7 + 5 + (5n + 10)7 + 27

chia hết cho 42.


á n
To
2. Với x, y là các số thực dương thỏa mãn điều kiện 4x2 + 4y2 + 17xy + 5x + 5y ≥ 1, tìm giá
trị nhỏ nhất của biểu thức
P = 17x2 + 17y2 + 16xy.

Câu 3. Cho tam giác ABC cân tại A, có đường tròn nội tiếp (I). Các điểm E, F theo thứ tự thuộc
cb

các cạnh CA, AB (E khác C và A; F khác B và A) sao cho EF tiếp xúc với đường tròn (I) tại
điểm P. Gọi K, L lần lượt là hình chiếu vuông góc của E, F trên BC. Giả sử FK cắt EL tại điểm
J. Gọi H là hình chiếu vuông góc của J trên BC.
lạ

1. Chứng minh rằng HJ là phân giác của ∠EHF.

2. Ký hiệu S1 và S2 lần lượt là diện tích của các tứ giác BFJL và CEJK. Chứng minh rằng
u

S1 BF 2
= .
S2 CE 2

3. Gọi D là trung điểm của cạnh BC. Chứng minh rằng ba điểm P, J, D thẳng hàng.

Câu 4. Cho M là tập tất cả 4039 số nguyên liên tiếp từ −2019 đến 2019. Chứng minh rằng
trong 2021 số đôi một phân biệt được chọn bất kỳ từ tập M luôn tồn tại 3 số đôi một phân biệt
có tổng bằng 0.
CÂU LẠC BỘ TOÁN A1, HOTLINE: 034 761 1986 - 035 290 3286 13

1.6. ĐỀ THI TUYỂN SINH VÀO LỚP 10 TRƯỜNG THPT CHUYÊN


SƯ PHẠM HÀ NỘI NĂM 2019, VÒNG 2
Câu 1. Cho hai số thực phân biệt a, b thỏa mãn a3 + b3 = a2 b2 (ab − 3). Tính giá trị của biểu
thức T = a + b − ab.

Câu 2. Cho các đa thức P(x) = m1 x2 +n1 x +k1 , Q(x) = m2 x2 +n2 x +k2 , R(x) = m3 x2 +n3 x +k3
với mi , ni , ki là các số thực và mi > 0, i = 1, 2, 3. Giả sử phương trình P(x) = 0 có hai nghiệm
phân biệt a1 , a2 , phương trình Q(x) = 0 có hai nghiệm phân biệt b1 , b2 , phương trình R(x) = 0
có hai nghiệm phân biệt c1 , c2 thỏa mãn

P(c1 ) + Q(c1 ) = P(c2 ) + Q(c2 )


P(b1 ) + R(b1 ) = P(b2 ) + R(b2 )

1
Q(a1 ) + R(a1 ) = Q(a2 ) + R(a2 )

A
Chứng minh rằng a1 + a2 = b1 + b2 = c1 + c2 .

Câu 3.

n
1. Tìm các số nguyên x, y thỏa mãn x2 y2 − 4x2 y + y3 + 4x2 − 3y2 + 1 = 0.
á
To
2. Cho ba số nguyên dương a, b, c thỏa mãn a3 + b3 + c3 chia hết cho 14. Chứng minh rằng
abc chia hết cho 14.

Câu 4. Cho tam giác ABC có ba góc nhọn nội tiếp đường tròn (O) và AB > AC. Gọi D, E lần

lượt là chân các đường cao của tam giác ABC hạ từ A, B. Gọi F là chân đường vuông góc hạ từ
cb

B lên đường thẳng AO.

1. Chứng minh rằng B, D, E, F là bốn đỉnh của một hình thang cân.
lạ

2. Chứng minh rằng EF đi qua trung điểm của BC.

3. Gọi P là giao điểm thứ hai của đường thẳng AO và đường tròn (O), M và N lần lượt là
u

trung điểm của EF và CP. Tính góc ∠BMN.


Câu 5. Cho tập X thỏa mãn tính chất sau: tồn tại 2019 tập con A1 , A2 , · · · , A2019 của X sao cho
mỗi tập con A1 , A2 , · · · , A2019 có đúng ba phần tử và hai tập con Ai , A j đều có đúng một phần tử
chung với mọi 1 ≤ i < j ≤ 2019.

1. Chứng minh rằng tồn tại 4 tập hợp trong các tập A1 , A2 , · · · , A2019 sao cho giao của 4 tập
hợp này có đúng một phần tử.

2. Chứng minh rằng số phần tử của X phải lớn hơn hoặc bằng 4039.
14 TUYỂN TẬP ĐỀ THI VÀO LỚP 10 CHUYÊN TOÁN

1.7. ĐỀ THI TUYỂN SINH VÀO LỚP 10 CHUYÊN TOÁN HÀ


NỘI, NĂM 2019
Câu 1.
√ √ √
1. Giải phương trình ( x + 5 − x)(1 + x2 + 5x) = 5.
(
x2 + 7 = 4y2 + 4y
2. Giải hệ phương trình
x2 + 3xy + 2y2 + x + y = 0.

Câu 2.
1. Cho biểu thức P = abc(a − 1)(b + 4)(c + 6) với a, b, c là các số nguyên thỏa mãn a + b +
c = 2019. Chứng minh rằng giá trị của biểu thức P chia hết cho 6.

1
2. Tìm tất cả các số tự nhiên n sao cho giá trị của biểu thức

A
√ √
q
Q = n+2+ n+ n+2

n
là số nguyên.
á
To
Câu 3. Cho biểu thức K = ab + 4ac − 4bc, với a, b, c là các số thực không âm thỏa mãn a + b +
2c = 1.
1
1. Chứng minh K ≥ − .
2

cb

2. Tìm giá trị lớn nhất của biểu thức K.


Câu 4. Cho tam giác ABC có ba góc nhọn (AB < AC), nội tiếp đường tròn (O). Gọi I là tâm
đường tròn nội tiếp tam giác ABC. Tia AI cắt đoạn thẳng BC tại điểm J, cắt đường tròn (O) tại
lạ

điểm thứ hai M (M khác A).


1. Chứng minh rằng MI 2 = MJ · MA.
u

2. Kẻ đường kính MN của đường tròn (O). Đường thẳng AN cắt các tia phân giác trong của

góc ABC và góc ACB lần lượt tại các điểm P và Q. Chứng minh rằng N là trung điểm của
đoạn thẳng PQ.

3. Lấy điểm E bất kỳ thuộc cung nhỏ MC của đường tròn (O) (E khác M). Gọi F là điểm đối
xứng với điểm I qua điểm E. Gọi R là giao điểm của hai đường thẳng PC và QB. Chứng
minh rằng bốn điểm P, Q, R, F cùng nằm trên một đường tròn.
Câu 5. Mỗi điểm trong mặt phẳng được tô bởi một trong hai màu xanh hoặc đỏ.
1. Chứng minh trong mặt phẳng đó tồn tại hai điểm được tô bởi cùng một màu và có khoảng
cách bằng d.

2. Gọi tam giác có ba đỉnh được tô cùng một màu là tam giác đơn sắc. Chứng minh trong
mặt phẳng đó tồn tại hai tam giác đơn sắc là hai tam giác vuông đồng dạng với nhau theo
1
tỉ số k = .
2019
CÂU LẠC BỘ TOÁN A1, HOTLINE: 034 761 1986 - 035 290 3286 15

1.8. ĐỀ THI TUYỂN SINH VÀO LỚP 10 TRƯỜNG THPT CHUYÊN


KHOA HỌC TỰ NHIÊN NĂM 2020, VÒNG 1
Câu 1.
1. Giải hệ phương trình (
x2 + y2 + xy = 7
9x3 = xy2 + 70(x − y).

2. Giải phương trình


√ √ p
11 5 − x + 8 2x − 1 = 24 + 3 (5 − x)(2x − 1).

1
Câu 2.

A
1. Tìm x, y nguyên dương thỏa mãn

x2 y2 − 16xy + 99 = 9x2 + 36y2 + 13x + 26y.

2. Với a, b là các số thực dương thỏa mãn


á n
To
2 ≤ 2a + 3b ≤ 5, 8a + 12b ≤ 2a2 + 3b2 + 5ab + 10.

Chứng minh rằng


3a2 + 8b2 + 10ab ≤ 21.

cb

Câu 3. Cho tam giác ABC có ∠BAC là góc nhỏ nhất trong ba góc của tam giác và nội tiếp đường
tròn (O). Điểm D thuộc cạnh BC sao cho AD là phân giác của ∠BAC. Lấy các điểm M, N thuộc
(O) sao cho các đường thẳng CM và BN cùng song song với đường thẳng AD.
lạ

1. Chứng minh rằng AM = AN.

2. Gọi giao điểm của đường thẳng MN với các đường thẳng AC, AB lần lượt là E, F. Chứng
u

minh rằng bốn điểm B,C, E, F cùng thuộc một đường tròn.

3. Gọi P, Q theo thứ tự là trung điểm của các đoạn thẳng AM, AN. Chứng minh rằng EQ, FP, AD
đồng quy.

Câu 4. Cho a, b, c là các số thực dương thỏa mãn a + b + c = 3. Chứng minh rằng

a(a + bc)2 b(b + ca)2 c(c + ab)2


+ + ≥ 4.
b(ab + 2c2 ) c(bc + 2a2 ) a(ca + 2b2 )
16 TUYỂN TẬP ĐỀ THI VÀO LỚP 10 CHUYÊN TOÁN

1.9. ĐỀ THI TUYỂN SINH VÀO LỚP 10 TRƯỜNG THPT CHUYÊN


KHOA HỌC TỰ NHIÊN NĂM 2020, VÒNG 2
Câu 1.
1. Giải hệ phương trình
(
(x + y)(x + 1) = 4
.
(y2 + xy + x + y + 5)(x3 + y3 + 12y + 13) = 243.

2. Giải phương trình


(x − 12)7 + (2x − 12)7 + (24 − 3x)7 = 0.

1
Câu 2.

A
1. Tìm tất cả các số nguyên dương a, b, c sao cho cả ba số 4a2 + 5b, 4b2 + 5c, 4c2 + 5a đều
là bình phương của số nguyên dương.

n
2. Từ một bộ bốn số thực (a, b, c, d) ta xây dựng bộ số mới (a + b, b + c, c + d, d + a) và liên
á
tiếp xây dựng các bộ số mới theo quy tắc trên. Chứng minh rằng nếu có hai thời điểm
khác nhau ta thu được cùng một bộ số (có thể khác thứ tự) thì bộ số ban đầu phải có dạng
To
(a, −a, a, −a).

Câu 3. Cho tam giác ABC cân tại A với ∠BAC < 90◦ . Điểm E thuộc cạnh AC sao cho ∠AEB >
90◦ . Gọi P là giao điểm của BE và trung trực của BC. Gọi K là hình chiếu vuông góc của P lên

AB. Gọi Q là hình chiếu vuông góc của E lên AP. Gọi giao điểm của EQ và PK là F.
cb

1. Chứng minh rằng bốn điểm A, E, P, F cùng thuộc một đường tròn.

2. Gọi giao điểm của KQ và PE là L. Chứng minh rằng LA vuông góc với LE.
lạ

3. Gọi giao điểm của FL và AB là S. Gọi giao điểm của KE và AL là T . Lấy R là điểm đối
xứng của A qua L. Chứng minh rằng đường tròn ngoại tiếp tam giác AST và đường tròn
u

ngoại tiếp tam giác BPR tiếp xúc với nhau.


Câu 4. Cho a, b, c là các số thực dương thỏa mãn a + b + c = 3. Chứng minh rằng
 2  
1 1 1 4 a b c
3 + + −1 +1 ≥ +3 + + .
a b c abc bc ca ab
CÂU LẠC BỘ TOÁN A1, HOTLINE: 034 761 1986 - 035 290 3286 17

1.10. ĐỀ THI TUYỂN SINH VÀO LỚP 10 TRƯỜNG THPT CHUYÊN


SƯ PHẠM HÀ NỘI NĂM 2020, VÒNG 2
Câu 1. Cho ba số thực x, y, z thỏa mãn các điều kiện sau

3 = 3y3 = 4z3
2x

p √ √
3
2x2 + 3y2 + 4z2 = 2 + 3 12 + 3 16

xyz > 0.

1 1 1
Tính giá trị của biểu thức P = + + .
x y z

1
Câu 2. Xét phương trình bậc hai ax2 + bx + c = 0 (1), trong đó a, b, c là các số nguyên dương.
Biết rằng các điều kiện sau được thỏa mãn: phương trình (1) có nghiệm, số a2020b chia hết cho

A
12, số c2 + 3 chia hết cho c + 3. Hãy tìm giá trị lớn nhất của tổng a + b + c.

Câu 3. Tìm số nguyên a bé nhất sao cho x4 + 2x2 − 4x + a ≥ 0 với mọi số thực x.

á n
Câu 4. Cho tam giác nhọn ABC nội tiếp đường tròn (O) có AB > BC. Một đường tròn đi qua
hai đỉnh A,C của tam giác ABC lần lượt cắt các cạnh AB, BC tại hai điểm K, N (K, N khác các
To
đỉnh của tam giác ABC.) Giả sử đường tròn (O) và đường tròn ngoại tiếp của tam giác BKN cắt
nhau tại giao điểm thứ hai là M (M khác B). Chứng minh rằng

1. Ba đường thẳng BM, KN, AC đồng quy tại điểm P.


cb

2. Tứ giác MNCP nội tiếp.

3. BM 2 − PM 2 = BK · BA − PC · PA.
lạ

Câu 5. Cho hai số A, B cùng có 2020 chữ số. Biết rằng số A có đúng 1945 chữ số khác 0, bao
gồm 1930 chữ số ngoài cùng về bên trái và 15 chữ số ngoài cùng về bên phải, số B có đúng
1954 chữ số khác 0, bao gồm 1930 chữ số ngoài cùng về bên trái và 24 chữ số ngoài cùng về
u

bên phải. Chứng minh rằng UCLN(A, B) là một số có không quá 1954 chữ số.

18 TUYỂN TẬP ĐỀ THI VÀO LỚP 10 CHUYÊN TOÁN

1.11. ĐỀ THI TUYỂN SINH VÀO LỚP 10 CHUYÊN TOÁN HÀ


NỘI, NĂM 2020
Câu 1.

1. Giải phương trình x2 + 3x + 5 = (x + 3) x2 + 5.

2. Cho các số thực a, b, c thỏa mãn a + b − 2c = 0 và 2ab − bc − ca = 0. Chứng minh rằng


a = b = c.

Câu 2.
1. Chứng minh với mọi số nguyên dương n, số A = 11n + 7n − 2n − 1 chia hết cho 15.
√ m

1
2. Cho hai số nguyên dương m và n thỏa mãn 11 − > 0. Chứng minh rằng
n

A

√ m 3( 11 − 3)
11 − ≥ .
n mn

n
Câu 3.
á
1. Cho đa thức P(x) với hệ số thực thỏa mãn P(1) = 3 và P(3) = 7. Tìm đa thức dư trong
To
phép chia đa thức P(x) cho đa thức x2 − 4x + 3.

2. Với a, b, c là các số thực không âm thỏa mãn a + b + c + abc = 4, tìm giá trị lớn nhất của
biểu thức P = ab + bc + ca.

cb

Câu 4. Cho tam giác ABC có ba góc nhọn và AB < AC. Gọi (I) là đường tròn nội tiếp của tam
giác ABC và K là tâm đường tròn bàng tiếp trong góc A của tam giác ABC. Gọi D, E, F lần lượt
là chân các đường vuông góc kẻ từ điểm I đến các đường thẳng BC,CA, AB. Đường thẳng AD
cắt đường tròn (I) tại hai điểm phân biệt D và M. Đường thẳng qua K song song với đường
lạ

thẳng AD cắt đường thẳng BC tại N.

1. Chứng minh rằng tam giác MFD đồng dạng với tam giác BNK.
u

2. Gọi P là giao điểm của BI và FD. Chứng minh rằng ∠BMF = ∠DMP.

3. Chứng minh rằng đường tròn ngoại tiếp của tam giác MBC đi qua trung điểm của đoạn
thẳng KN.

Câu 5. Cho một bảng ô vuông kích thước 6 × 7 (6 hàng, 7 cột) được tạo bởi các ô vuông có kích
thước 1 × 1. Mỗi ô vuông có kích thước 1 × 1 được tô bởi một trong hai màu đen hoặc trắng sao
cho trong mọi bảng ô vuông kích thước 2 × 3 hoặc 3 × 2, có ít nhất hai ô vuông kích thước 1 × 1
được tô màu đen có chung cạnh. Gọi m là số ô vuông kích thước 1 × 1 được tô màu đen trong
bảng

1. Chỉ ra một cách tô sao cho m = 20.

2. Tìm giá trị nhỏ nhất của m.


CÂU LẠC BỘ TOÁN A1, HOTLINE: 034 761 1986 - 035 290 3286 19

1.12. ĐỀ THI TUYỂN SINH VÀO LỚP 10 CHUYÊN TIN HÀ


NỘI, NĂM 2020
Câu 1.

1. Giải phương trình (x + 2) x2 + 1 = x2 + 2x + 1.

2. Chứng minh rằng


1 1 1 1
√ √ + √ √ +···+ √ √ = 1− √ .
2 1+1 2 3 2+2 3 2021 2020 + 2020 2021 2021
Câu 2.
1. Chứng minh rằng với mọi số nguyên dương n, số A = 59n − 17n − 9n + 2n chia hết cho 35.

1
A
2. Tìm tất cả các số nguyên x, y thỏa mãn điều kiện x2 y − 3y − 4x − 1 = 0.

Câu 3.

n
1. Tìm tất cả các số thực a, b, c thỏa mãn đồng thời các điều kiện a2 + b2 + c2 = 38, a + b = 8
và b + c ≥ 7.
á
To
2. Với a, b, c là các số thực không
√ âm và luôn thỏa mãn a2 + b2 + c2 = 2ab + 2bc + 2ca,
chứng minh rằng a + b + c ≥ 3 2abc.

Câu 4. Cho tam giác ABC có ba góc nhọn, AB < AC và ba đường cao AD, BE,CF cùng đi qua

điểm H. Gọi (S) là đường tròn ngoại tiếp của tam giác DEF.
cb

1. Chứng minh rằng đường tròn (S) đi qua trung điểm của đoạn thẳng AH.

2. Gọi M và N lần lượt là giao điểm của đường tròn (S) với các đoạn thẳng BH và CH. Tiếp
lạ

tuyến tại D của đường tròn (S) cắt MN tại điểm T . Chứng minh rằng đường thẳng HT
song song với đường thẳng EF.
u

3. Gọi P là giao điểm của hai đường thẳng BH và DF, Q là giao điểm của hai đường thẳng

CH và DE. Chứng minh rằng ba điểm T, P, Q là ba điểm thẳng hàng.

Câu 5. Trên bàn có 6 hộp kẹo, mỗi hộp có 5 viên kẹo. An và Bình cùng chơi một trò chơi như
sau: mỗi lượt chơi, An sẽ chọn một hộp tùy ý và lấy ít nhất 1 viên kẹo trong hộp đó; còn Bình
thì chọn một số hộp và trong các hộp đã chọn, mỗi hộp lấy đúng 1 viên kẹo. Hai bạn luân phiên
thực hiện lượt chơi của mình. Bạn đầu tiên không thể thực hiện được lượt chơi của mình là người
thua cuộc. Nếu An là người lấy kẹo trước, hãy chỉ ra chiến thuật chơi để Bình là người thắng
cuộc.
20 TUYỂN TẬP ĐỀ THI VÀO LỚP 10 CHUYÊN TOÁN

1.13. ĐỀ THI TUYỂN SINH VÀO LỚP 10 TRƯỜNG PHỔ THÔNG


NĂNG KHIẾU ĐHQG TPHCM NĂM 2020, VÒNG 2
Câu 1. Cho các phương trình x2 + ax + 3 = 0 và x2 + bx + 5 = 0 với a, b là các tham số
(a) Chứng minh rằng nếu ab ≥ 16 thì trong hai phương trình có ít nhất một phương trình có
nghiệm.
(b) Giả sử hai phương trình trên có nghiệm chung x0 . Tìm a, b sao cho |a| + |b| có giá trị nhỏ
nhất.
Câu 2. Cho phương trình 3x2 − y2 = 23n với n là số tự nhiên.
(a) Chứng minh rằng nếu n chẵn thì phương trình đã cho không có nghiệm nguyên (x, y).

1
(b) Chứng minh rằng nếu n lẻ thì phương trình đã cho có nghiệm nguyên (x, y).

A
Câu 3. Cho đường tròn (O), dây cung BC không chứa tâm O và điểm A thay đổi trên cung lớn
BC. Lấy các điểm E, F thỏa mãn ∠ABE = ∠CAE = ∠ACF = ∠BAF = 90◦ .

n
(a) Chứng minh rằng AE · AC = AF · AB và O là trung điểm của EF.
á
(b) Hạ AD vuông góc với EF (D ∈ EF). Chứng minh rằng các tam giác DAB và DCA đồng
To
dạng và D thuộc một đường tròn cố định.
(c) Gọi G là giao điểm của AD và đường tròn (O) (G 6= A). Chứng minh rằng AD đi qua một
điểm cố định và GB · AC = GC · AB.

cb

(d) Gọi K là tâm đường tròn ngoại tiếp của tam giác AEF. Chứng minh rằng AK đi qua một
điểm cố định.
Câu 4. Cho số tự nhiên a = 313 .57 .720 .
lạ

(a) Gọi A là tập hợp các số nguyên dương k sao cho k là ước của a và k chia hết cho 105. Hỏi
tập A có bao nhiêu phần tử?
u

(b) Giả sử B là một tập con bất kỳ của A có 9 phần tử. Chứng minh ta luôn có thể tìm được 2

phần tử của B sao cho tích của chúng là số chính phương.


Câu 5. Cho hệ phương trình với k là tham số:
 r r
x x x
√ + + =k





 yz y z





 y r r
y y
√ + + =k

 zx z x




 r r

 z z z
√ + + =k


xy x y
(a) Giải hệ với k = 1.
(b) Chứng minh hệ vô nghiệm với k ≥ 2 và k 6= 3.
CÂU LẠC BỘ TOÁN A1, HOTLINE: 034 761 1986 - 035 290 3286 21

1.14. ĐỀ THI TUYỂN SINH VÀO LỚP 10 CHUYÊN TOÁN THÀNH


PHỐ H Ồ C HÍ M INH , NĂM 2020

a b c
Câu 1. Cho ba số dương a, b, c thỏa mãn điều kiện + + = 2020.
 2 b +c c + a a + b
a b2 c2
Tính giá trị của biểu thức P = + + : (a + b + c).
b+c c+a a+b
Câu 2.
√ √
a) Giải phương trình: 2x2 + x + 9 + 2x2 − x + 1 = x + 4.
(
y2 − 2xy = 8x2 − 6x + 1
b) Giải hệ phương trình 2

1
y = x3 + 8x2 − x + 1.

A
Câu 3. Cho tam giác nhọn ABC (AB < BC < CA) nội tiếp đường tròn (O). Từ A kẻ đường thẳng
song song với BC cắt (O) tại A1 . Từ B kẻ đường thẳng song song với AC cắt (O) tại B1 . Từ C kẻ

n
đường thẳng song song với AB cắt (O) tại C1 . Chứng minh rằng các đường thẳng qua A1 , B1 ,C1
lần lượt vuông góc với BC,CA, AB đồng quy.
á
To
Câu 4.
a2 + b2 (a − b)2
a) Cho 2 số thực a, b. Chứng minh rằng: ≥ ab + 2 .
2 a + b2 + 2

b) Cho 2 số dương a, b thỏa mãn điều kiện a + b ≤ 3.


cb

20 7
Tìm giá trị nhỏ nhất của biểu thức: Q = b − a + + .
a b
Câu 5. Đường tròn (I) nội tiếp tam giác ABC tiếp xúc với các cạnh AB, BC,CA lần lượt tại
lạ

D, E, F. Kẻ đường kính EJ của đường tròn (I). Gọi d là đường thẳng qua A song song với BC.
Đường thẳng JD cắt d, BC lần lượt tại L, H.
u

(a) Chứng minh rằng E, F, L thẳng hàng.


(b) JA, JF cắt BC lần lượt tại M, K. Chứng minh rằng MH = MK.

Câu 6. Tìm tất cả các số nguyên dương x, y thỏa mãn phương trình 3x − y3 = 1.
22 TUYỂN TẬP ĐỀ THI VÀO LỚP 10 CHUYÊN TOÁN

1.15. ĐỀ THI TUYỂN SINH VÀO LỚP 10 TRƯỜNG THPT CHUYÊN


KHOA HỌC TỰ NHIÊN NĂM 2021, VÒNG 1
Câu 1. (2,0 điểm) Giải phương trình
√ √ p
13 5 − x + 18 x + 8 = 61 + x + 3 (5 − x)(x + 8)

Câu 2. (2,0 điểm) Giải hệ phương trình


 4
x + y4 + 6x2 y2 = 1
x(x + y)4 = x − y

Câu 3. (2,0 điểm) Tìm số nguyên dương n nhỏ nhất biết rằng khi chia n cho 7, 9, 11, 13 ta nhận

1
được các số dư là 3, 4, 5, 6.

A
Câu 4. (3,0 điểm) Cho tam giác nhọn ABC có điểm P nằm trong tam giác (P không nằm trên
các cạnh). Gọi J, K, L lần lượt là tâm đường tròn nội tiếp của các tam giác PBC, PCA, PAB.

n
d + CKA
1. Chứng minh rằng BJC d = 90◦ .
d + ALB
á
To
2. Giả sử PB = PC và PC < PA. Gọi X,Y, Z lần lượt là hình chiếu vuông góc của J, K, L trên
các cạnh BC,CA, AB. Dựng hình bình hành XYW Z. Chứng minh rằng W nằm trên phân
giác BAC.
d

Câu 5. (1,0 điểm) Cho tập A = {1, 2, ..., 2021}. Tìm số nguyên dương k lớn nhất (k ≥ 2) sao
cho ta có thể chọn được k số phân biệt từ tập A mà tổng của hai số phân biệt bất kỳ trong k số
cb

được chọn không chia hết cho hiệu của chúng.


lạ
u

CÂU LẠC BỘ TOÁN A1, HOTLINE: 034 761 1986 - 035 290 3286 23

1.16. ĐỀ THI TUYỂN SINH VÀO LỚP 10 TRƯỜNG THPT CHUYÊN


KHOA HỌC TỰ NHIÊN NĂM 2021, VÒNG 2
Câu 1. (4,0 điểm)
1. Với a, b, c là các số thực thoả mãn a + b + c 6= 0 và (a + b)(b + c)(c + a) = 1, chứng minh
rằng
a b 1 + abc + ab(a + b + c)
+ = .
a2 (a + b + c) + 1 + abc b2 (a + b + c) + 1 + abc (a + b + c)2

2. Giải hệ phương trình


(
x2 + 4y2 + 4xy + 2x2 y2 = 11

1
.
3xy(x + 2y) + 31 = 9x + 18y + 13xy.

A
Câu 2. (2,0 điểm)

n
1. Tìm x, y nguyên dương thoả mãn 3x + 29 = 2y .
á
2. Với a, b, c là các số thực dương thoả mãn điều kiện 2(a + b + c) + ab + bc + ca = 9, tìm
To
giá trị lớn nhất của biểu thức
a+1 b+1 c+1
M= + + .
a2 + 10a + 21 b2 + 10b + 21 c2 + 10c + 21

cb

Câu 3. (3,0 điểm) Cho hình thoi ABCD (BADd < 90◦ ) có đường tròn nội tiếp (O). Các điểm
M, N lần lượt thuộc các cạnh CB,CD sao cho MN tiếp xúc (O) tại P, và tam giác CMN nhọn
không cân. Đường thẳng MN lần lượt cắt các đường thẳng AB, AD tại E, F. Gọi K, L theo thứ tự
lạ

là trực tâm của các tam giác BME và DNF.


1. Chứng minh rằng OP đi qua trung điểm I của KL.
u

2. Gọi H là trực tâm của tam giác CMN. Chứng minh rằng

OI EF 1
− =− .
CH 2MN 2

3. Gọi giao điểm của EK, FL với BD lần lượt là S, T . Gọi giao điểm của NS và MT là Q.
Đường tròn nội tiếp của tam giác CMN tiếp xúc với MN tại G. Chứng minh rằng hai
đường thẳng PQ và GH song song.
Câu 4. (1,0 điểm) Giả sử a1 , a2 , ..., a2021 là các số thực thoả mãn
a1 a2 a2021
+ + · · · + = 0.
a21 + 1 a22 + 1 a22021 + 1

Chứng minh rằng tồn tại số nguyên k (1 ≤ k ≤ 2021) sao cho


a1 2a2 kak 2k + 1
2
+ 2
+···+ 2
≤ .
a1 + 1 a2 + 1 ak + 1 8
24 TUYỂN TẬP ĐỀ THI VÀO LỚP 10 CHUYÊN TOÁN

1.17. ĐỀ THI TUYỂN SINH VÀO LỚP 10 TRƯỜNG THPT CHUYÊN


SƯ PHẠM HÀ NỘI NĂM 2021, VÒNG 2

1+ 5
Câu 1. (2,5 điểm) Cho α = .
2
a) Tìm một đa thức bậc hai Q(x) với hệ số nguyên sao cho α là nghiệm của Q(x).

b) Cho đa thức P(x) = x5 − x4 − x + 1. Tính giá trị của P(α).

Câu 2. (3,0 điểm) Cho A, B là hai điểm cố định nằm trên đường tròn tâm O, bán kính R. Giả sử
C là điểm cố định trên tia đối của tia BA. Một cát tuyến thay đổi qua C cắt đường tròn (O) tại
D và E (D nằm giữa C và E). Các đường tròn ngoại tiếp các tam giác BCD và ACE cắt nhau tại

1
giao điểm thứ hai M. Biết rằng bốn điểm O, B, M, E tạo thành tứ giác OBME. Chứng minh rằng

A
a) Tứ giác OBME nội tiếp.

b) CD ·CE = CO2 − R2 .

n
c) M luôn di chuyển trên một đường tròn cố định.
á
To
Câu 3. (2,0 điểm) Tìm tất cả các số nguyên dương N sa cho N có thể biểu diễn một cách duy
x2 + y
nhất ở dạng với x, y là hai số nguyên dương.
xy + 1

Câu 4. (2,5 điểm) Cho a, b, c là ba số nguyên dương sao cho mỗi số đó đều biểu diễn được ở
cb

dạng luỹ thừa của 2 với số mũ tự nhiên. Biết rằng phương trình bậc hai ax2 − bx + c = 0 (1) có
cả hai nghiệm đều là số nguyên. Chứng minh rằng hai nghiệm của phương trình (1) bằng nhau.
lạ
u

CÂU LẠC BỘ TOÁN A1, HOTLINE: 034 761 1986 - 035 290 3286 25

1.18. ĐỀ THI TUYỂN SINH VÀO LỚP 10 CHUYÊN TOÁN HÀ


NỘI, NĂM 2021
Câu 1.

1. Giải phương trình x2 + x + 2 − 2 x + 1 = 0.

2. Cho ba số thực a, b và c thoả mãn ab + bc + ca = 1. Chứng minh


a−b b−c c−a
2
+ 2
+ = 0.
1+c 1+a 1 + b2
Câu 2.
1. Tìm tất cả các cặp số nguyên (x, y) thoả mãn x2 + 5xy + 6y2 + x + 2y − 2 = 0.

1
A
2. Chứng minh rằng với mỗi số nguyên n, số n2 + n + 16 không chia hết cho 49.

Câu 3.

n
2
1. Cho số thực x khác 0 thoả mãn x + và x3 là số hữu tỉ. Chứng minh x là số hữu tỉ.
x
á
To
2. Cho các số thực không âm a, b và c thoả mãn a + b + c = 5. Chứng minh rằng 2a + 2ab +
abc ≤ 18.

Câu 4. Cho tam giác nhọn ABC nội tiếp đường tròn (O), với BAC d = 60◦ và AB < AC. Các

đường thẳng BO,CO lần lượt cắt các đoạn thẳng AC, AB tại M, N. Gọi F là điểm chính giữa của
cung BC lớn.
cb

(a) Chứng minh năm điểm A, N, O, M và F cùng thuộc một đường tròn.
lạ

(b) Gọi P, Q lần lượt là các giao điểm thứ hai của hai tia FN, FM với đường tròn (O). Gọi J là
giao điểm của đường thẳng BC và đường thẳng PQ. Chứng minh tia AJ là tia phân giác của
góc BAC.
d
u

(c) Gọi K là giao điểm của đường thẳng OJ và đường thẳng CF. Chứng minh AB vuông góc

với AK.

Câu 5. Cho A là một con có 100 phần tử của tập hợp {1, 2, 3, ..., 178}.

1. Chứng minh A chứa hai số tự nhiên liên tiếp.

2. Chứng minh với mọi số tự nhiên n thuộc tập hợp {2, 3, ..., 22}, tồn tại hai phần tử của A
có hiệu bằng n.
26 TUYỂN TẬP ĐỀ THI VÀO LỚP 10 CHUYÊN TOÁN

1.19. ĐỀ THI TUYỂN SINH VÀO LỚP 10 CHUYÊN TIN HÀ


NỘI, NĂM 2021
Câu 1. (2,0 điểm)

1. Giải phương trình 4 + 2x − x2 = x − 2.
(
x3 + 2 = 3y
2. Giải hệ phương trình 3 .
y + 2 = 3x

Câu 2. (2,0 điểm)

1. Chứng minh rằng với mỗi số nguyên n, số n2 + 3n + 16 không chia hết cho 25.

1
2. Tìm tất cả các số nguyên x và y thoả mãn x2 − xy − 2y2 + x + y − 5 = 0.

A
Câu 3. (2,0 điểm)

n
1. Cho a, b, c thực đôi một phân biệt. Chứng minh:

a+b

b+c
 
b+c

ác+a
 
c+a

a+b

To
+ + = −1.
a−b b−c b−c c−a c−a a−b

a b c
2. Cho biểu thức P = √ +√ +√ với a, b, c không âm và a2 + b2 +

1 + 2bc 1 + 2ca 1 + 2ab


c2 = 1. Tìm giá trị lớn nhất của P.
cb

Câu 4. (3,0 điểm) Cho tam giác nhọn ABC nội tiếp đường tròn (O) và AB < AC. Gọi I là tâm
đường tròn nội tiếp của tam giác ABC. Đường thẳng AI cắt đường tròn (O) tại điểm thứ hai M
lạ

(M khác A). Gọi D, E và F lần lượt là các hình chiếu của điểm I trên các cạnh BC,CA và AB.

(a) Chứng minh tam giác MBI là tam giác cân.


u

(b) Đường tròn ngoại tiếp tam giác AEF cắt đường tròn (O) tại điểm thứ hai P (P khác A).

Chứng minh P, M và D là ba điểm thẳng hàng.

(c) Gọi H là giao điểm của đường thẳng IP và đường thẳng EF. Chứng minh HD song song
với AM.

Câu 5. (1,0 điểm) Trên bàn có n viên kẹo. Hai bạn An và Bình cùng chơi một trò chơi như sau:
Hai bạn luân phiên lấy kẹo trên bàn, mỗi lần chỉ được lấy 1, 2, 3, 4 hoặc 5 viên kẹo và phải lấy
số viên kẹo khác với số viên kẹo của bạn còn lại vừa lấy ngay trước đó. Bạn đầu tiên không thể
thực hiện được lượt chơi của mình là người thua cuộc. Nếu An là người lấy kẹo trước,

1. Với n = 7, hãy chỉ ra chiến thuận chơi của Bình khiến An là người thua cuộc.

2. Với n = 22, hãy chỉ ra chiến thuận chơi của An khiến là người thua cuộc.
CÂU LẠC BỘ TOÁN A1, HOTLINE: 034 761 1986 - 035 290 3286 27

1.20. ĐỀ THI TUYỂN SINH VÀO LỚP 10 TRƯỜNG THPT CHUYÊN


KHOA HỌC TỰ NHIÊN NĂM 2022, VÒNG 1
Câu 1 (4 điểm).
1) Giải hệ phương trình (
6(xy + 5) + x3 y + 5x2 = 42
.
x3 + 5x2 y + 6x + 30y = 42

2) Giải phương trình


√ √ p
( 3 x + 6 + 3 3 − x)(2 + 3 3 (x + 6)(3 − x)) = 24.

1
Câu 2 (2 điểm).

A
1) Tìm tất cả các cặp số nguyên (x, y) thoả mãn đẳng thức

25y2 + 354x + 60 = 36x2 + 305y + (5y − 6x)2022 .

á n
2) Trên bàn có 8 hộp rỗng (trong các hộp không có viên bi nào). Người ta thực hiện các lần
To
thêm bi vào các hộp theo quy tắc sau: mỗi lần ta chọn ra 4 bất kỳ và bỏ vào một hộp 1
viên, một hộp 2 viên, hai hộp còn lại mỗi hộp 3 viên. Hỏi số lần thêm bi ít nhất có thể
nhận được số bi ở 8 hộp trên là 8 số tự nhiên liên tiếp?

Câu 3 (3 điểm).
cb

Cho hình chữ nhật ABCD (AB < AD) nội tiếp trong đường (O). Trên cạnh AD lấy hai điểm E
và F (E, F không trùng với A, D) sao cho E nằm giữa A và F, đồng thời ABE [ = 1 BOC.
d + DCF d
2
lạ

1) Chứng minh BE và CF cắt nhau tại một điểm nằm trên đường tròn (O).

2) Đường thẳng qua O song song với BC cắt BE,CF theo thứ tự tại M, N. Chứng minh rằng
u

[ + ADN
DAM [ + 1 AOD[ = 180◦ .

2
3) Dựng hình chữ nhật MNPQ sao cho NQ song song với BD, đồng thời MP song song với
AC. Chứng minh rằng đường tròn ngoại tiếp hình chữ nhật MNPQ tiếp xúc với đường tròn
(O).

Câu 4. (1 điểm)
Cho a, b, c là các số thực dương. Chứng minh rằng
2a a + b 6a + 2c 4a + 3b + c 32a
+ + + ≥ .
a+b a+c 3b + c b+c 2a + b + c
28 TUYỂN TẬP ĐỀ THI VÀO LỚP 10 CHUYÊN TOÁN

1.21. ĐỀ THI TUYỂN SINH VÀO LỚP 10 TRƯỜNG THPT CHUYÊN


KHOA HỌC TỰ NHIÊN NĂM 2022, VÒNG 2
Câu 1 (3.5 điểm).
1 1 1
1) Với a, b, c là những số thực dương thỏa mãn điều kiện + + = 1. Chứng minh rằng
a b c
  s
1 1 1 1 abc
+ + = .
2 a + bc b + ca c + ab (a + bc)(b + ca)(c + ab)

2) Giải hệ phương trình (


2x2 + 3xy + y2 = 6

1

3x + 2y + 1 = 2 2x + y + 6.

A
Câu 2 (2.5 điểm).

n
1) Tìm tất cả các cặp số nguyên dương (x, y) thỏa mãn đẳng thức

á
(x + y)(5x + y)3 + xy3 = (5x + y)3 + x2 y3 + xy4 .
To
2) Với a, b, c là những số thực dương thoả mãn các điều kiện sau
(
c ≤ b ≤ a ≤ 3, b2 + 2a ≤ 10, b2 + 2a + 2c ≤ 14,

(a2 + 1)(b2 + 1) + 4ab ≤ 2a3 + 2b3 + 2a + 2b.


cb

Tìm giá trị lớn nhất của biểu thức

P = 4a2 + b4 + 2b2 + 4c2 .


lạ

Câu 3 (3 điểm).
u

Cho tam giác ABC nhọn, không cân, nội tiếp đường tròn (O). Điểm P nằm trong tam giác ABC.
Gọi E, F lần lượt là hình chiếu vuông góc của P trên các cạnh CA, AB. Giả sử tứ giác BCEF nội

tiếp trong đường tròn (K).


1) Chứng minh rằng AP vuông góc với BC.

2) Chứng minh rằng AP = 2OK.

3) Đường thẳng qua P vuông góc với AP cắt đường tròn (O) tại hai điểm Q và R. Chứng minh
rằng đường tròn tâm A bán kính AP tiếp xúc với đường tròn ngoại tiếp tam giác KQR.
Câu 4. (1 điểm)
Cho các điểm A1 , A2 , ..., A30 theo thứ tự nằm trên một đường thẳng sao cho độ dài các đoạn
Ai Ai+1 bằng k (đơn vị dài), với k = 1, 2, ..., 29. Ta tô màu mỗi đoạn thẳng A1 A2 , A2 A3 , ..., A29 A30
bởi một trong ba màu (mỗi đoạn được tô bởi đúng một màu). Chứng minh rằng với mọi cách tô
màu, ta luôn chọn được hai số nguyên dương 1 ≤ i ≤ j ≤ 29 sao cho hai đoạn Ai Ai+1 , A j A j+1
được tô cùng màu và i − j là bình phương của một số nguyên dương.
CÂU LẠC BỘ TOÁN A1, HOTLINE: 034 761 1986 - 035 290 3286 29

1.22. ĐỀ THI TUYỂN SINH VÀO LỚP 10 TRƯỜNG THPT CHUYÊN


SƯ PHẠM HÀ NỘI NĂM 2022, VÒNG 2
Câu 1 (2,5 điểm).
p
3 √ p
3 √
a) Không sử dụng máy tính, hãy tính giá trị của biểu thức P = 7 + 5 2 + 7 − 5 2.

b) Cho đa thức P(x) = ax2 + bx + c. Chứng minh rằng nếu P(x) nhận giá trị nguyên với
mỗi số nguyên x thì ba số 2a, a + b, c đều là những số nguyên. Sau đó, chứng tỏ rằng nếu
2a, a + b, c là những số nguyên thì P(x) cũng nhận giá trị nguyên với mỗi số nguyên x.

Câu 2 (3.0 điểm). Cho tam giác đều ABC ngoại tiếp đường tròn (O). Cung nhỏ OB của đường
tròn ngoại tiếp tam giác tam giác OBC cắt đường tròn (O) tại điểm E. Tia BE cắt đường tròn

1
(O) tại điểm thứ hai F.

A
a) Chứng minh tia EO là tia phân giác của góc CEF.

b) Chứng minh tứ giác ABOF nội tiếp.

á n
c) Gọi D là giao điểm thứ hai của CE với đường tròn (O). Chứng minh ba điểm A, F, D thẳng
hàng.
To
Câu 3 (2,0 điểm). Cho a, b, c, d là các số nguyên dương thoả mãn ab = cd. Chứng minh rằng
N = a2022 + b2022 + c2022 + d 2022 là hợp số.

Câu 4 (2,0 điểm). Ta viết 10 số 0, 1, 2, . . . , 9 vào mười ô tròn trong hình bên, mỗi số được viết
cb

đúng một lần. Sau đó ta tính tổng ba số trên mỗi đoạn thẳng để nhận được 6 tổng. Có hay không
một cách viết 10 số như thế sao cho 6 tổng nhận được là bằng nhau?

Câu 5 (1,0 điểm).


lạ

a) Trong mặt phẳng cho 5 điểm sao cho không có ba điểm nào thẳng hàng. Chứng minh rằng
tồn tại ít nhất một tam giác tù có các đỉnh được lấy từ 5 điểm đã cho.
u

b) Trong mặt phẳng cho 2022 điểm sao cho không có ba điểm nào thẳng hàng. Chứng minh

rằng tồn tại ít nhất 2018 tam giác tù mà mỗi tam giác tù đó có các đỉnh được lấy từ 2022
điểm đã cho.
30 TUYỂN TẬP ĐỀ THI VÀO LỚP 10 CHUYÊN TOÁN

1.23. ĐỀ THI TUYỂN SINH VÀO LỚP 10 CHUYÊN TOÁN HÀ


NỘI, NĂM 2022
Câu 1 (2.0 điểm).

1. Giải phương trình x2 − 4x + 2 2x − 1 + 1 = 0.

2. Cho các số thực a, b và c thỏa mãn điều kiện ab + bc + ca = 1. Tính giá trị của biểu thức
a b c 2
P= 2
+ 2
+ 2

1+a 1+b 1+c a + b + c − abc

Câu 2 (2.0 điểm).

1
1) Chứng minh rằng nếu n là số tự nhiên lẻ thì 32n+1 − 7 chia hết cho 20.

A
2) Tìm tất cả cặp số nguyên dương (x, y) sao cho y(x2 + x + 1) = (x + 1)(y2 − 1)

Câu 3 (2.0 điểm).

n
m3 n3
1) Tìm hai số nguyên dương m, n sao cho và đều là các số nguyên tố.
m+ná m+n
To
2) Với a, b và c là các số thực không âm thoả mãn điều kiện a + b + c = 3, tìm giá trị lớn
nhất của biểu thức P = ab + 2bc + 3ca − 3abc.

Câu 4 (3.0 điểm).


Cho tam giác ABC nhọn với AB < AC. Đường tròn (I) nội tiếp tam giác ABC, tiếp xúc với ba
cb

cạnh BC,CA và AB lần lượt tại ba điểm D, E, F.

1) Gọi M là giao điểm của hai đường thẳng AI và DF. Chứng minh đường thẳng CM vuông
lạ

góc với đường thẳng AI.

2) Gọi N là giao điểm của hai đường thẳng AI và DE. Gọi K là trung điểm của đoạn thẳng
u

BC. Chứng minh tam giác KMN là tam giác cân.


3) Các tiếp tuyến tại M và N của đường tròn (K; KM) cắt nhau tại điểm S. Chứng minh
đường thẳng AS song song với đường thẳng ID.

Câu 5 (1.0 điểm).


Cho tập hợp A gồm 70 số nguyên dương không vượt quá 90. Gọi B là tập hợp gồm các số có
dạng x + y với x, y ∈ A và x, y không nhất thiết phân biệt,

1) Chứng minh 68 ∈ B.

2) Chứng minh B chứa 91 số nguyên liên tiếp.


CÂU LẠC BỘ TOÁN A1, HOTLINE: 034 761 1986 - 035 290 3286 31

1.24. ĐỀ THI TUYỂN SINH VÀO LỚP 10 CHUYÊN TIN HÀ


NỘI, NĂM 2022
Câu 1 (2.0 điểm).
p
1) Giải phương trình x2 − 2x + 2 = (x2 + 4)(x + 1).

2) Với a, b và c là các số thực dương thỏa mãn abc = 3, tính giá trị của biểu thức
1 1 1
P= + + .
a2 (b + c) + 3 b2 (c + a) + 3 c2 (a + b) + 3

Câu 2 (2.0 điểm).

1
1) Với p là số nguyên tố lớn hơn 3 , chứng minh số A = 5 p + p2 chia hết cho 6.

A
2) Tìm tất cả cặp số nguyên (x, y) thỏa mãn x3 − x2 y + 2x = 5x2 − 2y − 1.

Câu 3 (2.0 điểm).

n
1) Với a, b, c là các số thực dương thỏa mãn abc = 2, chứng minh
á
To
9
a2 + b2 + c2 − 3 (a + b − c) ≥ − .
4

2) Tìm tất cả các số nguyên dương a, b và c sao cho các phương trình x2 − 2ax + b = 0,

x2 − 2bx + c = 0 và x2 − 2cx + a = 0 đều có nghiệm là các số nguyên dương.


cb

Câu 4 (3.0 điểm).


Cho tam giác ABC với AB < AC nội tiếp đường tròn (O). Ba đường cao AD, BE và CF của tam
giác ABC cùng đi qua điểm H. Gọi I và K lần lượt là trung điểm của EF và BC.
lạ

AI HI
1) Chứng minh = .
u

AK HK

2) Chứng minh đường thẳng AH là tiếp tuyến của đường tròn ngoại tiếp tam giác HIK.

3) Gọi P là chân đường vuông góc kẻ từ điểm H đến đường thẳng EF. Chứng minh rằng
đường thẳng DP song song với đường thẳng AI.

Câu 5 (1.0 điểm).


Trên bảng có hai số tự nhiên m và n . An và Bình chơi một trò chơi như sau : Mỗi lượt chơi,
một bạn chọn một trong hai số trên bảng để xóa và viết lên bảng một số mới là hiệu không âm
của số vừa xóa với một ước số tự nhiên bất kỳ của số vừa xóa. Hai bạn luân phiên thực hiện lượt
chơi. Bạn đầu tiên không thể thực hiện được lượt chơi của mình là người thua cuộc , người còn
lại là người thắng cuộc . Biết rằng An là người thực hiện lượt chơi đầu tiên.

1) Với m = 2022, n = 2023 hãy chỉ ra chiến thuật chơi của An để An là người thắng cuộc.

2) Vơi m = 2022, n = 1981 hãy chỉ ra chiến thuật chơi của An để An là người thắng cuộc.
32 TUYỂN TẬP ĐỀ THI VÀO LỚP 10 CHUYÊN TOÁN

1.25. ĐỀ THI TUYỂN SINH VÀO LỚP 10 CHUYÊN TOÁN THÀNH


PHỐ H Ồ C HÍ M INH , NĂM 2022
p
Câu 1. Cho x, y là hai số thực thỏa mãn xy + (1 + x2 ) (1 + y2 ) = 1. Tính giá trị của biểu thức
 p  p 
M = x + 1 + y2 y + 1 + x2 .

Câu 2.

a) Giải phương trình x + 4 + |x| = x2 − x − 4.
 x

 = 2x − 1
y + z


y

1
b) Giải hệ phương trình = 3y − 1
z+x

A

 z
= 5z − 1


x+y

n
Câu 3. Cho hình vuông ABCD. Trên các cạnh BC,CD lần lượt lấy các điểm M và N sao cho
[ = 45◦ .
MAN
á
To
a) Chứng minh rằng MN tiếp xúc với đường tròn tâm A bán kính AB.

b) Kẻ MP k AN (P ∈ AB) và kẻ NQ k AM (Q ∈ AD). Chứng minh rằng AP = AQ.


Câu 4. Cho ba số thực dương a, b, c thỏa a + b + c = 3.


cb

a) Chứng minh rằng ab + bc + ca ≤ 3.


a b c
b) Tìm giá trị nhỏ nhất của biểu thức P = + + .
b2 + 1 c2 + 1 a2 + 1
lạ

Câu 5. Cho tam giác ABC nhọn (AB < AC) có các đường cao AD, BE,CF cắt nhau tại H. Đường
thẳng EF cắt đường thẳng BC tại I. Đường thẳng qua A vuông góc với IH tại K và cắt BC tại M.
u

BI CI
a) Chứng minh rằng tứ giác IKFC nội tiếp và = CD .

BD

b) Chứng minh M là trung điểm của BC.

Câu 6. Số nguyên dương n được gọi là "số tốt" nếu n + 1 và 8n + 1 đều là các số chính phương.

a) Hãy chỉ ra ví dụ ba "số tốt" lần lượt có 1, 2, 3 chữ số.

b) Tìm các số nguyên k thỏa mãn |k| ≤ 10 và 4n + k là hợp số với mọi n là "số tốt".
CÂU LẠC BỘ TOÁN A1, HOTLINE: 034 761 1986 - 035 290 3286 33

1.26. ĐỀ THI TUYỂN SINH VÀO LỚP 10 TRƯỜNG THPT CHUYÊN


KHOA HỌC TỰ NHIÊN NĂM 2023, VÒNG 1
Câu 1 (3,5 điểm).
1) Giải phương trình
p √ p √
x2 + 6x + 2023 + x + 3 = x2 + 5x + 2025 + 5.

2) Giải hệ phương trình



(x + 6y) (3x + 2y) = 12
2x3 + 6y3 + 15x2 y + 19y2 x + x + 6y = 12.

1
A
Câu 2 (2,5 điểm).
1) Giả sử n là số nguyễn sao cho 3n3 − 1011 chia hết cho 1008. Chứng minh rằng n − 1 chia hết
cho 48 .

á n
2) Với a, b, c là các số dương thỏa mãn điều kiện ab + bc + ca = 1. Chứng minh rằng
To
   
1 1 1
1+ 1+ 1+ > 4.
1 + a2 1 + b2 1 + c2

Câu 3 (3 điểm). Cho hai đường tròn (O) và (O0 ) cắt nhau tại A và B sao cho O nằm ngoài (O0 )

và O0 nằm ngoài (O). Trên đường tròn (O) lấy điểm P di chuyển sao cho P nằm trong đường
cb

tròn (O0 ). Đường thẳng AP cắt (O0 ) tại C khác A.

1) Chứng minh rằng hai tam giác OBP và O0 BC đồng dạng.


lạ

2) Gọi Q là giao điểm của hai đường thẳng OP và O0C. Chứng minh rằng

d = 90◦ .
d + ABP
QBC
u

3) Lấy điểm D thuộc (O) sao cho AD vuông góc với O0C. Chứng minh rằng trung điểm của
đoạn thẳng DQ luôn nằm trên một đường tròn cố định khi P thay đổi.

Câu 4 (1 điểm). Giả sử A là tập hợp con của tập hợp gồm 30 số tự nhiên đầu tiên {0, 1, 2, 3, . . . , 29}
sao cho với k nguyên bất kỳ, a, b ∈ A bất kỳ (có thể a = b ) thì a + b + 30k không là tích của hai
số nguyên liên tiếp. Chứng minh rằng số phần tử của tập hợp A nhỏ hơn hoặc bằng 10.
34 TUYỂN TẬP ĐỀ THI VÀO LỚP 10 CHUYÊN TOÁN

1.27. ĐỀ THI TUYỂN SINH VÀO LỚP 10 TRƯỜNG THPT CHUYÊN


KHOA HỌC TỰ NHIÊN NĂM 2023, VÒNG 2
Câu 1 (3.5 điểm).
1) Giải phương trình p p
2x + 1 + 2 4x2 + 6x = 4 5x − x2

2) Giải hệ phương trình 


xy(x + y) = 30 √
x3 + y3 = 30 + 3 x + y + 120

Câu 2 (2.5 điểm).

1
1) Tìm tất cả các cặp số nguyên dương (x; y) thỏa mãn

A
4x + (1 + 3y ) (1 + 7y ) = 2x (3y + 7y + 2)

n
2) Với x, y, z là những số thực dương, tìm giá trị nhỏ nhất của biểu thức:

x14 − x6 + 3 á
y14 − y6 + 3 z14 − z6 + 3
To
M= + +
x2 y2 + zx + zy y2 z2 + xy + xz z2 x2 + yz + yx

Câu 3 (3 điểm). Cho tam giác ABC nhọn vói AB < AC nội tiếp trong đường tròn (O) có tiếp

tuyến tại A của (O) cắt BC ở T sao cho T B > BC. Gọi P và E lần lượt là trung điểm của TA và
TC.
cb

1) Chứng minh rằng tứ giác APEB nội tiếp.

2) Gọi giao điểm thứ hai của AE với (O) là F. Láy G thuộc (O) sao cho FG song song vói AC.
lạ

Chửng minh rằng ATdG = TAF.


d

3) Gọi H là trực tâm của tam giác ABC, D là giao điểm của AH và BC. M là trung điểm BC.
u

K đối xứng với A qua BC. N thuộc đường thẳng AM sao cho KN song song với HM. Lấy S

thuộc BC sao cho NS ⊥ NK. Dựng R thuộc tia AK sao cho AR · AH = AD2 . Q là điểm sao
cho PQ ⊥ AS và SQ ⊥ AO. Chứng minh rẳng điểm đối xứng của A qua QR thuộc đường tròn
đường kinh DN.

Câu 4 (1 điểm). Viết 100 số nguyên dương đầu tiên 1, 2, ..., 100 vào một bảng ô vuông kích
thước 10 × 10 một cách tuỳ ý sao cho mỗi ô vuông được viết đúng một số. Chứng minh rằng
tồn tại hai ô kề nhau (hai ô có cạnh chung) mà hai số được viết ở hai ô này có hiệu lớn hơn hoặc
bằng 10.
CÂU LẠC BỘ TOÁN A1, HOTLINE: 034 761 1986 - 035 290 3286 35

1.28. ĐỀ THI TUYỂN SINH VÀO LỚP 10 TRƯỜNG THPT CHUYÊN


SƯ PHẠM HÀ NỘI NĂM 2023, VÒNG 2
Câu 1. (2,5 điểm)
a) Chứng minh rằng tích của bốn số nguyên liên tiếp cộng với 1 là bình phương của một số
nguyên.
(
2xy − x = 10,
b) Tìm các cặp số nguyên của hệ phương trình
x + y + xy = 11.
Câu 2. (3,0 điểm)
a) Cho a, b là các số thực không âm và c là số thực dương thoả mãn

1
√ √ √ √
a − a + b − c = b + c.

A
√ √ √ √
Chứng minh rằng 3 a + 3 b − 3 c = 3 a + b − c.
√ √

n
3+ a
b) Tìm các số nguyên dương a, b sao cho √ √ là số hữu tỉ.
5+ b
á
To
Câu 3. (2,5 điểm) Cho tam giác ABC có đường tròn nội tiếp (I) tiếp xúc với ba cạnh BC, CA,
AB tại D, E, G. Hai đường thẳng DE, DG lần lượt cắt đường phân giác ngoài BAC
d tại M, N. Hai
đường thẳng MG, NE cắt nhau tại P. Chứng minh rằng

a) EG k MN.
cb

b) Điểm P thuộc đường tròn (I).


Câu 4. (1,0 điểm) Bảy lục giác đều được sắp xếp và tô màu bằng hai màu trắng, đen như ở Hình
1. Mỗi lần cho phép chọn ra một lục giác đều, đổi màu của lục giác đó và của tất cả các lục giác
lạ

đều chung cạnh với lục giác đó (trắng thành đen và đen thành trắng). Chứng minh rằng dù có
thực hiện cách làm trên bao nhiêu lần đi nữa, cũng không thể nhận được các lục giác đều được
ô màu như ở Hình 2.
u

Hình 1 Hình 2
Câu 5. (1,0 điểm) Chứng minh rằng tồn tại số nguyên dương n > 102023 sao cho tổng tất cả các
số nguyên tố nhỏ hơn n là một số nguyên tố cùng nhau với n.
36 TUYỂN TẬP ĐỀ THI VÀO LỚP 10 CHUYÊN TOÁN

1.29. ĐỀ THI TUYỂN SINH VÀO LỚP 10 CHUYÊN TOÁN HÀ


NỘI, NĂM 2023
Câu 1 (2,0 điểm).
1) Giải phương trình √ √
x − 3 − 2x − 7 = 2x − 8

2) Cho a, b, c là các số thực khác 0 thỏa mãn điều kiện a2 − c2 = c, c2 − b2 = b và b2 − a2 = a.


Chứng minh
(a − b)(b − c)(c − a) = 1.

Câu 2 (2,0 điểm).

1
1) Cho ba số nguyên a, b và c thỏa mãn a2 + b2 + c2 − 2abc chia hết cho 6 . Chứng minh abc

A
chia hết cho 54.

2) Tìm tất cả cặp số nguyên dương (x, y) thỏa mãn x3 y − x2 y − 4x2 + 5xy − y2 = 0.

n
Câu 3 (2,0 điểm).
á
1) Tìm tất cả cặp số nguyên (x, y) sao cho xy là số chính phương và x2 + xy + y2 là số nguyên
To
tố.

2) Với các số thực không âm a, b và c thỏa mãn a + 2b + 3c = 1, tìm giá trị lớn nhất và giá trị
nhỏ nhất của biểu thức P = (a + 6b + 6c)(a + b + c).

Câu 4 (3 điểm). Cho tam giác ABC có ba góc nhọn (AB < AC), nội tiếp đường tròn (O). Ba
cb

đường cao AD, BE và CF của tam giác ABC cùng đi qua điểm H. Đường thẳng EF cắt đường
thẳng AD tại điểm Q. Gọi M và I lần lượt là trung điểm của các đoạn thẳng BC và AH. Đường
thẳng IM cắt đường thẳng EF tại điểm K.
lạ

1) Chứng minh rằng tam giác AEK đồng dạng với tam giác ABM.
u

2) Đường thẳng EF cắt đường thẳng BC tại điểm S, đường thẳng SI cắt đường thẳng MQ tại
điểm T . Chứng minh rằng bốn điểm A, T, H và M cùng thuộc một đường tròn.

3) Tia T H cắt đường tròn (O) tại điểm P. Chứng minh rằng ba điểm A, K và P thẳng hàng.

Câu 5 (1 điểm). Cho 2023 điểm nằm trong một hình vuông cạnh 1. Một tam giác đều được gọi
là phủ điểm M nếu điểm M nằm trong tam giác hoặc nằm trên cạnh của tam giác.
1
1) Chứng minh tồn tại tam giác đều cạnh √ phủ ít nhất 253 điểm trong 2023 điểm đã cho.
2
11
2) Chứng minh tồn tại tam giác đều cạnh phủ ít nhất 506 điểm trong 2023 điểm đã cho.
12
CÂU LẠC BỘ TOÁN A1, HOTLINE: 034 761 1986 - 035 290 3286 37

1.30. ĐỀ THI TUYỂN SINH VÀO LỚP 10 CHUYÊN TIN HÀ


NỘI, NĂM 2023
Câu 1 (2,0 điểm).

1) Giải phương trình 2x + 2 = (5 − x) 3x − 2.

x + y + 3xy = 9
2) Giải hệ phương trình .
x3 + y3 = 9

Câu 2 (2,0 điểm).


2 +2
1) Cho p là số nguyên tố lớn hơn 3. Chứng minh số A = 2 p − 8 chia hết cho 21 .

2) Tìm tất cả các số nguyên x và y thỏa mãn x3 − y3 = 2(x − y)2 + 17.

1
A
Câu 3 (2,0 điểm).
1) Cho đa thức f (x) = x4 + 2x3 + 3x2 + 2022x + 2023. Chứng minh đa thức f (x) không có

n
nghiệm hữu tỉ.

á
2) Với các số thực a, b và c thỏa mãn (a + 1)(b + 1)(c + 1) = (a − 1)(b − 1)(c − 1), tìm giá trị
To
nhỏ nhất của biểu thức A = |a| + |b| + |c|.

Câu 4 (3 điểm). Cho hai đường tròn (O; R) và (O0 ; R0 ) cắt nhau tại hai điểm phân biệt A và
B (R < R0 < OO0 ). Gọi PQ là tiếp tuyến chung của hai đường tròn (O) và (O0 ) với P ∈ (O)

và Q ∈ (O0 ). Đường thẳng PQ cắt đường thẳng OO0 tại điểm S. Qua điểm S vẽ một đường
thẳng cắt đường tròn (O) tại hai điểm E, F và cắt đường tròn (O0 ) tại hai điểm G, H sao cho
cb

SE < SF < SG < SH.

1) Chứng minh đường thẳng OE song song với đường thẳng O0 G.


lạ

2) Chứng minh SA2 = SP · SQ.

3) Tiếp tuyến tại điểm A của đường tròn (O) cắt đường thẳng OO0 tại điểm M. Tiếp tuyến tại
u

điểm A của đường tròn (O0 ) cắt đường thẳng OO0 tại điểm N. Đường thẳng ME cắt đoạn

EA2 IA
thẳng AB tại điểm I. Chứng minh = và ba điểm N, I, H là ba điểm thẳng hàng.
EB2 IB
Câu 5 (1 điểm). Trên bàn có hai túi kẹo: túi thứ nhất có 18 viên kẹo, túi thứ hai có 21 viên kẹo.
An và Bình cùng chơi một trò chơi như sau: mỗi lượt chơi, một bạn sẽ lấy đi 1 viên kẹo từ một
túi bất kỳ hoặc là mỗi túi lấy đi 1 viên kẹo. Hai bạn luân phiên thực hiện lượt chơi của mình.
Người đầu tiên không thể thực hiện được lượt chơi của mình là người thua cuộc, người còn lại
là người thắng cuộc. Nếu An là người lấy kẹo trước, hãy chỉ ra chiến thuật chơi của An để An là
người thắng cuộc.
38 TUYỂN TẬP ĐỀ THI VÀO LỚP 10 CHUYÊN TOÁN

1
A
á n
To

cb
lạ
u

Chương 2

Lời giải

1
A
á n
To

cb
lạ
u

39
40 TUYỂN TẬP ĐỀ THI VÀO LỚP 10 CHUYÊN TOÁN

2.1. ĐỀ THI TUYỂN SINH VÀO LỚP 10 TRƯỜNG THPT CHUYÊN


KHOA HỌC TỰ NHIÊN NĂM 2018, VÒNG 1
Câu 1
1. Giải phương trình p √
x2 − x + 2 x3 + 1 = 2 x + 1.

2. Giải hệ phương trình (


xy + y2 = 1 + y
x2 + 2y2 + 2xy = 4 + x

1
Lời giải.

A
1. Điều kiện xác định: x ≥ −1.
Phương trình đã cho tương đương với

n
q √
2
á
⇐⇒ x − x + 1 + 2 (x + 1)(x2 − x + 1) + x + 1 = x + 1 + 2 x + 1 + 1
To
p √ 2 √ 2
⇐⇒ x2 − x + 1 + x + 1 = x+1+1 .

Từ đó có 2 trường hợp xảy ra.


√ √ √
Trường hợp 1. x2 − x + 1 + x + 1 = x + 1 + 1
cb


Phương trình ⇐⇒ x2 − x + 1 = 1 ⇐⇒ x2 − x = 0 ⇐⇒ x(x − 1) = 0.
Như vậy, x = 0 ∨ x = 1.
lạ

√ √ √
Trường hợp 2. x2 − x + 1 + x + 1 = − x + 1 − 1
Trường hợp này vô nghiệm vì VT ≥ 0 và VP < 0.
u

Kết luận. S = {0; 1}.


2. Nhân phương trình đầu với 2 rồi cộng với phương trình thứ hai, ta được

(x + 2y)2 − (x + 2y) − 6 = 0.

Giải tìm được x + 2y = 3 hoặc x + 2y = −2. Xét hai trường hợp

Trường hợp 1. Với x + 2y = 3 thì x = 3 − 2y, thay vào phương trình đầu, ta được

y2 − 2y + 1 = 0.

Giải tìm được (x; y) = (1; 1).

Trường hợp 2. Với x + 2y = −2 thì x = −2 − 2y, thay vào phương trình đầu, ta được

y2 + 3y + 1 = 0.
CÂU LẠC BỘ TOÁN A1, HOTLINE: 034 761 1986 - 035 290 3286 41

√ ! √ !
√ −3 − 5 √ −3 + 5
Giải tìm được (x; y) = 1 + 5; và (x; y) = 1 − 5; .
2 2

Câu 2
1. Tìm tất cả các cặp số nguyên (x, y) thỏa mãn

(x + y)(3x + 2y)2 = 2x + y − 1.
r
√ b
2. Với a, b là các số thực dương thỏa mãn a + 2b = 2 + , tìm giá trị nhỏ nhất của
3
biểu thức
a b

1
M=√ +√ .
a + 2b b + 2a

A
Lời giải.

n
1. Viết lại phương trình dưới dạng (x + y)(3x + 2y)2 + x + y = 3x + 2y − 1 hay
á
3x + 2y − 1
To
x+y = .
(3x + 2y)2 + 1
t −1
Đặt t = 3x + 2y thì x + y = . Ta phải có t − 1 chia hết cho t 2 + 1, suy ra (t − 1)(t +
t2 + 1

1) = t 2 − 1 chia hết cho t 2 + 1. Suy ra 2 chia hết cho t 2 + 1, nghĩa là t 2 + 1 = 1 hoặc


cb

t 2 + 1 = 2. Xét ba khả năng

Trường hợp 1. t = 0 thì x + y = −1, 3x + 2y = 0, tìm được x = 2, y = −3.


lạ

Trường hợp 2. t = 1 thì x + y = 0, 3x + 2y = 1, tìm được x = 1, y = −1.


Trường hợp 3. t = −1 thì x + y = −1, 3x + 2y = −1 tìm được x = 1, y = −2.
u

Kết luận. Phương trình đã cho có ba nghiệm nguyên (2; 3), (1; −1) và (1; −2).

2. Áp dụng bất đẳng thức cộng mẫu số, ta có


b a2 b2 b2 (a + b + b)2
M+ √ = √ + √ + √ ≥ √ √ √ .
b + 2b a a + 2b b b + 2a b b + 2b a a + 2b + b b + 2a + b b + 2b
Lại có
 √ √ √ 2 √ p √ p √ p 2
a a + 2b + b b + 2a + b b + 2b = a a2 + 2ab + b b2 + 2ab + b b2 + 2b2
≤ (a + b + b)(a2 + 2ab + b2 + 2ab + b2 + 2b2 )
= (a + 2b)3 .

Do đó, r
b √
M+ ≥ a + 2b,
3
42 TUYỂN TẬP ĐỀ THI VÀO LỚP 10 CHUYÊN TOÁN

r
√ b
kéo theo M ≥ a + 2b − = 2. Dấu bằng xảy ra khi a = b = 3.
3

Nhận xét. Xét bài toán sau

Cho các số thực a, b, c thỏa mãn điều kiện


√ √ √
a + 2b + b + 2c + c + 2a = 9.

Tìm giá trị nhỏ nhất của biểu thức


a b c
M=√ +√ +√ .
a + 2b b + 2c c + 2a

1
A
Như vậy câu 2.2 trong đề là một trường hợp đặc biệt của bài toán trên khi cho b = c.

Câu 3 á n
To
Cho tam giác ABC có đường tròn nội tiếp (I) tiếp xúc với các cạnh BC,CA, AB lần lượt
tại các điểm D, E, F. Gọi K là hình chiếu vuông góc của B trên đường thẳng DE, M là
trung điểm của đoạn thẳng DF.

1. Chứng minh rằng hai tam giác BKM và DEF đồng dạng.
cb

2. Gọi L là hình chiếu vuông góc của C trên đường thẳng DF, N là trung điểm của
đoạn thẳng DE. Chứng minh rằng hai đường thẳng MK và NL song song.
lạ

3. Gọi J, X lần lượt là trung điểm của các đoạn thẳng KL và ID. Chứng minh rằng
đường thẳng JX vuông góc với đường thẳng EF.
u

Lời giải.
1. Do 4BFD cân tại B có M là trung điểm của đáy DF nên BM ⊥ DF. Từ đó suy ra tứ giác
BMDK nội tiếp. Như vậy, kết hợp với giả thiết BC tiếp xúc với (I) tại D, ta được

∠BKM = ∠BDM = ∠DEF; ∠BMK = ∠BDK = ∠CDE = ∠DFE

Vậy 4BKM ∼ 4DEF.

2. Do MN là đường trung bình của tam giác DEF nên MN k EF. Từ đó ta có

∠DMK = ∠DBK = 90◦ − ∠BDK = 90◦ − ∠CDE = 90◦ − ∠DFE = 90◦ − ∠DMN

Suy ra ∠KMN = 90◦ , hay KM ⊥ MN. Chứng minh tương tự như trên ta cũng được LN ⊥
NM. Như vậy, ta thu được MK k NL.
CÂU LẠC BỘ TOÁN A1, HOTLINE: 034 761 1986 - 035 290 3286 43

F
I
N
M P
X

1
A
B D C

n
J
á
To
L

3. Gọi P là trung điểm của MN. Khi đó, ta được JP là đường trung bình của hình thang
MKLN. Mặt khác, do ∠KMN = ∠LNM = 90◦ nên JP ⊥ MN. Từ đó suy ra JP là trung
cb

trực của MN và JM = JN.


Trong khi đó, do ∠IMD = ∠IND = 90◦ nên tứ giác IMDN nội tiếp đường tròn đường
kính ID. Suy ra XM = XN. Kết hợp với JM = JN chứng minh ở trên ta được JX là trung
lạ

trực của MN. Vậy JX ⊥ MN, mà MN k EF nên JX ⊥ EF.


u

Câu 4

Trên mặt phẳng cho hai điểm P, Q phân biệt. Xét 10 đường thẳng nằm trong mặt phẳng
trên thỏa mãn tính chất sau:

i) không có hai đường thẳng nào song song hoặc trùng nhau.

ii) mỗi đường thẳng đều đi qua P hoặc Q, không có đường thẳng nào đi qua cả P và
Q.

Hỏi 10 đường thẳng trên có thể chia mặt phẳng thành tối đa bao nhiêu miền? Hãy giải
thích.

Lời giải. Giả sử có x đường kẻ từ P (gọi là x đường gốc P) và y đường kẻ từ Q (gọi là y đường
gốc Q) với x + y = 10. Ta thấy x đường gốc P chia mặt phẳng thành 2x miền. Nếu có thêm một
đường gốc Q thì để số miễn nhiều nhất, đường này sẽ cắt x đường gốc P, tạo thêm x + 1 miền
44 TUYỂN TẬP ĐỀ THI VÀO LỚP 10 CHUYÊN TOÁN

mới. Số miền mới sẽ bằng là 2x + (x + 1). Nếu có thêm một đường gốc Q nữa thì đường này
sẽ cắt x đường gốc P và một đường gốc Q, tạo thêm x + 2 miền mới, cứ thế, cứ thế...Thế thì số
miền sẽ là

2x + (x + 1) + (x + 2) + · · · + (x + 2) = 2x + (x + 2)y − 1 = xy + 19.

(x + y)2
Lại có xy ≤ = 25, nên số miền sẽ không vượt quá 44 miền. Để đạt được điều này thì
4
phải có 5 đường gốc P và 5 đường gốc Q, trong đó không có hai đường thẳng nào song song.

1
A
á n
To

cb
lạ
u

CÂU LẠC BỘ TOÁN A1, HOTLINE: 034 761 1986 - 035 290 3286 45

2.2. ĐỀ THI TUYỂN SINH VÀO LỚP 10 TRƯỜNG THPT CHUYÊN


KHOA HỌC TỰ NHIÊN NĂM 2018, VÒNG 2
Câu 1
1. Giải hệ phương trình
(
xy(x + y) = 2
x3 + y3 + x3 y3 + 7(x + 1)(y + 1) = 31

2. Giải phương trình


√ √

1
p
9 + 3 x(3 − 2x) = 7 x + 5 3 − 2x.

A
Lời giải.

n
1. Đặt a = x+y, b = xy thì hệ có thể viết lại thành ab = 2 và a3 −3ab+b3 +7(a+b)+7 = 31.
Từ đó, ta có á
To
(a + b)3 + (a + b) − 30 = 0.
Giải tìm được a + b = 3 và ab = 2. Suy ra (x; y) = (1; 1)
Hệ đã cho có một nghiệm (1, 1).

3 √ √
cb

2. Điều kiện: 0 ≤ x ≤ . Đặt a = x, b = 3 − 2x với a, b ≥ 0 thì theo đề ta có 2a2 + b2 = 3


2
và 9 + 3ab = 7a + 5b. Cộng cả hai phương trình lại, ta được
lạ

2a2 + 3ab + b2 − 7a − 5b + 6 = 0.

Tới đây, phân tích vế trái thành nhân tử, ta được


u

(a + b − 2)(2a + b − 3) = 0.

Xét hai trường hợp

√ √
Trường hợp 1. Nếu a + b = 2 thì x + 3 − 2x = 2, tới đây bình phương hai lần tìm được
1
x = 1 và x = .
9

Trường hợp 2. Nếu 2a + b = 3, tương tự như trường hợp 1 thì trường hợp này có duy nhất
nghiệm x = 1.

1
Tóm lại, phương trình đã cho có hai nghiệm x = 1 và x = .
9
46 TUYỂN TẬP ĐỀ THI VÀO LỚP 10 CHUYÊN TOÁN

Câu 2
1. Cho x, y là các số nguyên sao cho x2 − 2xy − y và xy − 2y2 − x đều chia hết cho 5.
Chứng minh rằng 2x2 + y2 + 2x + y cũng chia hết cho 5.

2. Cho a1 , a2 , · · · , a50 là các số nguyên thỏa mãn 1 ≤ a1 ≤ a2 ≤ · · · ≤ a50 ≤ 50 và


a1 + a2 + · · · + a50 = 100. Chứng minh rằng từ các số đã cho có thể chọn được một
vài số có tổng bằng 50.

Lời giải.
1. Từ giả thiết suy ra x2 − 2xy − y − (xy − 2y2 − x) = x(x − y) − 2y(x − y) + x − y = (x − y)(x −
2y + 1) chia hết cho 5. Tới đây, có hai trường hợp

1
A
Trường hợp 1. Nếu x − y chia hết cho 5 thì từ x2 − 2xy − y = x(x − y) − y(x + 1) chia hết
cho 5 ta suy ra hoặc x, y đều chia hết cho 5 hoặc x, y chia 5 dư 4. Trong mọi tình huống, ta
đều suy ra đpcm.

n
Trường hợp 2. Nếu x − 2y + 1 chia hết cho 5 thì từ x2 − 2xy − y = x(x − 2y + 1) − (x + y)
á
chia hết cho 5. Từ đó x + y chia hết cho 5, kết hợp x − 2y + 1 chia hết cho 5, ta suy ra x
To
chia 5 dư 3, còn y chia 5 dư 2. Từ đây 2x2 + y2 + 2x + y ≡ 3 + 4 + 1 + 2 (mod 5), tức là
2x2 + y2 + 2x + y chia hết cho 5.

2. Xét 50 tổng Si = a1 + a2 + · · · + ai , với i = 1, 2, · · · , 50. Xét hai trường hợp


cb

Trường hợp 1. Nếu trong 50 tổng nói trên có hai tổng có cùng dư khi chia cho 50, giả sử
là Si , S j với i > j thì Si − S j chia hết cho 50 và rõ ràng 0 < Si − S j < 100, nên Si − S j = 50.
lạ

Từ đó suy ra điều phải chứng minh.

Trường hợp 2. Nếu 50 tổng trên cho số dư đôi một khác nhau khi chia cho 50. Thay vị trí
u

của a50 và a49 trong dãy tổng, thì ta được hai tổng thứ 49 và 50 mới, giả sử là S49 ∗ , S∗ . Xét
50
∗ ∗

dãy tổng mới S1 , S2 , · · · , S48 , S49 , S50 mà có hai tổng có cùng số dư khi chia cho 50 thì trở
lại trường hợp 1, kể cả trường hợp S49 ∗ − S∗ = a − a chia hết cho 50 khi đó ta phải có
50 50 1
a50 = a49 = · · · = a1 = 2 và ta cũng có điều phải chứng minh. Nếu ngược lại thì 50 tổng
này có số dư đôi một khác nhau khi chia cho 50, trong đó 48 tổng đầu tiên không đổi nên
ta phải có tập số dư của hai tổng S49 ∗ , S∗ khi chia cho 50 trùng với tập số dư của hai tổng
50
S49 , S50 . Xét hai trường hợp nhỏ
∗ ≡ S
(i) Nếu S49 49 (mod 50) thì ta có a50 ≡ a49 (mod 50). Lặp lại tương tự ta sẽ có
a50 ≡ a49 ≡ · · · ≡ a1 (mod 50) và từ đó ai = 2 với mọi i = 1, 2, · · · , 50. Từ đó ta có đpcm.
∗ ≡S ∗
(ii) Nếu S49 50 (mod 50) thì S50 − S49 = a49 chia hết cho 50, nghĩa là a49 phải bằng
50. Trường hợp này không xảy ra được.

Trong mọi trường hợp, ta có điều phải chứng minh.


CÂU LẠC BỘ TOÁN A1, HOTLINE: 034 761 1986 - 035 290 3286 47

Nhận xét. Thực ra, ta có thể trình bày bài toán trên gọn hơn, nhưng chúng tôi chọn
cách trình bày thế này thể hiện mạch tư duy khi giải bài toán. Đề bài có thể thay
dữ kiện a50 ≤ 50 bằng dữ kiện a50 khác 51. Có thể thay 50 bằng n và 100 bằng 2n.

Câu 3
Cho ngũ giác lồi ABCDE nội tiếp đường tròn (O) có CD song song với BE. Hai đường
chéo CE và BD cắt nhau tại P. Điểm M thuộc đoạn thẳng BE sao cho ∠MAB = ∠PAE.
Điểm K thuộc đường thẳng AC sao cho MK song song với AD, điểm L thuộc đường thẳng
AD sao cho ML song song với AC. Đường tròn ngoại tiếp tam giác KBC lần lượt cắt BD
và CE tại Q, S (Q khác B, S khác C)

1
A
1. Chứng minh rằng ba điểm K, M, Q thẳng hàng.

2. Đường tròn ngoại tiếp tam giác LDE lần lượt cắt BD,CE tại T, R (T khác D, R khác

n
E). Chứng minh rằng năm điểm M, S, Q, R, T cùng thuộc một đường tròn.

á
3. Chứng minh rằng đường tròn ngoại tiếp của tam giác PQR tiếp xúc với đường tròn
To
(O).

A
cb

L
K
lạ

B E
u

M

T
O
S

P
Q
R

C D

Lời giải.
48 TUYỂN TẬP ĐỀ THI VÀO LỚP 10 CHUYÊN TOÁN

1. Do tứ giác KBCQ nội tiếp và MK k AD nên ta có

∠CKQ = ∠CBQ = ∠CBD = ∠CAD = ∠CKM

Từ đó suy ra K, M, Q thẳng hàng.

2. Do các tứ giác BCQS và DET R là các tứ giác nội tiếp nên ta có

∠RSQ = ∠CSQ = ∠CBD = ∠CED = ∠RED = ∠RT Q

Mặt khác, do MK k AD, ML k AC nên

∠RMQ = ∠CAD = ∠CED = ∠RED = ∠RT Q

Như vậy, ∠RSQ = ∠RT Q = ∠RMQ. Vậy năm điểm M, S, Q, R, T cùng thuộc một đường

1
tròn.

A
Ghi chú. Hơn nữa, trong mô hình của ý 2, ta còn có thể thu được một kết quả nữa

n
như sau:

á
Đường tròn đi qua năm điểm M, S, Q, R, T tiếp xúc với đường thẳng BE tại
To
M.

3. Để đơn giản hơn, ta có thể phát biểu lại bài 3.3 như sau:
cb

Bài toán. Cho ngũ giác lồi ABCDE nội tiếp đường tròn (O) có CD song song với
lạ

BE. Hai đường chéo CE và BD cắt nhau tại P. Điểm M thuộc đoạn thẳng BE sao
cho ∠MAB = ∠PAE. Lấy điểm R, Q tương ứng nằm trên các đường thẳng PC, PD
sao cho MR k AC và MQ k AD. Chứng minh rằng đường tròn ngoại tiếp của tam
u

giác PQR tiếp xúc với đường tròn (O).


Đặt AM, AP lần lượt cắt đường tròn (O) tại X,Y (X,Y khác A). Vì ∠MAB = ∠PAE nên
XY k BE k CD.
Trước hết, ta sẽ chứng minh X ∈ (PQR). Thật vậy, do MR k AC nên ∠RMX = ∠CAX =
∠CEX = ∠REX nên tứ giác RMEX nội tiếp, vì vậy ∠RXM = ∠REM. Chứng minh tương
tự ta cũng được ∠QXM = ∠QBM. Từ đó,

∠RXQ = ∠RXM + ∠QXM = ∠REM + ∠QBM = 180◦ − ∠BPE = 180◦ − ∠CPD

Vậy tứ giác PQXR nội tiếp, hay X ∈ (PQR).


Dựng tia tiếp tuyến Xt của đường tròn (O). Khi đó ta có

∠RXt = ∠AXt − ∠AXR = ∠AEX − ∠MEP = ∠AEB + ∠PEX


CÂU LẠC BỘ TOÁN A1, HOTLINE: 034 761 1986 - 035 290 3286 49

B E
M
O

P
Q

H R
t

1
C D

A
X Y

n
Mặt khác, do tính đối xứng qua trung trực của CD nên ta có
á
∠RPX = ∠CPX = ∠DPY = ∠PDA + ∠PAD = ∠ADB + ∠CAX = ∠AEB + ∠PEX
To
Như vậy, ∠RXt = ∠RPX, vì vậy nên đường tròn (PQR) tiếp xúc với Xt. Từ đó suy ra
đường tròn (PQR) tiếp xúc với (O).

cb

Câu 4
Cho a, b, c là các số thực dương. Chứng minh rằng
lạ

r r ! 
ab bc 1 1
+ √ +√ ≤ 2.
a+b b+c a+b b+c
u

Lời giải. Áp dụng bất đẳng thức AM−GM và Cauchy−Schwarz, ta có


√ √ √ √ √ √ √ √
ab bc ab + bc ab bc ab + bc
VT = + +p ≤ √ + √ +√ √ = 2.
a+b b+c (a + b)(b + c) 2 ab 2 bc ab + bc

Dấu bằng xảy ra khi a = b = c.


50 TUYỂN TẬP ĐỀ THI VÀO LỚP 10 CHUYÊN TOÁN

2.3. ĐỀ THI TUYỂN SINH VÀO LỚP 10 CHUYÊN TOÁN HÀ


NỘI, NĂM 2018

Câu 1

1. Giải phương trình: x2 + 3x + 8 = (x + 5) x2 + x + 2.
(
y2 − 2xy = 8x2 − 6x + 1
2. Giải hệ phương trình 2
y = x3 + 8x2 − x + 1

1
Lời giải.

A
1. Phương trình đã cho có thể viết lại
p
(x2 + x + 2) − (x + 5) x2 + x + 2 + 2(x + 3) = 0

hay tương dương với á n


To
p  p 
x2 + x + 2 − 2 x2 + x + 2 − (x + 3) = 0.

Tới đây, xét hai trường hợp


cb


Trường hợp 1. x2 + x + 2 = 2, tìm được x = 1 hoặc x = −2.
√ 7
Trường hợp 2. x2 + x + 2 = x + 3, tìm được x = − .
lạ

2. Cộng hai vế của phương trình đầu với x2 , ta được (y − x)2 = (3x − 1)2 . Tới đây, xét hai
u

trường hợp

Trường hợp 1. y − x = 3x − 1 hay y = 4x − 1. Thay vào phương trình thứ hai, tìm được
các nghiệm là
(0; −1), (1; 3), (7; 27).

Trường hợp 2. x − y = 3x − 1 hay y = 1 − 2x. Thay vào phương trình thứ hai, tìm được
các nghiệm là
(0; 1), (−1; 3), (−3; 7).

Vậy, hệ đã cho có 6 nghiệm (đã liệt kê ở trên).


CÂU LẠC BỘ TOÁN A1, HOTLINE: 034 761 1986 - 035 290 3286 51

Câu 2
1. Cho p, q là hai số nguyên tố lớn hơn 5. Chứng minh rằng p4 + 2019q4 chia hết cho
20.
√ √
2. Cho các số nguyên dương a, b, c, d thỏa mãn a < b ≤ c < d, ad = bc và d − a ≤
1.

a. Chứng minh rằng a + d > b + c.


b. Chứng minh rằng a là một số chính phương.

Lời giải.

1
1. Ta có p4 + 2019q4 ≡ p4 − q4 (mod 20). Ta có p > 5 nên p4 không chia hết cho 5, dẫn tới

A
p4 chia 5 dư 1, suy ra p4 − 1 chia hết cho 5. Mặt khác, p4 − 1 = (p2 − 1)(p2 + 1) chia hết
cho 4. Từ đó, ta có p4 − 1 chia hết cho 20. Chứng minh tương tự q4 − 1 chia hết cho 20.

n
Từ đó, suy ra điều phải chứng minh.

á
To
2. a. Từ giả thiết suy ra d − a > b − c ≥ 0, kéo theo (d − a)2 > (b − c)2 . Khai triển

d 2 − 2da + a2 > b2 − 2bc + c2 .


Sử dụng giả thiết da = bc, nên từ bất đẳng thức trên, cộng vế trái với 4ad, cộng vế
cb

phải với 4bc, ta được


(d + a)2 > (b + c)2 ,
lạ

kéo theo d + a > b + c.


√ √ √
b. Mặt khác ( b − c)2 ≥ 0, suy ra b + c ≥ 2 bc. Từ câu a), ta suy ra
u

√ √
d + a ≥ b + c + 1 ≥ 2 bc + 1 = 2 da + 1.
√ √ √ √
Suy ra ( √d − a)2 ≥ 1. Theo giả thiết thì ta phải có d − a = 1. Suy ra d =
a + 1 + 2 a. Do vế phải là số nguyên dương nên suy ra a là số chính phương.
Nhận xét. Từ chứng minh trên, ta cũng suy ra được d là số chính phương. Ngoài
a c x
cách giải trên, ta còn có thể giải theo hướng = = với (x, y) = 1. Từ đó, suy ra
b d y
a = mx, b = my, c = nx, d = ny. Theo đề, ta có x < y, m < n nên x + 1 ≤ y, m + 1 ≥ n.
Từ đó
√ √
d ≥ (x + 1)(m + 1) ≥ ( xm + 1)2 = ( a + 1)2 ,
√ √
kéo theo d ≥ a + 1. Từ đó, ta cũng có điều phải chứng minh.
52 TUYỂN TẬP ĐỀ THI VÀO LỚP 10 CHUYÊN TOÁN

Câu 3
1. Với x, y, z là các số thực dương thỏa mãn xyz = 1. Chứng minh rằng
1 1 1
+ + = 1.
xy + x + 1 yz + y + 1 zx + z + 1

1 1 1
2. Với x, y, z là các số thự dương thay đổi và thỏa mãn + + = 3, tìm giá trị lớn
x y z
nhất của biểu thức
1 1 1
P= p +p +√ .
2x2 + y2 + 3 2y2 + z2 + 3 2z2 + x2 + 3

1
Lời giải.

A
xyz 1 y
1. Từ giả thiết, ta có V T = + + = 1.
xy + x + xyz yz + y + 1 xyz + yz + 1

á n
a b c
Nhận xét. Ngoài cách giải trên, ta còn có thể đặt x = , y = , z = .
b c a
To
2. Theo bất đẳng thức Cauchy-Schwarz, ta có (2x2 + y2 + 3)(2 + 1 + 3) ≥ (2x + y + 3)2 , suy
ra


6
VT ≤ ∑ .
cb

2x + y + 3
(2 + 1 + 3)2 4 1 9
Áp dụng bất đẳng thức ≤ + + , từ trên, suy ra
2x + y + z 2x y 3
lạ

√   √
6 3 3 3 6
VT ≤ + + +9 = .
36 x y z 2
u

Dấu bằng xảy ra khi và chỉ khi x = y = z = 1.


Câu 4
Cho tứ giác ABCD (không có hai cạnh nào song song) nội tiếp đường tròn (O). Các tia
BA và CD cắt nhau tại điểm F. Gọi E là giao điểm của hai đường chéo AC và BD. Vẽ
hình bình hành AEDK.

1. Chứng minh rằng tam giác FKD đồng dạng với tam giác FEB.

2. Gọi M, N tương ứng là trung điểm của các cạnh AD, BC. Chứng minh rằng đường
thẳng MN đi qua trung điểm của đoạn thẳng EF.

3. Chứng minh rằng đường thẳng EF tiếp xúc với đường tròn ngoại tiếp của tam giác
EMN.
CÂU LẠC BỘ TOÁN A1, HOTLINE: 034 761 1986 - 035 290 3286 53

E
P
M O N
F K L

1
C

A
n
Lời giải.
1. Ta có phép biến đổi góc như sau:
á
To
∠FDK = ∠FDA − ∠KDA = ∠FBC − ∠EAD = ∠FBC − ∠EBC = ∠FBE

FD AD EA AD
Do 4FDA ∼ 4FBC nên = . Mặt khác, cũng do 4EAD ∼ 4EBC nên = .

FB BC EB BC
FD EA DK
cb

Từ đó, kết hợp với giả thiết AEDK là hình bình hành, suy ra = = . Vì vậy
FB EB BE
nên 4FKD ∼ 4FEB (cgc).

2. Do 4FKD ∼ 4FEB nên ∠DFK = ∠BFE.


lạ

Mặt khác, dựng hình bình hành EBLC, chứng minh tương tự như câu a ta sẽ thu được
4FLC ∼ 4FEA. Từ đó ta cũng thu được ∠CFL = ∠AFE.
u

Như vậy, ∠DFK = ∠CFL = ∠DFL. Vì vậy, F, K, L thẳng hàng. Chú ý rằng M, N cũng

là trung điểm của EK, EL. Từ đó, M, N và trung điểm P của EF thẳng hàng trên đường
trung bình của 4EKL.

FK FD FE FA
3. Do 4FKD ∼ 4FEB nên = . Mặt khác, do 4FLC ∼ 4FEA nên = .
FE FB FL FC
FD FA
Hơn nữa, vì tứ giác ABCD nội tiếp nên FA · FB = FC · FD nên = .
FB FC
FK FE
Kết hợp những điều trên ta thu được = . Suy ra FE 2 = FK · FL. Từ đó,
FE FL
1 1
PE 2 = · FE 2 = · FK · FL = PM · PN
4 4

Từ đó suy ra EF tiếp xúc với đường tròn (EMN).


54 TUYỂN TẬP ĐỀ THI VÀO LỚP 10 CHUYÊN TOÁN

Câu 5
Cho tập hợp S = {x ∈ Z | 1 ≤ x ≤ 50}. Xét A là một tập con bất kỳ của tập hợp S và có
tính chất: Không có ba phần tử nào của tập hợp A là số đo độ dài ba cạnh của một tam
giác vuông.

1. Tìm một tập hợp A có đúng 40 phần tử và thỏa mãn điều kiện đề bài.

2. Có hay không có một tập hợp A có đúng 41 phần tử và thỏa mãn điều kiện của đề
bài? Hãy giải thích câu trả lời.

Lời giải. Nếu a, b, c lập thành độ dài ba cạnh của một tam giác vuông thì ta gọi bộ đó là bộ
Pythagore. Trước hết, ta có thể liệt kê được đúng 20 bộ Pythagore thỏa mãn yêu cầu bài toán.

1
1. Bỏ đi tất cả các số là bội của 5 trong tập A thì ta được tập gồm 40 số thỏa mãn đề bài.

A
Chú ý là nếu bộ (a, b, c) là bộ Pythagore thì abc chia hết cho 5, nghĩa là trong ba số a, b, c
phải có một số là bội của 5, nên nếu bỏ đi tất cả các bội của 5 thì sẽ thu được tập hợp thỏa
mãn.

á n
2. Từ tập hợp A tìm được ở câu a) ta bỏ khỏi A các số 8, 9, 24, 36 và thêm vào 10, 15, 25, 40, 45
ta sẽ thu được một tập hợp gồm 41 phần tử thỏa mãn. Vì tập này không chứa các phần tử
To
8, 9, 24, 36 nên ta không thể tìm được một tam giác vuông với cạnh là 10, 15, 25, 40, 45.
Điều này có nghĩa là A chứa đúng 41 phần tử thỏa mãn yêu cầu bài toán.

cb

Nhận xét. Bài toán này sẽ hay hơn nếu hỏi như sau

Tìm số nguyên dương k nhỏ nhất để mọi tập con k phần tử của S luôn chứa ít nhất
lạ

một bộ Pythagore.
u

Đáp số: kmin = 42.



CÂU LẠC BỘ TOÁN A1, HOTLINE: 034 761 1986 - 035 290 3286 55

2.4. ĐỀ THI TUYỂN SINH VÀO LỚP 10 TRƯỜNG THPT CHUYÊN


KHOA HỌC TỰ NHIÊN NĂM 2019, VÒNG 1

Câu 1
1. Giải phương trình
26x + 5 √ p
√ + 2 26x + 5 = 3 x2 + 30.
x2 + 30

2. Giải hệ phương trình


(
x2 + y2 = 2

1
(x + 2y)(2 + 3y2 + 4xy) = 27

A
á n
Lời giải. .
To
1. Điều kiện xác định: 26x + 5 ≥ 0

Phương trình đã cho tương đương với


√  √ 
cb

p p
2
26x + 5 + 3 x + 30 2
26x + 5 − x + 30 = 0
√ p
⇐⇒ 26x + 5 = x2 + 30 ⇐⇒ x2 − 26x + 25 = 0
lạ

Từ đây ta tìm được x ∈ {1, 25}.


u

2. Ta có

( (
x2 + y2 = 2 x2 + y2 = 2
⇐⇒
(x + 2y)(2 + 3y2 + 4xy) = 27 (x + 2y)(x2 + 4y2 + 4xy) = 27
( (
x2 + y2 = 2 x2 + y2 = 2
⇐⇒ ⇐⇒
(x + 2y)3 = 27 x + 2y = 3

Từ đây ta tìm được


  
1 7
(x, y) ∈ (1, 1); ,
5 5
56 TUYỂN TẬP ĐỀ THI VÀO LỚP 10 CHUYÊN TOÁN

Câu 2
1. Tìm tất cả các cặp (x, y) nguyên thỏa mãn

(x2 − x + 1)(y2 + xy) = 3x − 1.

2. Với x, y là các số thực thỏa mãn 1 ≤ y ≤ 2, xy + 2 ≥ 2y, tìm giá trị nhỏ nhất của biểu
thức
x2 + 4
M= 2 .
y +1

Lời giải.

1
1 2 3
 

A
1. Ta có x2 − x + 1 | 3x − 1, chú ý rằng x2 − x + 1 = x− + > 0 với mọi x, suy ra
2 4

n
x2 − x + 1 ≤ |3x − 1| (2.1)

Ta xét hai trường hợp á


To
Trường hợp 1. 3x − 1 > 0
√ √
Từ (2.6) suy ra x2 − 4x + 2 ≤ 0, do đó 2 − 2 < x < 2 + 2, từ đó x ∈ {1, 2, 3}.

Trường hợp 2. 3x − 1 ≤ 0
cb

Từ (2.6) suy ra x2 + 2x ≤ 0, do đó −2 ≤ x ≤ 0 hay x ∈ {−2, −1, 0}


lạ

Bằng cách thử trực tiếp, ta tìm được các nghiệm là


u

(x, y) ∈ {(−2, 1); (1, 1); (1, −2)}


2y − 2
2. Từ xy + 2 ≥ 2y =⇒ x ≥ > 0. Do đó
y

2y − 2 2
 
+4
y 8y2 − 8y + 4
M≥ =
y2 + 1 y4 + y2

Mà 8y2 − 8y + 4 ≥ y4 + y2 ⇐⇒ (2 − y)(y3 + 2y2 − 3y + 2) ≥ 0 (đúng do 1 ≤ y ≤ 2). Vì


vậy nên M ≥ 1.

Vậy minM = 1, dấu bằng xảy ra ⇐⇒ x = 1, y = 2


CÂU LẠC BỘ TOÁN A1, HOTLINE: 034 761 1986 - 035 290 3286 57

Câu 3
Cho hình vuông ABCD, đường tròn (O) nội tiếp hình vuông ABCD tiếp xúc với các cạnh
AB, AD lần lượt tại các điểm E, F. Gọi giao điểm của CE và BF là G.

1. Chứng minh rằng năm điểm A, F, O, G, E cùng nằm trên một đường tròn.

2. Gọi giao điểm của FB và đường tròn (O) là M (M 6= F). Chứng minh rằng M là
trung điểm của đoạn thẳng BG.

3. Chứng minh rằng trực tâm của tam giác GAF nằm trên đường tròn (O).

1
A E B

A
H G M

á n
To
F O

cb

D C
lạ

Lời giải. 1. Ta có E, F là trung điểm của AB, AD. Từ đó 4AFB = 4BEC(cgc), suy ra
u

∠EBG = ∠ABF = ∠BCE = 90◦ − ∠BEG. Vậy ∠BGE = 90◦ hay BF ⊥ CE. Từ đó, do
∠EAF = ∠EGF = 90◦ nên tứ giác AEGF nội tiếp. Hơn nữa, ∠OEA = ∠OFA = 90◦ nên

tứ giác AEOF nội tiếp. Từ đó suy ra năm điểm A, F, O, G, E cùng nằm trên một đường
tròn.

2. Do EB là tiếp tuyến của đường tròn (O) nên ∠BEM = ∠EFM = ∠EFG. Mặt khác, ta có
tứ giác AEGF nội tiếp nên ∠EFG = ∠EAG. Như vậy, ∠BEM = ∠EAG suy ra EM k AG.
Kết hợp với E là trung điểm của AB suy ra M là trung điểm của BG (tính chất đường trung
bình).

3. Gọi H là trực tâm của tam giác GAF. Ta cần chứng minh rằng H ∈ (O). Thật vậy, ta có
AE k GH(⊥ AD) và AH k GE(⊥ BF) nên AEGH là hình bình hành. Do đó, GH = AE =
BE, từ đó BEHG là hình bình hành, suy ra HE k BF. Mặt khác, ta có ∠AGF = ∠AOF =
45◦ , từ đó ∠EMF = ∠AGF = 45◦ và ∠HFM = 90◦ − ∠AGF = 45◦ =⇒ ∠EMF =
∠HFM. Suy ra EHFM là hình thang cân, do đó cũng là một tứ giác nội tiếp. Vì vậy,
H ∈ (EFM) ≡ (O).
58 TUYỂN TẬP ĐỀ THI VÀO LỚP 10 CHUYÊN TOÁN

Câu 4
Cho x, y, z là các số thực dương thỏa mãn xy + yz + xz = 1. Chứng minh rằng:
!3
1 1 1 2 x y z
2
+ 2
+ 2
≥ √ +p +√ .
1+x 1+y 1+z 3 1 + x2 1 + y2 1 + z2

Lời giải (Cách 1). Ta có:


√ √ √
2(x + y + z)(xy + yz + xz) (x y + z + y x + z + z x + y)3
VT = ; VP = √ √ √
(x + y)(y + z)(z + x) (x + y)(y + z)(z + x) x + y y + z z + x

1
Ta cần chứng minh:

A
√ √ √ p
(x y + z + y x + z + z x + y)3 ≤ 3(x + y + x) (x + y)(y + z)(z + x) (2.2)

Thật vậy, ta có:

n
√ √ √ √ √ √ √ √√
á
(x y + z + y x + z + z x + y)2 = ( x xy + xz + y yx + yz + z zx + zy)2
To
≤ 2(x + y + z)(xy + yz + zx) = 2(x + y + z)

Từ đó suy ra

√ √ √ 3
p 3 √ √
(x y + z + y x + z + z x + y) ≤ 2(x + y + z) = 2 2(x + y + z) x + y + z
cb

Mặt khác,
r
lạ

p 8
3(x + y + x) (x + y)(y + z)(z + x) ≥ 3(x + y + z) · (x + y + z)(xy + yz + xz)
9
√ √
= 2 2(x + y + z) x + y + z
u

Từ đó suy ra bất đẳng thức (2.2) đúng. Vậy ta có điều phải chứng minh.

Lời giải (Cách 2). Sử dụng giả thiết 1 = xy + yz + zx ta có 1 + x2 = (x + y)(x + z). Áp dụng bất
đẳng thức AM-GM ta có
r  
x x x 1 x x
√ = · ≤ +
1 + x2 x+y x+z 2 x+y x+z

Tương tự, ta có  
y 1 y y
p ≤ +
1 + y2 2 y+x y+z
và  
z 1 z z
√ ≤ +
1 + z2 2 z+x z+y
CÂU LẠC BỘ TOÁN A1, HOTLINE: 034 761 1986 - 035 290 3286 59

Cộng các bất đẳng thức trên suy ra


x 3
∑ √1 + x2 ≤ 2
Từ đó, !2
x
VT ≤ ∑ p(x + y)(x + z) (2.3)

Sử dụng bất đẳng thức Cauchy-Schwarz, ta có


!2 2

 r  
x x x
∑ p(x + y)(x + z) = ∑ x ≤ (x + y + z) ∑
(x + y)(x + z) (x + y)(x + z)

1
(2.4)

A
Quy đồng mẫu số biểu thức trong ngoặc và sử dụng giả thiết thì
 
x
(x + y + z) ∑ = VP (2.5)

n
(x + y)(x + z)

Từ (2.3), (2.4), (2.5) ta có điều phải chứng minh. á


To

cb
lạ
u

60 TUYỂN TẬP ĐỀ THI VÀO LỚP 10 CHUYÊN TOÁN

2.5. ĐỀ THI TUYỂN SINH VÀO LỚP 10 TRƯỜNG THPT CHUYÊN


KHOA HỌC TỰ NHIÊN NĂM 2019, VÒNG 2

Câu 1
1. Giải hệ phương trình (
3x2 + y2 + 4xy = 8
(x + y)(x2 + xy + 2) = 8

2. Giải phương trình √ √


27 + x2 + x 27 + 2x
p = √
2 + 5 − (x + x) 2 + 5 − 2x
2

1
A
Lời giải. 1. Phương trình thứ nhất có thể viết lại (3x + y)(x + y) = 8. Thay vào phương trình

n
thứ hai, ta được
á
(x + y)(x2 + xy + 2 − 3x − y) = 0.
To
hay tương đương
(x + y)(x − 1)(x + y − 2) = 0.

Từ đây, ta có ba khả năng. Giải, ta thu được các nghiệm


cb

(x, y) ∈ {(1; 1), (1; −5)}.


lạ

27 5
2. Điều kiên xác định: x2 + x ≤ 5 và − ≤x≤ .
2 2
u


27 + t

Xét biểu thức f (t) = √ với −27 ≤ t ≤ 5 thì dễ thấy khi t tăng thì tử số tăng, còn
2+ 5−t
mẫu số giảm, do đó phân số sẽ tăng. Sử dụng điều này, ta thấy

(a) Nếu x2 + x > 2x thì vế trái lớn hơn vế phải.


(b) Nếu x2 + x < 2x thì vế trái nhỏ hơn vế phải.

Do đó, ta phải có x2 + x = 2x. Giải ta tìm được x = 0 và x = 1. Các nghiệm này đều thỏa
mãn.
Vậy, phương trình đã cho có nghiệm x ∈ {0; 1}.

Nhận xét. Ý thứ 2 có nhiều cách trình bày, nhưng chúng tôi chọn cách giải trên cho ngắn gọn.
CÂU LẠC BỘ TOÁN A1, HOTLINE: 034 761 1986 - 035 290 3286 61

Câu 2
1. Chứng minh rằng với mọi số nguyên dương n, ta luôn có
7 7 7
(27n + 5)7 + 10 + (10n + 27)7 + 5 + (5n + 10)7 + 27

chia hết cho 42.

2. Với x, y là các số thực dương thỏa mãn điều kiện 4x2 + 4y2 + 17xy + 5x + 5y ≥ 1,
tìm giá trị nhỏ nhất của biểu thức

P = 17x2 + 17y2 + 16xy

1
A
Lời giải.

n
1. Ta có bổ đề sau

á
To
Bổ đề. Ta có a7 ≡ a (mod 42) với mọi số nguyên a.

Bổ đề này quen thuộc, ta không chứng minh.


cb

Áp dụng bổ đề thì ta có ngay A ≡ 27n + 5 + 10 + 10n + 27 + 5 + 5n + 10 + 27 (mod 42)


hay A ≡ 42n + 84 (mod 42). Do đó, A chia hết cho 42.
lạ

2. Đặt S = x + y, T = xy thì áp dụng bất đẳng thức (x + y)2 ≥ 4xy ta có S2 ≥ 4T. Từ giả thiết,
suy ra 4S2 + 5S + 9T ≥ 1, kết hợp với S2 ≥ 4T, ta suy ra
u

9
4S2 + 5S + S2 ≥ 1.
4

2 √
Chú ý là S > 0 nên từ bất đẳng thức trên, ta suy được S ≥ ( 2 − 1). Do đó,
5

18 2 25 2 √
P = 17S2 − 18T ≥ 17S2 − S = S ≥ 6 − 4 2.
4 2

2 √ S2 2−1
Dấu bằng đạt được khi và chỉ khi S = ( 2 − 1), T = , giải được x = y = .
5 4 5
62 TUYỂN TẬP ĐỀ THI VÀO LỚP 10 CHUYÊN TOÁN

Câu 3
Cho tam giác ABC cân tại A, có đường tròn nội tiếp (I). Các điểm E, F theo thứ tự thuộc
các cạnh CA, AB (E khác C và A; F khác B và A) sao cho EF tiếp xúc với đường tròn (I)
tại điểm P. Gọi K, L lần lượt là hình chiếu vuông góc của E, F trên BC. Giả sử FK cắt EL
tại điểm J. Gọi H là hình chiếu vuông góc của J trên BC.

1. Chứng minh rằng HJ là phân giác của ∠EHF.

2. Ký hiệu S1 và S2 lần lượt là diện tích của các tứ giác BFJL và CEJK. Chứng minh
rằng
S1 BF 2
=
S2 CE 2

1
3. Gọi D là trung điểm của cạnh BC. Chứng minh rằng ba điểm P, J, D thẳng hàng.

A
n
A

á
To
Y E

cb

N M
P
F X
lạ

R Q
u

J I

B L H D K C

Lời giải. 1. Áp dụng định lý Thales ta có

FL FJ LH
= =
EK JK KH
Từ suy ra 4FLH ∼ 4EKH(cgc), vì vậy nên ta được ∠FHJ = ∠LFH = ∠KEH = ∠EHJ
và do đó HJ là phân giác của ∠EHF.
CÂU LẠC BỘ TOÁN A1, HOTLINE: 034 761 1986 - 035 290 3286 63

2. Ta có 4FLB ∼ 4EKC(gg) vì vậy nên

SFLB BF 2 FL2
= =
SEKC CE 2 EK 2
Hơn nữa ta cũng có 4FJL ∼ 4KJE(gg) nên

SFJL FL2 BF 2
= =
SKJE EK 2 CE 2
Từ đó suy ra
BF 2 SFLB SFJL SFLB + SFJL S1
2
= = = =
CE SEKC SKJE SEKC + SKJE S2

1
3. Ta có D là tiếp điểm của đường tròn (I) với BC và A, I, D thẳng hàng. Gọi Q, R là tiếp

A
điểm của đường tròn (I) với CA, AB. X,Y lần lượt là giao điểm của PQ, PR với AD.
1
Ta có IE, IF tương ứng là phân giác của các góc ∠PIQ, ∠PIR nên ∠EIF = ∠QIR =
2

n
∠AIQ = ∠AIR, từ đó thu được ∠AIE = ∠FIR và ∠AIF = ∠EIQ. Do đó, ∠XPE =
∠QPE = ∠EIQ = ∠AIF suy ra tứ giác XPFI nội tiếp và ∠FXI = ∠FPI = 90◦ hay
á
FX ⊥ AD. Chứng minh tương tự ta cũng được tứ giác Y PIE nội tiếp và EY ⊥ AD. Từ đó,
To
các tứ giác FLDX, EKDY là các hình chữ nhật, do đó DX = FL và DY = EK.

Mặt khác, ta có ∠IPX = ∠IEP = ∠IY P suy ra 4IXP ∼ 4IPY =⇒ IP2 = IX · IY . Dựng

đường kính DM của đường tròn (I). Tiếp tuyến tại M của (I) cắt EF tại N. Từ đó, ta được
ID2 = IM 2 = IX · IM = IX · IY và do đó
cb

IX ID ID + IX DX FL FJ
= = = = =
ID IY ID + IY DY EK JK
lạ

IX IM IM − IX MX FN
= = = =
IM IY IY − IM MY NE
u

FJ FN
Từ đó suy ra = =⇒ JN k EK. Theo tiên đề Euclid suy ra N, J, H thẳng hàng. Từ

JK NE
JN FJ LJ JH
đó suy ra MNHD là hình chữ nhật. Hơn nữa, ta có = = = suy ra J là
EK FK LE EK
trung điểm của NH. Kết hợp với I là trung điểm MD, ta suy ra các tứ giác MNJI và IJHD
là hình chữ nhật. Từ đó ta cũng có JN = IM = ID và JN k ID nên NJDI là hình bình hành
và ta được JD k IN. Mặt khác, chú ý rằng phân giác góc ngoài ở đỉnh của tam giác cân
song song với đáy, ta suy ra DP k IN (dễ thấy IN là phân giác của ∠MIP). Từ đó, áp dụng
tiên đề Euclid suy ra P, J, D thẳng hàng.

Câu 4
Cho M là tập tất cả 4039 số nguyên liên tiếp từ −2019 đến 2019. Chứng minh rằng trong
2021 số đôi một phân biệt được chọn bất kỳ từ tập M luôn tồn tại 3 số đôi một phân biệt
có tổng bằng 0.
64 TUYỂN TẬP ĐỀ THI VÀO LỚP 10 CHUYÊN TOÁN

Lời giải. Ta giải bài toán tổng quát như sau: Trong 2n + 1, n ∈ Z+ phần tử bất kỳ của tập
M = {−(2n − 1), −(2n − 2), ..., 0, ..., 2n − 2, 2n − 1}
luôn có ba phần tử có tổng bằng 0.
Ta chứng minh bằng phương pháp quy nạp. Thật vậy, dễ dàng kiểm tra với n = 1 thì M =
{−1; 0; 1} thì hiển nhiên khẳng định được chứng minh. Giả sử khẳng định đúng với n. Với n + 1
xét tập
M = {−(2n + 1), −2n, ..., 0, ..., 2n, 2n + 1}.
Đặt N = {−(2n + 1), −2n, 2n, 2n + 1}, xét các khả năng sau
Trường hợp 1. Nếu trong 2n + 3 phần tử có tối đa hai phần tử thuộc N thì sẽ có ít nhất 2n + 1
phần tử thuộc tập {−(2n − 1), −(2n − 2), ..., 0, ..., 2n − 2, 2n − 1}. Sử dụng giả thiết quy nạp, ta
có ngay điều phải chứng minh.

1
Trường hợp 2. Nếu 2n + 3 phần tử đều chứa cả bố phần tử của N, thì bỏ đi hai phần tử −2n −

A
1, 2n + 1, xét 2n + 1 phần tử còn lại, thuộc vào trong 2n nhóm.

(1, 2n), (2, 2n − 1), ..., (n, n + 1), (0, −2n − 1), (−1, −2n), ..., (−n, −n − 1).

n
Theo nguyên tắc Dirichlet, tồn tại hai phần tử thuộc cùng một nhóm, tổng của chúng là −2n − 1
á
hoặc 2n + 1. Trong tình huống nào ta cũng có điều phải chứng minh.
To
Trường hợp 3. Nếu 2n + 3 phần tử chứa đúng ba phần tử của N. Xét các khả năng sau
1. Nếu 3 phần tử đó là −2n − 1, 2n + 1, 2n thì nếu trong 2n phần tử còn lại chứa 0 hoặc 1 thì
xong, còn nếu không chứa 0 và 1 thì 2n phần tử còn lại sẽ thuộc 2n − 1 nhóm

(−1, −2n), (−2, −2n + 1), ..., (−n, −n − 1), (2, 2n − 1), (3, 2n − 2), ..., (n, n + 1)
cb

nên theo nguyên tắc Dirichlet, tồn tại hai phần tử thuộc cùng một nhóm, tổng hai phần tử
này là −(2n + 1) hoặc 2n + 1. Trong tình huống nào ta cũng có điều phải chứng minh.
lạ

2. Nếu 3 phần tử đó là −2n, 2n, 2n + 1 thì lập luận tương tự như trên bằng cách thay 0,1 bằng
0, −1. Ta bỏ đi phần tử n, thì 2n − 1 phần tử còn lại sẽ thuộc 2n − 2 nhóm
u

(1, 2n − 1), (2, 2n − 2), ..., (n − 1, n + 1), (−2, −2n + 1), ..., (−n, −n − 1),

nên theo nguyên lý Dirichlet, tồn tại hai phần tử thuộc cùng một nhóm, hai phần tử này
có tổng là 2n hoặc −2n − 1. Trong tình huống nào ta cũng có điều phải chứng minh.
Các trường hợp còn lại được xét tương tự bằng cách đổi dấu lại.

2.6. ĐỀ THI TUYỂN SINH VÀO LỚP 10 TRƯỜNG THPT CHUYÊN


SƯ PHẠM HÀ NỘI NĂM 2019, VÒNG 2
Câu 1
Cho hai số thực phân biệt a, b thỏa mãn a3 + b3 = a2 b2 (ab − 3). Tính giá trị của biểu thức
T = a + b − ab.
CÂU LẠC BỘ TOÁN A1, HOTLINE: 034 761 1986 - 035 290 3286 65

Lời giải. Giả thiết có thể viết lại thành

(a + b)3 − a3 b3 + 3a2 b2 − 3ab(a + b) = 0

hay tương đương


(a + b − ab).M = 0

a+b 2 3
 
2 2 2
với M = (a + b) + ab(a + b) + a b − 3ab = ab + + (a − b)2 > 0 do a, b phân biệt.
2 4
Từ đó suy ra T = a + b − ab = 0.

Câu 2

1
Cho các đa thức P(x) = m1 x2 + n1 x + k1 , Q(x) = m2 x2 + n2 x + k2 , R(x) = m3 x2 + n3 x + k3

A
với mi , ni , ki là các số thực và mi > 0, i = 1, 2, 3. Giả sử phương trình P(x) = 0 có hai
nghiệm phân biệt a1 , a2 , phương trình Q(x) = 0 có hai nghiệm phân biệt b1 , b2 , phương
trình R(x) = 0 có hai nghiệm phân biệt c1 , c2 thỏa mãn

á n
P(c1 ) + Q(c1 ) = P(c2 ) + Q(c2 )
P(b1 ) + R(b1 ) = P(b2 ) + R(b2 )
To
Q(a1 ) + R(a1 ) = Q(a2 ) + R(a2 )

Chứng minh rằng a1 + a2 = b1 + b2 = c1 + c2 .



cb

Lời giải. Ta có P(c1 ) + Q(c1 ) = P(c2 ) + Q(c2 ) tương đương với

(m1 + m2 )(c1 + c2 ) + (n1 + n2 ) = 0.


lạ

n1 + n2 n3
Suy ra c1 + c2 = − . Áp dụng định lý Viete cho R(x), ta có c1 + c2 = − . Suy ra
m1 + m2 m3
u

n3 n1 + n2 n1 + n2 + n3

= = .
m3 m1 + m2 m1 + m2 + m3
n3 n2 n1
Tương tự với các đẳng thức còn lại, suy ra = = . Áp dụng định lý Viete, ta có ngay
m3 m2 m1
a1 + a2 = b1 + b2 = c1 + c2 .

Câu 3
1. Tìm các số nguyên x, y thỏa mãn x2 y2 − 4x2 y + y3 + 4x2 − 3y2 + 1 = 0.

2. Cho ba số nguyên dương a, b, c thỏa mãn a3 + b3 + c3 chia hết cho 14. Chứng minh
rằng abc chia hết cho 14.

Lời giải.
66 TUYỂN TẬP ĐỀ THI VÀO LỚP 10 CHUYÊN TOÁN

1. Phương trình đã cho có thể viết lại thành

(y2 − 4y + 4)x2 = −(y3 − 3y2 + 1).

Dễ thấy y = 2 không thỏa mãn nên ta có y − 2 | y3 − 3y2 + 1 = y2 (y − 2) − (y2 − 4) − 3.


Suy ra y − 2 | 3. Từ đây, suy ra y ∈ {−1; 1; 3; 5}. Bằng cách thử trực tiếp, ta tìm được 2
cặp (x, y) thỏa mãn là (1; 1), (−1; 1).

2. Từ giả thiết suy ra trong ba số a, b, c phải có ít nhất một số chẵn, kéo theo abc chia hết
cho 2.
Mặt khác, dễ dàng kiểm tra x3 ≡ 0, −1, 1 (mod 7) với mọi số nguyên x nên từ điều kiện
7 | a3 + b3 + c3 ta suy ra trong ba số a, b, c phải có ít nhất một số chia hết cho 7, vì nếu
không có số nào chia hết cho 7 thì a3 + b3 + c3 ≡ −3, 3, −1, 1 (mod 7), vô lí. Từ đó suy

1
ra abc chia hết cho 7.

A
Từ các điều vừa chứng minh ở trên, với chú ý (2, 7) = 1 nên abc chia hết cho 14.

n
Câu 4

á
Cho tam giác ABC có ba góc nhọn nội tiếp đường tròn (O) và AB > AC. Gọi D, E lần lượt
To
là chân các đường cao của tam giác ABC hạ từ A, B. Gọi F là chân đường vuông góc hạ
từ B lên đường thẳng AO.

1. Chứng minh rằng B, D, E, F là bốn đỉnh của một hình thang cân.

2. Chứng minh rằng EF đi qua trung điểm của BC.


cb

3. Gọi P là giao điểm thứ hai của đường thẳng AO và đường tròn (O), M và N lần lượt
là trung điểm của EF và CP. Tính góc ∠BMN.
lạ
u

A

O E

M
D
B C
K
F
N

P
CÂU LẠC BỘ TOÁN A1, HOTLINE: 034 761 1986 - 035 290 3286 67

Lời giải.
1. Ta có ∠ADB = ∠AEB = ∠AFB = 90◦ nên ba điểm A, B, D, E, F cùng nằm trên đường
tròn đường kính AB. Như vậy, tứ giác BEDF nội tiếp. Chú ý rằng ta có kết quả quen thuộc
∠OAB = ∠DAC nên ∠FDB = ∠FAB = ∠DAC = ∠DBE suy ra DF k BE. Từ đó suy ra
BEDF là hình thang cân.

2. Gọi K là giao điểm của EF và BC. Do BEDF là hình thang cân nên KE = KB và từ đó
∠KEB = ∠KBE. Do đó, ∠KCE = 90◦ − ∠KBE = 90◦ − ∠KEB = ∠KEC suy ra tam giác
KEC cân tại K. Như vậy, KC = KE = KB và vì vậy K là trung điểm của BC.

3. Do các tứ giác ABFE và ABPC nội tiếp nên ∠BFE = 180◦ − ∠BAE = 180◦ − ∠BAC =
∠BPC và ∠BEF = ∠BAF = ∠BAP = ∠BCP. Suy ra 4BFE ∼ 4BPC(gg). Hai tam
BM BE
giác đồng dạng này có cặp trung tuyến tương ứng là BM và BN, suy ra = và

1
BN BC
∠MBN = ∠MBC + ∠NBC = ∠MBC + ∠MBE = ∠EBC. Từ đó 4BMN ∼ 4BEC, vì vậy

A
nên ∠BMN = ∠BEC = 90◦ .

n
Câu 5

á
Cho tập X thỏa mãn tính chất sau: tồn tại 2019 tập con A1 , A2 , · · · , A2019 của X sao cho
To
mỗi tập con A1 , A2 , · · · , A2019 có đúng ba phần tử và hai tập con Ai , A j đều có đúng một
phần tử chung với mọi 1 ≤ i < j ≤ 2019.

1. Chứng minh rằng tồn tại 4 tập hợp trong các tập A1 , A2 , · · · , A2019 sao cho giao của

4 tập hợp này có đúng một phần tử.


cb

2. Chứng minh rằng số phần tử của X phải lớn hơn hoặc bằng 4039.
lạ

Lời giải. 1. Xét tập A1 = {a, b, c}, do giao của hai tập có đúng một phần tử nên 2008 tập
con còn lại phải chứa đúng 1 trong 3 phần tử a, b, c. Do đó tồn tại ba tập con cùng chứa
một phần tử, giả sử là a và ba tập con đó cùng với A tạo ra bốn tập con thỏa mãn bài toán.
u

(Thực ra ta có thể chứng minh có ít nhất 674 tập có đúng một phần tử chung).

2. Gọi a là phần tử thuộc vào ít nhất 4 tập con. Ta chứng minh a thuộc vào tất cả các tập hợp.
Thật vậy, giả sử phản chứng tồn a ∈ A1 , A2 , A3 , A4 nhưng a ∈
/ A5 . Vì A5 đều có chung với
bốn tập A1 , A2 , A3 , A4 một phần tử nên A5 chứa ít nhất 4 phần tử, vô lý. Do vậy, a thuộc
vào tất cả các tập. Gọi Bi là tập sau khi bỏ khỏi Ai phần tử a. Thế thì các tập Bi đôi một
giao nhau bằng rỗng. Suy ra số phần tử trong hợp các tập B1 , B2 , · · · , B2019 là 2 × 2019.
Suy ra nếu gọi n là số phần tử của X thì n ≥ 2.2019 + 1 = 4039 vì có thêm phần tử a nữa.
68 TUYỂN TẬP ĐỀ THI VÀO LỚP 10 CHUYÊN TOÁN

2.7. ĐỀ THI TUYỂN SINH VÀO LỚP 10 CHUYÊN TOÁN HÀ


NỘI, NĂM 2019
Câu 1
√ √ √
1. Giải phương trình ( x + 5 − x)(1 + x2 + 5x) = 5.
(
x2 + 7 = 4y2 + 4y
2. Giải hệ phương trình
x2 + 3xy + 2y2 + x + y = 0.

Lời giải.
√ √

1
1. Điều kiện: x ≥ 0. Với điều kiện này thì x + x + 5 > 0, nên phương trình đã cho có thể

A
viết lại dưới dạng p √ √
1 + x(x + 5) = x + x + 5
hay tương đương

n
√ √
( x − 1)( x + 5 − 1) = 0.
á
Giải phương trình trên và so sánh điều kiên, ta tìm được đúng một nghiệm là x = 1.
To
2. Phương trình thứ hai có thể viết lại (x + y)(x + 2y + 1) = 0. Từ đó ta có hai trường hợp
x = −y hoặc x = −2y − 1. Thay từng trường hợp vào phương trình đầu, ta tìm được hai
nghiệm của hệ phương trình là

cb

 
7 7
(−1; 1), ;−
3 3
lạ

Câu 2
u

1. Cho biểu thức P = abc(a − 1)(b + 4)(c + 6) với a, b, c là các số nguyên thỏa mãn
a + b + c = 2019. Chứng minh rằng giá trị của biểu thức P chia hết cho 6.

2. Tìm tất cả các số tự nhiên n sao cho giá trị của biểu thức
√ √
q
Q = n+2+ n+ n+2

là số nguyên.

Lời giải.
1. a và a − 1 là hai số nguyên liên tiếp nên trong hai số này luôn có một số chẵn, nghĩa là P
luôn chia hết cho 2.
Nếu trong ba số a, b, c có một số chia hết cho 3 thì P chia hết cho 3. Nếu cả ba số a, b, c
đều không chia hết cho 3 thì vì a + b + c chia hết cho 3 nên a, b, c phải có cùng dư khi
chia cho 3. Nếu a, b, c cùng dư 1 khi chia cho 3 thì a − 1 chia hết cho 3, nếu a, b, c cùng
CÂU LẠC BỘ TOÁN A1, HOTLINE: 034 761 1986 - 035 290 3286 69

dư 2 khi chia cho 3 thì b + 4 chia hết cho 3. Trong mọi tình huống thì P luôn chia hết cho
3.
Mà (2, 3) = 1 nên P chia hết cho 2.3 = 6.

2. Vì Q nhận giá trị tự nhiên và dễ thấy Q > 0 nên ta viết


√ √
n + n + 2 = Q2 − 2Q n + 2 + n + 2,

kéo theo
√ Q2 + 2
n+2 = .
2Q + 1
(Q2 + 2)2
Suy ra n + 2 = . Do đó 2Q + 1 phải là ước của Q2 + 2. Từ đây, ta có 4(Q2 + 2) =

1
(2Q + 1)2
4Q2 − 1 + 9 chia hết cho 2Q + 1, nên 9 chia hết cho 2Q + 1. Tìm được Q = 1 hoặc Q = 4.

A
Nhưng nếu Q = 1 thì n = −1 không thỏa mãn, vậy Q = 4, tương ứng với n = 2.
Vậy, n = 2 là số tự nhiên duy nhất thỏa mãn bài toán.

Câu 3 á n
To
Cho biểu thức K = ab + 4ac − 4bc, với a, b, c là các số thực không âm thỏa mãn a + b +
2c = 1.

1
1. Chứng minh K ≥ − .
cb

2
2. Tìm giá trị lớn nhất của biểu thức K.
lạ

Lời giải.
1. Ta có 1 = a + b + 2c ≥ b + 2c nên suy ra 1 ≥ (b + 2c)2 ≥ 4 · b · 2c = 8bc. Suy ra −4bc ≥
u

1 1
− . Từ đây với chú ý a, b, c ≥ 0 nên suy ra ngay K ≥ − .

2 2
2. Ta có

K = a(b + 2c) + 2ac − 4bc = a(1 − a) + 2ac − 4bc ≤ a(1 − a) + 2ca.

Từ điều trên, áp dụng bất đẳng thức AM − GM, ta suy ra

(a + 1 − a)2 (2c + a)2 1 (2c + a + b)2 1


K≤ + ≤ + = .
4 4 4 4 2

1 1
Dấu bằng xảy ra khi và chỉ khi b = 0, 2c = a, a + b + 2c = 1, hay a = , b = 0, c = . Vậy
2 4
1
giá trị lớn nhất của K là .
2
70 TUYỂN TẬP ĐỀ THI VÀO LỚP 10 CHUYÊN TOÁN

Câu 4
Cho tam giác ABC có ba góc nhọn (AB < AC), nội tiếp đường tròn (O). Gọi I là tâm
đường tròn nội tiếp tam giác ABC. Tia AI cắt đoạn thẳng BC tại điểm J, cắt đường tròn
(O) tại điểm thứ hai M (M khác A).

1. Chứng minh rằng MI 2 = MJ · MA.

2. Kẻ đường kính MN của đường tròn (O). Đường thẳng AN cắt các tia phân giác
trong của góc ABC và góc ACB lần lượt tại các điểm P và Q. Chứng minh rằng N
là trung điểm của đoạn thẳng PQ.

3. Lấy điểm E bất kỳ thuộc cung nhỏ MC của đường tròn (O) (E khác M). Gọi F là
điểm đối xứng với điểm I qua điểm E. Gọi R là giao điểm của hai đường thẳng PC

1
và QB. Chứng minh rằng bốn điểm P, Q, R, F cùng nằm trên một đường tròn.

A
n
P
á
To
A N
X
Q

Y L
cb

I O
lạ

B J C

E
u

M

Lời giải. 1. Ta có ∠MBJ = ∠MBC = ∠MAC = ∠MAB suy ra 4MJB ∼ 4MBA. Từ đó ta


được MB2 = MJ · MA. Mặt khác, ta lại có ∠MIB = ∠IAB + ∠IBA = ∠MBJ + ∠IBC =
∠IBM, vì vậy tam giác MIB cân tại M và MI = MB. Do đó, MI 2 = MB2 = MJ · MA.

2. Ta có P, Q lần lượt là tâm đường tròn bàng tiếp ứng với các đỉnh B,C của tam giác ABC.
Từ đó ta có CP là phân giác ngoài của 4ABC, ∠PCQ = 90◦ và tứ giác AIBQ nội tiếp.
Như vậy, ∠CNP = ∠ABC = 2∠ABI = 2∠AQI = 2∠NQC. Suy ra NQ = NC, chú ý rằng
tam giác CPQ cân tại C, ta thu được N là trung điểm của PQ.
CÂU LẠC BỘ TOÁN A1, HOTLINE: 034 761 1986 - 035 290 3286 71

3. Ta có R là tâm đường tròn bàng tiếp ứng với đỉnh A của tam giác ABC. Từ đó tam giác IBR
vuông tại B và kết hợp với MB = MI (chứng minh ở ý 1) thì ta được M là trung điểm của
IR. Tương tự các trung điểm X,Y của các cung nhỏ CA, AB của đường tròn (O) tương ứng
là trung điểm của IP, IQ. Lấy điểm L đối xứng I qua O. Khi đó, OE, OM, OX, OY là các
đường trung bình của các tam giác ILF, ILR, ILP, ILQ. Từ đó suy ra LF = LP = LQ = LR
(= 2OE = 2OX = 2OY = 2OM). Vậy P, Q, R, F cùng thuộc đường tròn tâm L.

Câu 5
Mỗi điểm trong mặt phẳng được tô bởi một trong hai màu xanh hoặc đỏ.

1. Chứng minh trong mặt phẳng đó tồn tại hai điểm được tô bởi cùng một màu và có
khoảng cách bằng d.

1
2. Gọi tam giác có ba đỉnh được tô cùng một màu là tam giác đơn sắc. Chứng minh

A
trong mặt phẳng đó tồn tại hai tam giác đơn sắc là hai tam giác vuông đồng dạng
1
với nhau theo tỉ số k = .

n
2019

á
To
F E

cb

A B
lạ
u

C D

Lời giải. 1. Câu này quen thuộc, có thể tìm thấy ở bất kỳ tài liệu nào. Chúng tôi không trình
bày lời giải ở đây.

2. Theo ý trên thì với mọi số thực dương d luôn tồn tại hai điểm được tô cùng màu có khoảng
cách bằng d. Thế thì tồn tại hai điểm A, B có khoảng cách là 2019 được tô cùng màu, giả
sử là màu đỏ. Dựng đường tròn đường kính AB. Dựng lục giác đều ACDBEF. Trong các
điểm C, D, E, F nếu có một điểm được tô đỏ, giả sử là C thì tam giác ABC là tam giác đơn
sắc nửa đều có ba đỉnh được tô đỏ. Ngược lại, nếu cả 4 điểm C, D, E, F được tô xanh thì
tam giác CDE là tam giác đơn sắc nửa đều có ba đỉnh được tô xanh. Điều này có nghĩa là
ta chứng minh được tồn tại một tam giác đơn sắc nửa đều nội tiếp đường tròn đường kính
2019, giả sử là PQR.
72 TUYỂN TẬP ĐỀ THI VÀO LỚP 10 CHUYÊN TOÁN

Tương tự, cũng tồn tại hai điểm M, N được tô cùng màu và MN = 1. Chứng minh tương
tự như trên, ta cũng chỉ ra được tồn tại một tam giác đơn sắc nửa đều nội tiếp đường tròn
đường kính 1, giả sử là XY Z.
1
Rõ ràng XY Z và PQR là hai tam giác đơn sắc đồng dạng với tỉ số k = .
2019

1
A
á n
To

cb
lạ
u

CÂU LẠC BỘ TOÁN A1, HOTLINE: 034 761 1986 - 035 290 3286 73

2.8. ĐỀ THI TUYỂN SINH VÀO LỚP 10 TRƯỜNG THPT CHUYÊN


KHOA HỌC TỰ NHIÊN NĂM 2020, VÒNG 1
Câu 1
1. Giải hệ phương trình (
x2 + y2 + xy = 7
9x3 = xy2 + 70(x − y).

2. Giải phương trình


√ √ p
11 5 − x + 8 2x − 1 = 24 + 3 (5 − x)(2x − 1).

1
A
Lời giải.
1. Từ hệ phương trình ta suy ra

á n
9x3 = xy2 + 10(x2 + y2 + xy)(x − y) = xy2 + 10(x3 − y3 ).
To
Từ đó ta được x3 + xy2 − 10y3 = 0 ⇐⇒ (x − 2y)(x2 + 2xy + 5y2 ) = 0. Chú ý rằng x2 +
2xy + 5y2 = (x + y)2 + 4y2 ≥ 0. Dấu bằng xảy ra khi x = y = 0, tuy nhiên điều này mâu
thuẫn với phương trình thứ nhất của hệ phương trình đã cho. Như vậy, x2 + 2xy + 5y2 > 0

và do đó x = 2y. Thay x = 2y vào phương trình thứ nhất ta suy ra 7y2 = 7 hay y = ±1.
cb

Vậy hệ phương trình đã cho có hai cặp nghiệm (x, y) là (2, 1) và (−2, −1).

1 √ √
2. Điều kiện xác định: ≤ x ≤ 5. Đặt ẩn phụ a = 5 − x, b = 2x − 1 với a, b ≥ 0. Từ đó
2
lạ

ta có 2a2 + b2 = 9. Khi đó, ta có 24 + ab = 15 + 2a2 + b2 + 3ab = 15 + (2a + b)(a + b).


Hơn nữa, cũng chú ý rằng 11a + 8b = 3(2a + b) + 5(a + b). Như vậy phương trình ban
đầu sẽ tương đương với
u

3(2a + b) + 5(a + b) = 15 + (2a + b)(a + b)


⇐⇒ (2a + b − 5)(a + b − 3) = 0.

Từ đó suy ra 2a + b = 5 hoặc a + b = 3.

Trường hợp 1. 2a + b = 5. Từ 2a2 + b2 = 9 suy ra 2a2 + (5 − 2a)2 = 9 ⇐⇒ (a − 2)(3a −


4) = 0. Dẫn đến a = 2 hoặc a = 43 . Giải ra ta thu được hai nghiệm x = 1 và x = 29
9.

Trường hợp 2. a + b = 3. Từ 2a2 + b2 = 9 suy ra 2a2 + (3 − a)2 = 9 ⇐⇒ 3a(a − 2) = 0.


Dẫn đến a = 0 hoặc a = 2. Giải ra ta thu được hai nghiệm x = 5 và x = 29
9.
 
29
Kết luận. S = 1, , 5 .
9
74 TUYỂN TẬP ĐỀ THI VÀO LỚP 10 CHUYÊN TOÁN

Câu 2
1. Tìm x, y nguyên dương thỏa mãn

x2 y2 − 16xy + 99 = 9x2 + 36y2 + 13x + 26y.

2. Với a, b là các số thực dương thỏa mãn

2 ≤ 2a + 3b ≤ 5, 8a + 12b ≤ 2a2 + 3b2 + 5ab + 10.

Chứng minh rằng


3a2 + 8b2 + 10ab ≤ 21.

1
Lời giải.

A
1. Phương trình tương đương với

(xy + 10)2 = (3x + 6y)2 + 13(x + 2y) + 1. (2.6)

n
Do x, y là các số nguyên dương nên ta có x + 2y ≥ 3. Vì thế nên ta có
á
To
(3x + 6y)2 + 13(x + 2y) + 1 ≥ (3x + 6y + 2)2


(3x + 6y)2 + 13(x + 2y) + 1 < (3x + 6y + 3)2 .

Do vế phải của (2.6) là số chính phương nên 9(x + 2y)2 + 13(x + 2y) + 1 = (3x + 6y + 2)2 .
cb

Từ đó suy ra x + 2y = 3. Vì x, y là các số nguyên dương nên x = y = 1. Như vậy phương


trình có một cặp nghiệm là (x, y) = (1, 1).
2. Cách 1. Đặt 2a + 3b = x và a + b = y. Từ giả thiết ta thu được 2 ≤ x ≤ 5 và 4x ≤ xy + 10.
lạ

4x − 10
Vì vậy nên y ≥ . Trong khi đó ta có
x
u

P = 3a2 + 8b2 + 10ab = (a + 2b)(3a + 4b) = (x − y)(x + y) = x2 − y2 .


Ta cần chứng minh rằng x2 − y2 ≤ 21.


5
Trường hợp 1. Nếu x < 2 thì x2 < 21 và vì thế nên x2 − y2 < 21.
 2
5 4x − 10
Trường hợp 2. Nếu x > thì 4x − 10 ≥ 0. Do đó, ta suy ra được ≥
2 y2 =⇒
x
4x − 10 2
 
2 2 2
x −y ≤ x − . Như vậy ta chỉ cần chứng minh bất đẳng thức sau
x

4x − 10 2
 
2
x − ≤ 21 ⇐⇒ x4 − 37x2 + 80x − 100 ≤ 0.
x

Bất đẳng thức này tương đương với

(x − 5)(x3 + 5x2 − 12x + 20) ≤ 0.


CÂU LẠC BỘ TOÁN A1, HOTLINE: 034 761 1986 - 035 290 3286 75

2
Bất đẳng thức này luôn đúng do x ≤ 5 và x3 + 5x2 − 12x + 20 = x3 + 5 x − 56 + 64
5 > 0.
Như vậy ta có điều phải chứng minh.

Cách 2. Từ giả thiết ta suy ra


8a + 12b ≤ (2a + 3b)(a + b) + 10 ≤ 5(a + b) + 10 =⇒ 3a + 7b ≤ 10.
Từ đó ta có
1
3a2 + 8b2 + 10ab = (3a + 4b)(a + 2b) = (9a + 12b)(7a + 14b)
21
1 9a + 12b + 7a + 14b 2
 
1
≤ = (16a + 26b)2
21 2 84
 2  2
1 34 4 1 34 4
= (2a + 3b) + (3a + 7b) ≤ · 5 + · 10 = 21.

1
84 5 5 84 5 5

A
Câu 3

n
Cho tam giác ABC có ∠BAC là góc nhỏ nhất trong ba góc của tam giác và nội tiếp đường
á
tròn (O). Điểm D thuộc cạnh BC sao cho AD là phân giác của ∠BAC. Lấy các điểm M, N
To
thuộc (O) sao cho các đường thẳng CM và BN cùng song song với đường thẳng AD.

1. Chứng minh rằng AM = AN.


2. Gọi giao điểm của đường thẳng MN với các đường thẳng AC, AB lần lượt là E, F.
Chứng minh rằng bốn điểm B,C, E, F cùng thuộc một đường tròn.
cb

3. Gọi P, Q theo thứ tự là trung điểm của các đoạn thẳng AM, AN. Chứng minh rằng
EQ, FP, AD đồng quy.
lạ

Lời giải.
u

1. Do AD là phân giác ∠BAC và BN,CM cùng song song với AD nên ta có ∠ABN = ∠BAD =

∠CAD = ∠ACM. Từ đó suy ra số đo các cung nhỏ AM và AN của đường tròn (O) là bằng
nhau, vì vậy AM = AN.
2. Ta có ∠AEF = ∠EAM + ∠AMN = ∠MBC + ∠ANM = ∠MBC + ∠ABM = ∠ABC. Do
đó, tứ giác BCEF là một tứ giác nội tiếp.
3. Đặt AD cắt đường tròn (O) lần thứ hai tại điểm X. Khi đó ta có X là trung điểm của cung
BC không chứa A của (O). Gọi J là trung điểm của AX. Ta có JQ k XN (tính chất đường
trung bình). Trong khi đó, vì ∠NXA = ∠ABN = ∠BAD = ∠CAD, nên XN k AC. Như vậy
ta thu được JQ k AC. Chứng minh tương tự ta cũng được XM k AB và JP k AB. Mặt khác,
từ XN k AE và XM k AF, ta cũng suy ra được 4XMN ∼ 4AFE (gg) và XN XM
AE = AF .
GJ JQ XN XM JP
Đặt EQ ∩ AD ≡ G. Từ AE k JQ ta suy ra GA = AE = 2AE = 2AF = AF . Từ đó ta thu được
4GJP ∼ 4GAF (cgc). Vì vậy, ∠AGF = ∠JGP = 180◦ − ∠AGP hay ∠FGP = 180◦ . Như
vậy, F, G, P thẳng hàng và do đó ba đường thẳng EQ, FP, AD đồng quy tại G.
76 TUYỂN TẬP ĐỀ THI VÀO LỚP 10 CHUYÊN TOÁN

A
M
P
G
Q E
N F

O
J

1
A
D
B C

á n
To
X

Câu 4

Cho a, b, c là các số thực dương thỏa mãn a + b + c = 3. Chứng minh rằng


cb

a(a + bc)2 b(b + ca)2 c(c + ab)2


+ + ≥ 4.
b(ab + 2c2 ) c(bc + 2a2 ) a(ca + 2b2 )
lạ

Lời giải. Đặt


a(a + bc)2 b(b + ca)2 c(c + ab)2
u

P= + + .
b(ab + 2c2 ) c(bc + 2a2 ) a(ca + 2b2 )

Áp dụng bất đẳng thức Cauchy - Schwarz dạng phân thức ta có


(a2 + abc)2 (b2 + abc)2 (c2 + abc)2
P= + +
ab(ab + 2c2 ) bc(bc + 2a2 ) ca(ca + 2b2 )
(a2 + b2 + c2 + 3abc)2
≥ 2 2
a b + b2 c2 + c2 a2 + 2abc2 + 2a2 bc + 2ab2 c
(a2 + b2 + c2 + 3abc)2
= .
(ab + bc + ca)2
a2 + b2 + c2 + 3abc
Tiếp theo ta cần chứng minh ≥ 2. Ta có hai cách cho bước này.
ab + bc + ca
Cách 1: Ta trước hết nêu ra bất đẳng thức sau
(a + b + c)3 + 9abc ≥ 4(a + b + c)(ab + bc + ca). (2.7)
CÂU LẠC BỘ TOÁN A1, HOTLINE: 034 761 1986 - 035 290 3286 77

Bất đẳng thức trên tương đương với

∑ a(a − b)(a − c) ≥ 0.
cyc

Đây chính là bất đẳng thức Schur.


12q − 27 4q − 9
Đặt q = ab + bc + ca, r = abc. Từ bất đẳng thức (2.7) ta suy ra r ≥ = .
9 3
Khi đó
a2 + b2 + c2 + 3abc 9 − 2q + 3r
=
ab + bc + ca q
9 − 2q + 4q − 9
≥ = 2.
q

1
Ta có điều phải chứng minh.

A
Cách 2: Trong ba số a − 1, b − 1, c − 1 phải có hai số cùng dấu (theo nguyên lý Dirichlet), không
mất tính tổng quát giả sử hai số dương đó là b − 1, c − 1. Khi đó

á
a(b − 1)(c − 1) ≥ 0

n
To
dẫn đến abc ≥ ab + ac − a.
Do đó ta có

a2 + b2 + c2 + 3abc ≥ a2 + b2 + c2 + 3ab + 3ac − 3a


cb

= a2 + b2 + c2 + 3ab + 3ac − a(a + b + c)


= b2 + c2 + 2ab + 2ac
= (b − c)2 + 2(ab + bc + ca) ≥ 2(ab + bc + ca).
lạ

Ta có điều phải chứng minh.


u

78 TUYỂN TẬP ĐỀ THI VÀO LỚP 10 CHUYÊN TOÁN

2.9. ĐỀ THI TUYỂN SINH VÀO LỚP 10 TRƯỜNG THPT CHUYÊN


KHOA HỌC TỰ NHIÊN NĂM 2020, VÒNG 1
Câu 1
1. Giải hệ phương trình
(
(x + y)(x + 1) = 4
.
(y2 + xy + x + y + 5)(x3 + y3 + 12y + 13) = 243.

2. Giải phương trình

1
(x − 12)7 + (2x − 12)7 + (24 − 3x)7 = 0.

A
Lời giải.

n
1. Từ đề bài suy ra x2 + xy + x + y = 4. Suy ra
(y2 + xy + x + y + 5)(x3 + y3 + 12y + 13) á
To
= (y2 +xy+x+y+x2 +xy+x+y+1)(x3 +y3 +3y(x2 +xy+x+y)+3(x2 +xy+x+y)+1)

= (x + y + 1)5 .
cb

Từ đó, ta phải có x + y = 2. Giải tìm được x = y = 1.

2. Đặt a = x − 12, b = 2x − 12, c = 24 − 3x thế thì a + b + c = 0. Từ đây, ta có


lạ

• a7 + b7 = (a3 + b3 )(a4 + b4 ) + a3 b3 c,
• a3 + b3 = (a + b)3 − 3ab(a + b) = −c3 + 3abc,
u

• a4 + b4 = (a2 + b2 )2 − 2a2 b2 = c4 − 4abc2 + 2a2 b2 .

Từ các đẳng thức trên, suy ra

a7 + b7 = (−c3 + 3abc)(c4 − 4abc2 + 2a2 b2 ) + 3a3 b3 c.

Khai triển, ta thu được

a7 +b7 +c7 = 7abc(c2 (c2 −2ab)+a2 b2 ) = 7abc(c2 (a2 +b2 )+a2 b2 ) = 7abc(a2 b2 +b2 c2 +c2 a2 ).

Theo giả thiết vì a7 + b7 + c7 = 0 nên từ đẳng thức trên ta phải có ít nhất một trong ba
thừa số a, b, c bằng 0 hoặc ab = bc = ca = 0. Tuy nhiên trường hợp hai không thể xảy ra,
do đó ta phải có ít nhất một trong ba thừa số a, b, c bằng 0. Từ đây tìm được x ∈ {12, 6, 8}
và các giá trị này thỏa mãn. Tóm lại, phương trình đã cho có ba nghiệm là 6, 8, 12.
CÂU LẠC BỘ TOÁN A1, HOTLINE: 034 761 1986 - 035 290 3286 79

Câu 2
1. Tìm tất cả các số nguyên dương a, b, c sao cho cả ba số 4a2 + 5b, 4b2 + 5c, 4c2 + 5a
đều là bình phương của số nguyên dương.

2. Từ một bộ bốn số thực (a, b, c, d) ta xây dựng bộ số mới (a + b, b + c, c + d, d + a)


và liên tiếp xây dựng các bộ số mới theo quy tắc trên. Chứng minh rằng nếu có hai
thời điểm khác nhau ta thu được cùng một bộ số (có thể khác thứ tự) thì bộ số ban
đầu phải có dạng (a, −a, a, −a).

Lời giải.
1. Không mất tổng quát, giả sử a = max{a, b, c}. Thế thì vì (2a)2 < 4a2 + 5b < (2a + 2)2
16a + 9

1
nên 4a2 + 5b = (2a + 1)2 . Suy ra 5b = 4a + 1. Tương tự thì 5c ≥ 4b + 1 = hay
5

A
25c − 9
a≤ < 2c. Từ đó
16
(2c)2 < 4c2 + 5a < (2c + 3)2

n
nên ta phải có 4c2 + 5a ∈ {(2c + 1)2 , (2c + 2)2 }. Từ đây, xét hai trường hợp
á
• Nếu 4c2 + 5a = (2c + 1)2 thì 5a = 4c + 1 và do a ≥ c nên trong trường hợp này thì
To
a = b = c = 1.
128 64
• Nếu 4c2 + 5a = (2c + 2)2 thì 5a = 8c + 4. Khi đó 16a = c+ > 25c ≥ 16a + 9
5 5

vô lí.
cb

Tóm lại a = b = c = 1.

2. Giả sử ở thời điểm thứ n ta thu được bộ số (an , bn , cn , dn ) và nếu đặt Sn = an + bn + cn + dn


thì Sn = 2Sn−1 nên suy ra Sn = 2n S0 với S0 = a + b + c + d. Vì tồn tại hai thời điểm ta thu
lạ

được cùng một bộ số nên S0 = 0, kéo Sn = 0 với mọi n ∈ N. Nếu đặt Pn = a2n + b2n + c2n + dn2
thì
u

2
Pn+1 = 2Pn + 2(an + cn )(bn + dn ) = 2Pn + 2Sn−1 = 2Pn ∀n ≥ 1.

Suy ra Pn = 2n−1 P1 với mọi n ≥ 2. Cũng vì tồn tại hai thời điểm thu được hai bộ số giống
nhau nên P1 = 0, suy ra a1 = b1 = c1 = d1 = 0. Điều đó có nghĩa là bộ số ban đầu phải là
(a, −a, a, −a).
80 TUYỂN TẬP ĐỀ THI VÀO LỚP 10 CHUYÊN TOÁN

Câu 3
Cho tam giác ABC cân tại A với ∠BAC < 90◦ . Điểm E thuộc cạnh AC sao cho ∠AEB >
90◦ . Gọi P là giao điểm của BE và trung trực của BC. Gọi K là hình chiếu vuông góc của
P lên AB. Gọi Q là hình chiếu vuông góc của E lên AP. Gọi giao điểm của EQ và PK là
F.

1. Chứng minh rằng bốn điểm A, E, P, F cùng thuộc một đường tròn.

2. Gọi giao điểm của KQ và PE là L. Chứng minh rằng LA vuông góc với LE.

3. Gọi giao điểm của FL và AB là S. Gọi giao điểm của KE và AL là T . Lấy R là điểm
đối xứng của A qua L. Chứng minh rằng đường tròn ngoại tiếp tam giác AST và
đường tròn ngoại tiếp tam giác BPR tiếp xúc với nhau.

1
A
A

á n
To
L
S T

Q
F
E
cb

G
K
x X R
P
lạ
u

C
B

Lời giải. 1. Do ∠AQF = ∠AKF = 90◦ nên tứ giác AQKF là tứ giác nội tiếp. Vì vậy ∠PFE =
∠KFQ = ∠KAQ = ∠EAQ. Do đó, tứ giác AEPF là tứ giác nội tiếp.

2. Do hai tứ giác AEPF và AQKF là các tứ giác nội tiếp nên ∠AEL = ∠AFK = ∠AQL. Như
vậy tứ giác AQEL là tứ giác nội tiếp và vì thế nên ∠ALE = 90◦ .

3. Do các tứ giác AQKF và AQEL nội tiếp nên ta có PK · PF = PQ · PA = PE · PL. Như


vậy, tứ giác EKFL nội tiếp và ∠PKE = ∠PLS. Chú ý rằng ∠AKP = ∠ALE = 90◦ nên
ta suy ra ∠ALS = ∠AKE. Do đó, tứ giác KSLT nội tiếp. Từ đó suy ra ∠AST = ∠ALK =
∠APK = ∠ABC, dẫn đến ST k BC.
CÂU LẠC BỘ TOÁN A1, HOTLINE: 034 761 1986 - 035 290 3286 81

Gọi G là hình chiếu của P trên AC. Ta sẽ chứng minh ba điểm B, G, T thẳng hàng. Thật
LE LA LA LB
vậy, từ 4ELA ∼ 4EGP và 4BKP ∼ 4BLA, ta suy ra GE = PG = PK = KB . Chú ý rằng
KA = GA nên ta có
KA GE LB GE LB
· · = · = 1.
KB GA LE LE KB
Sử dụng định lý Ceva cho tam giác EAB ta suy ra EK, AL, BG đồng quy. Mà T là giao
điểm KE và AL, nên dẫn đến ba điểm B, G, T thẳng hàng.
Gọi X là giao điểm thứ hai của BG và đường tròn đường kính AP. Khi đó, từ tứ giác nội
tiếp APXG và A, R đối xứng qua đường thẳng BP, ta có ∠PXB = ∠PAC = ∠PAB = ∠PRB
và vì vậy X ∈ (BPR).
Trong khi đó, chú ý rằng ST k KG (cùng k BC) và B, X, G, T thẳng hàng, ta cũng có
∠AXT = ∠AXG = ∠AKG = ∠AST nên X ∈ (AST ).

1
Như vậy, điểm X nằm trên cả hai đường tròn (AST ) và (BPR). Khi đó, do ∠BLT = ∠90◦
nên ∠PBX + ∠XTA = 90◦ = ∠PXA. Kẻ tia tiếp tuyến Xx của đường tròn (AST ) thì ta

A
được ∠xXA = ∠XTA, dẫn đến ∠xXP = ∠PBX. Vì vậy Xx cũng tiếp xúc với (BPR). Do
đó, (AST ) tiếp xúc với đường tròn (BPR).

á n
Nhận xét. Ý 3 của bài toán này có thể phát biểu gọn lại thành bài toán sau:
To
Bài toán. Cho tam giác ABC cân tại A với ∠BAC < 90◦ . P là một điểm bất kỳ nằm
trên trung trực của BC. L, G lần lượt là hình chiếu của A, P trên các đường thẳng

BP, AC. Gọi T là giao điểm AL và BG. R là điểm đối xứng với A qua L. Dựng điểm
S trên cạnh AB sao cho ST k BC. Chứng minh rằng hai đường tròn (AST ) và (BPR)
cb

tiếp xúc với nhau.


lạ

Câu 4
Cho a, b, c là các số thực dương thỏa mãn a + b + c = 3. Chứng minh rằng
u

2

  
1 1 1 4 a b c
3 + + −1 +1 ≥ +3 + + .
a b c abc bc ca ab

Lời giải. Đặt x = 1a , y = 1b , z = 1c , ta có


1 1 1
+ + = 3. (2.8)
x y z
Hơn nữa bất đẳng thức trở thành
 
2 xy yz zx
3(x + y + z − 1) + 1 ≥ 4xyz + 3 + + (2.9)
z x y
Đặt p = x + y + z, q = xy + yz + zx, r = xyz. Khi đó điều kiện (2.8) trở thành

q = 3r.
82 TUYỂN TẬP ĐỀ THI VÀO LỚP 10 CHUYÊN TOÁN

Hơn nữa ta có
 
xy yz zx 1 1 1
+ + = xyz 2 + 2 + 2
z x y x y z
" #
1 1 1 2 2

2 2
= xyz + + − − −
x y z xy yz zx
 
2p
= r 9−
r
= 9r − 2p

cho nên bất đẳng thức (2.9) trở thành

3(p − 1)2 + 1 ≥ 4r + 3(9r − 2p).

1
A
Khai triển ra thì ta có bất đẳng thức tương đương với

3p2 + 4 ≥ 31r.

Áp dụng bất đẳng thức Schur


á n
To
(x + y + z)3 + 9xyz ≥ 4(x + y + z)(xy + yz + zx)

ta có
p3 + 9r ≥ 4pq = 12pr.

 
cb

1 1 1 1 1 1
Vì (x + y + z) + + ≥ 9 và + + = 3 nên p = x + y + z ≥ 3. Thế thì từ p3 + 9r ≥
x y z x y z
12pr ta suy ra
31p3
lạ

≥ 31r.
3(4p − 3)
31p3
Ta sẽ chứng minh 3p2 + 4 ≥
u

, từ đây sẽ hoàn tất chứng minh. Bất đẳng thức này tương
3(4p − 3)

đương với
(p − 3)(5p2 − 12p + 12) ≥ 0.
Bất đẳng thức cuối cùng này luôn đúng vì p ≥ 3 và 5p2 − 12p + 12 > 0.
CÂU LẠC BỘ TOÁN A1, HOTLINE: 034 761 1986 - 035 290 3286 83

2.10. ĐỀ THI TUYỂN SINH VÀO LỚP 10 TRƯỜNG THPT CHUYÊN


SƯ PHẠM HÀ NỘI NĂM 2020, VÒNG 2
Câu 1
Cho ba số thực x, y, z thỏa mãn các điều kiện sau

3 = 3y3 = 4z3
2x

p √ √
3
2x2 + 3y2 + 4z2 = 2 + 3 12 + 3 16

xyz > 0.

1 1 1
Tính giá trị của biểu thức P = + + .

1
x y z

A
Lời giải. Đặt 2x3 = 3y3 = 4z3 = t. Khi đó

n
s  
p
3 1 1 1
2x2 + 3y2 + 4z2 =
á3
t + +
x y z
.
To
√ √ √
√ 3
t √ 3
t √ 3
t
Mặt khác 3 2 = 3
, 3= , 34= z nên
x y

√ √ √ √ √ √  √ √
3
t

3
t

3
t



1 1 1

3 3 3 3 3 3 3 3
2 + 12 + 16 = 4 2+ 3+ 4 = 4 + + = 4t + +
cb

x y z x y z
Từ đó ta suy ra s 

lạ

  
3 1 1 1 3 1 1 1
t + + = 4t + + .
x y z x y z
1 1 1 1
u

Từ đó ta tìm được + + = .
x y z 2

Câu 2
Xét phương trình bậc hai ax2 + bx + c = 0 (1), trong đó a, b, c là các số nguyên dương.
Biết rằng các điều kiện sau được thỏa mãn: phương trình (1) có nghiệm, số a2020b chia
hết cho 12, số c2 + 3 chia hết cho c + 3. Hãy tìm giá trị lớn nhất của tổng a + b + c.

(
3 | a + b + 4,
Lời giải. Vì a2020b chia hết cho 12 nên
4 | b.
Từ đó suy ra b ∈ {0, 4, 8}.
Lại có b2 − 4ac ≥ 0 nên b2 > 0 và ac ≤ 16. Suy ra b ∈ {4, 8} và c ≤ 16. Hơn nữa c + 3 là ước của
c3 + 3 = c3 + 27 − 24 = (c + 3)(c2 − 3c + 9) − 24 nên c + 3 là ước của 24. Kết hợp với c + 3 ≥ 4
ta có c + 3 ∈ {4, 6, 8, 12, 24}, hay c ∈ {1, 3, 5, 9, 21}. Kết hợp với c ≤ 16 ta có c ∈ {1, 3, 5, 9}.
Xét các trường hợp
84 TUYỂN TẬP ĐỀ THI VÀO LỚP 10 CHUYÊN TOÁN


ac ≤ 16,

• b = 8. Khi đó 3 | a,

c ∈ {1, 3, 5, 9}.

Thử tất cả các bộ số ta thấy a + c max khi (a, c) = (9, 1). Lúc này a + b + c = 18.

ac ≤ 4,

• b = 4. Khi đó a ≡ 1 (mod 3),

c ∈ {1, 3, 5, 9}.

Thử tất cả các bộ số ta thấy a + c max khi (a, c) = (4, 1). Lúc này a + b + c = 9.
Từ hai trường hợp trên ta suy ra giá trị lớn nhất của a + b + c bằng 18, xảy ra tai (a, b, c) =
(9, 8, 1).

1
A
Câu 3
Tìm số nguyên a bé nhất sao cho x4 + 2x2 − 4x + a ≥ 0 với mọi số thực x.

á
Lời giải. Ta chứng minh giá trị nguyên nhỏ nhất của a là 2.

n
To
Trước hết, ta chứng minh a = 2 thõa mãn điều kiện đề bài, nghĩa là x4 + 2x2 − 4x + 2 ≥ 0 với
mọi x. Thật vậy, ta có

x4 + 2x2 − 4x + 2 = x4 + 2(x − 1)2 ≥ 0 ∀x.



cb

Bây giờ ta chứng minh a ≤ 1 không thỏa mãn đề bài, nghĩa là tồn tại x để

x4 + 2x2 − 4x + a < 0 với a ≤ 1.


lạ

Ta chỉ cần chứng minh tồn tại x để

x4 + 2x2 − 4x + 1 < 0
u

1 8
Chọn x = thì x4 + 2x2 − 4x + 1 = − < 0.

3 81
Kết luận. amin = 2.

Câu 4
Cho tam giác nhọn ABC nội tiếp đường tròn (O) có AB > BC. Một đường tròn đi qua hai
đỉnh A,C của tam giác ABC lần lượt cắt các cạnh AB, BC tại hai điểm K, N (K, N khác các
đỉnh của tam giác ABC.) Giả sử đường tròn (O) và đường tròn ngoại tiếp của tam giác
BKN cắt nhau tại giao điểm thứ hai là M (M khác B). Chứng minh rằng

1. Ba đường thẳng BM, KN, AC đồng quy tại điểm P.

2. Tứ giác MNCP nội tiếp.

3. BM 2 − PM 2 = BK · BA − PC · PA.
CÂU LẠC BỘ TOÁN A1, HOTLINE: 034 761 1986 - 035 290 3286 85

M
K

N
O

J
P C A

1
A
n
Lời giải.
á
1. Gọi P là giao điểm của NK và BC, M 0 là giao điểm thứ hai (khác B) của PB và đường tròn
To
(O). Do các tứ giác ACNK và ABM 0C nội tiếp nên ta được PM 0 · PB = PA · PC = PN · PK.
Từ đó suy ra tứ giác BKNM 0 nội tiếp. Nói cách khác M 0 là giao điểm khác B của hai đường
tròn (O) và đường tròn (BKN). Do đó, M ≡ M 0 dẫn đến ba đường thẳng BM, KN, AC đồng

quy tại điểm P.


cb

2. Từ các tứ giác nội tiếp ACNK và BKNM ta có ∠NCP = ∠NKA = ∠NMB. Do vậy, tứ giác
MNCP nội tiếp.
lạ

3. Từ các tứ giác nội tiếp ACNK, NMCP, ta được BK · BA = BN · BC = BM · BP. Từ tứ giác


nội tiếp ABMC ta suy ra PC · PA = PM · PB. Vì vậy ta thu được
u

BK · BA − PC · PA = BM · BP − PM · PB = (BM − PM)BP

= (BM − PM)(BM + PM) = BM 2 − PM 2 .

Câu 5
Cho hai số A, B cùng có 2020 chữ số. Biết rằng số A có đúng 1945 chữ số khác 0, bao
gồm 1930 chữ số ngoài cùng về bên trái và 15 chữ số ngoài cùng về bên phải, số B có
đúng 1954 chữ số khác 0, bao gồm 1930 chữ số ngoài cùng về bên trái và 24 chữ số ngoài
cùng về bên phải. Chứng minh rằng UCLN(A, B) là một số có không quá 1954 chữ số.

Lời giải. Ta viết dạng tổng quát của hai số A và B như sau:

A = x0 . . . 0y, B = z0 . . . 0t,
86 TUYỂN TẬP ĐỀ THI VÀO LỚP 10 CHUYÊN TOÁN

trong đó x, z có 1930 chữ số, y có 15 chữ số, t có 24 chữ số, A và B đều có tổng cộng 2020 chữ
số. Phân tích cấu tạo số của hai số A, B ta được

A = 1090 x + y, B = 1090 z + t.

Đặt d = (A, B). Khi đó d là ước của xB − zA, tức là d là ước của xt − yz. Không khó để thấy
xt > yz, nên xt − yz > 0. Do đó d ≤ xt − yz < xt. Lại có x < 101930 , t < 1024 (do x có 1930 chữ
số và t có 24 chữ số) nên
d < xt < 101930 .1024 = 101954 .
Từ đó ta suy ra d có không quá 1954 chữ số.

1
A
á n
To

cb
lạ
u

CÂU LẠC BỘ TOÁN A1, HOTLINE: 034 761 1986 - 035 290 3286 87

2.11. ĐỀ THI TUYỂN SINH VÀO LỚP 10 CHUYÊN TOÁN HÀ


NỘI, NĂM 2020
Câu 1

1. Giải phương trình x2 + 3x + 5 = (x + 3) x2 + 5.

2. Cho các số thực a, b, c thỏa mãn a + b − 2c = 0 và 2ab − bc − ca = 0. Chứng minh


rằng a = b = c.

Lời giải.

1
1. Phương trình ban đầu tương đương với

A
p p
x2 + 5 − x x2 + 5 + 3x − 3 x2 + 5 = 0.

n
p p
⇐⇒ ( x2 + 5 − x)( x2 + 5 − 3) = 0.
√ √ á √
To
Từ đó ta suy ra x2 + 5 = x√hoặc x2 + 5 = 3. Giải phương trình x2 + 5 = x suy ra vô
nghiệm. Giải phương trình x2 + 5 = 3 suy ra x = ±2.
Vậy S = {±2}.

2. Chú ý rằng a + b = 2c thì ta có 2ab = bc + ca = c(a + b) = 2c2 hay ab = c2 . Từ đó ta suy


cb

ra (a − b)2 = (a + b)2 − 4ab = (2c)2 − 4c2 = 0. Như vậy a = b. Thay vào a + b = 2c ta


suy ra a = b = c.
lạ

Câu 2
u

1. Chứng minh với mọi số nguyên dương n, số A = 11n + 7n − 2n − 1 chia hết cho 15.

√ m
2. Cho hai số nguyên dương m và n thỏa mãn 11 − > 0. Chứng minh rằng
n

√ m 3( 11 − 3)
11 − ≥ .
n mn

.
Lời giải. 1. Chú ý rằng (an − bn ) .. (a − b) với a, b, n ∈ Z+ nên ta có

.
A = (11n − 2n ) + (7n − 1n ) .. 3,

.
A = (11n − 1n ) + (7n − 2n ) .. 5.
Từ đó ta có điều phải chứng minh.
88 TUYỂN TẬP ĐỀ THI VÀO LỚP 10 CHUYÊN TOÁN

2. Ta cần chứng minh bất đẳng thức sau



√ 3( 11 − 3)
11n − m ≥ .
m

Bất đẳng thức này tương đương với



2 2
√ 9( 11 − 3)2
11n ≥ m + 6( 11 − 3) + .
m2
Ta xét ba trường hợp như sau
√ √
• Nếu m = 1 thì V P = 1 + 6( 11 − 3) + 9( 11 − 3)2 < 11 ≤ 11n2 .

1
√ 9( 11 − 3)2
• Nếu m = 2 thì V P = 4 + 6( 11 − 3) + < 11 ≤ 11n2 .

A
4

2
√ 9( 11 − 3)2
• Nếu m ≥ 3 thì V P ≤ m + 6( 11 − 3) + = m2 + 2.

n
9
Ta sẽ chứng minh 11n2 ≥ m2 + 2 ⇐⇒ 11n2 − m2 ≥ 2.

á
Thật vậy, từ giả thiết 11 − mn > 0 ta suy ra 11n2 − m2 > 0 và do đó 11n2 − m2 ≥ 1.
To
Nếu 11n2 − m2 = 1 thì m2 chia 11 dư 10, tuy nhiên điều này không thể xảy ra (Kiểm
tra bằng đồng dư). Từ đó suy ra 11n2 − m2 ≥ 2. Chứng minh của bài toán hoàn tất.

cb

Câu 3
1. Cho đa thức P(x) với hệ số thực thỏa mãn P(1) = 3 và P(3) = 7. Tìm đa thức dư
trong phép chia đa thức P(x) cho đa thức x2 − 4x + 3.
lạ

2. Với a, b, c là các số thực không âm thỏa mãn a + b + c + abc = 4, tìm giá trị lớn
nhất của biểu thức P = ab + bc + ca.
u

Lời giải.
1. Hiển nhiên deg P ≥ 1. Khi đó ta có thể viết

P(x) = (x − 1)(x − 3)Q(x) + ax + b

với a, b ∈ R. Thay x = 1 và x = 3 vào phương trình trên ta có


(
a + b = P(1) = 3
3a + b = P(3) = 7.

Giải hệ phương trình ta được a = 2, b = 1. Vậy dư của P(x) trong phép chia cho x2 − 4x +
3 = (x − 1)(x − 3) là 2x + 1.
CÂU LẠC BỘ TOÁN A1, HOTLINE: 034 761 1986 - 035 290 3286 89

4 − (a + b)
2. Cách 1. Ta có a + b + c + abc = 4 nên c = (1).
ab + 1
Dự đoán: Dấu đẳng thức xảy ra khi a = b = 2, c = 0. Khi đó P = 4.
Ta cần chứng minh P ≤ 4, tương đương với ab + c(a + b) ≤ 4. Thay (1) vào và biến đổi
thì bất đẳng thức tương đương với

(ab + 1)(ab − 4) ≤ (a + b)(a + b − 4).

Giả sử (a − 1)(b − 1) ≥ 0 (Dirichlet) thì ab + 1 ≥ a + b ≥ 0.



+ abc ≥ a + b nên a + b ≤ 4. Mặt khác 4 ≥ a + b ≥ 2 ab, nên ab ≤ 4.
Do 4 = a + b + c√
Khi đó a + b ≥ 2 ab ≥ ab, cho nên 4 − ab ≥ 4 − a − b.
Khi đó (ab + 1)(4 − ab) ≥ (a + b)(4 − a − b), bất đẳng thức đã được chứng minh xong.

1
Dấu bằng xảy ra, ví dụ tại a = b = 2, c = 0.

A
Cách 2. Không mất tính tổng quát, giả sử a = max{a, b, c}. Từ giả thiết ta suy ra a3 +3a ≥
4. Bất đẳng thức này tương đương với (a − 1)(a2 + a + 4) ≥ 0. Lại do a2 + a + 4 > 0 nên

n
ta suy ra a ≥ 1 hay a2 ≥ 1. Từ đó suy ra a2 bc ≥ bc. Như vậy,
á
To
P = ab + bc + ca = a(b + c) + bc ≤ a(b + c) + a2 bc
(a + b + c + abc)2
= a(b + c + abc) ≤ = 4.
4

Dấu bằng xảy ra khi (a, b, c) là hoán vị của (2, 2, 0).


cb

Câu 4
lạ

Cho tam giác ABC có ba góc nhọn và AB < AC. Gọi (I) là đường tròn nội tiếp của tam
giác ABC và K là tâm đường tròn bàng tiếp trong góc A của tam giác ABC. Gọi D, E, F lần
u

lượt là chân các đường vuông góc kẻ từ điểm I đến các đường thẳng BC,CA, AB. Đường
thẳng AD cắt đường tròn (I) tại hai điểm phân biệt D và M. Đường thẳng qua K song

song với đường thẳng AD cắt đường thẳng BC tại N.

1. Chứng minh rằng tam giác MFD đồng dạng với tam giác BNK.

2. Gọi P là giao điểm của BI và FD. Chứng minh rằng ∠BMF = ∠DMP.

3. Chứng minh rằng đường tròn ngoại tiếp của tam giác MBC đi qua trung điểm của
đoạn thẳng KN.

Lời giải.
1. Trước hết để ý rằng D, E, F chính là ba tiếp điểm của (I) với BC,CA, AB. Ta có FD k BK
(cùng ⊥ BI) nên ∠FMD = ∠FDB = ∠NBK. Chú ý rằng NK k AD nên ta được ∠BNK =
∠ADC = ∠MFD. Từ đó suy ra 4MFD ∼ 4BNK (gg).
90 TUYỂN TẬP ĐỀ THI VÀO LỚP 10 CHUYÊN TOÁN

M
E
G
I
F

B N C

1
D

A
X

á n
To
K

2. Bổ đề. Cho tam giác ABC với đường cao AH (H ∈ BC) và tâm ngoại tiếp O. Khi đó,
cb

∠HAB = ∠OAC.

Gọi G là hình chiếu của M trên FD. Áp dụng hệ thức lượng trong tam giác vuông thì ta
lạ

có IM 2 = ID2 = IP · IB. Từ đó suy ra IM IB


IP = IM , kéo theo 4IMP ∼ 4IBM (cgc). Như vậy,
∠IMP = ∠IBM = ∠GMB. Mặt khác, áp dụng bổ đề trên cho tam giác MFD ta suy ra
∠GMF = ∠IMD. Vì vậy, ta được
u

∠BMF = ∠GMB − ∠GMF = ∠IMP − ∠IMD = ∠DMP.

3. Để ý rằng từ câu 2 ta suy ra được ∠FMP = ∠DMB. Gọi X là trung điểm của KN. Chú ý
rằng P là trung điểm của FD và từ câu 1 ta có 4MFD ∼ 4BNK với hai trung tuyến tương
ứng là MP và BX. Từ đó suy ra 4BNX ∼ 4MFP nên ta thu được ∠XBC = ∠FMP =
∠DMB. Chứng minh tương tự câu 1 ta có 4MED ∼ 4CNK và tiếp tục chứng minh tương
tự như trên ta thu được ∠XCB = ∠DMC. Như vậy, ∠BMC + ∠BXC = ∠DMB + ∠DMC +
∠BXC = ∠XBC + ∠XCB + ∠BXC = 180◦ . Do đó, tứ giác MBXC nội tiếp hay đường tròn
(MBC) luôn đi qua trung điểm X của đoạn thẳng KN.
CÂU LẠC BỘ TOÁN A1, HOTLINE: 034 761 1986 - 035 290 3286 91

Câu 5
Cho một bảng ô vuông kích thước 6 × 7 (6 hàng, 7 cột) được tạo bởi các ô vuông có kích
thước 1 × 1. Mỗi ô vuông có kích thước 1 × 1 được tô bởi một trong hai màu đen hoặc
trắng sao cho trong mọi bảng ô vuông kích thước 2 × 3 hoặc 3 × 2, có ít nhất hai ô vuông
kích thước 1 × 1 được tô màu đen có chung cạnh. Gọi m là số ô vuông kích thước 1 × 1
được tô màu đen trong bảng

1. Chỉ ra một cách tô sao cho m = 20.

2. Tìm giá trị nhỏ nhất của m.

Lời giải. 1. Ta chỉ ra một cách tô như sau

1
A
á n
To

cb
lạ

2. Xét ba ô như sau


u

thì khi đó theo giả thiết, trong ba ô này có ít nhất một ô được tô màu đen.
Xét bốn ô như sau

Trong ba ô 1, 2, 3 có ít nhất một ô được tô màu đen. Giả sử ô được tô là ô 1. Trong ba ô


2, 3, 4 có ít nhất một ô được tô màu đen. Như vậy, trong bốn ô này có ít nhất 2 ô được tô
màu đen.
Trong bảng 6 × 7, ta đánh số các ô như hình vẽ sau
92 TUYỂN TẬP ĐỀ THI VÀO LỚP 10 CHUYÊN TOÁN

2 4

7 1 7 8 2 8

1 7 1 3 2 8 2

1 3 5 3 2

1
4 5 3 5 6

A
4 9 4 5 6 10 6

n
9 4 9 10 6 10

á
To
• Trong nhóm các ô được đánh số từ 1 đến 6, mỗi nhóm ô có ít nhất hai ô được tô màu
đen.
• Trong nhóm các ô được đánh số từ 7 đến 10, mỗi nhóm ô có ít nhất một ô được tô

màu đen.
cb

Do đó, số ô được tô màu tối thiểu là 2 · 6 + 1 · 4 = 16 ô.


lạ
u

CÂU LẠC BỘ TOÁN A1, HOTLINE: 034 761 1986 - 035 290 3286 93

Sau đây là một cách để tô được 16 ô đen.

1
A
á n
To

cb
lạ
u

94 TUYỂN TẬP ĐỀ THI VÀO LỚP 10 CHUYÊN TOÁN

2.12. ĐỀ THI TUYỂN SINH VÀO LỚP 10 CHUYÊN TIN HÀ


NỘI, NĂM 2020
Câu 1

1. Giải phương trình (x + 2) x2 + 1 = x2 + 2x + 1.

2. Chứng minh rằng


1 1 1 1
√ √ + √ √ +···+ √ √ = 1− √ .
2 1+1 2 3 2+2 3 2021 2020 + 2020 2021 2021

1
Lời giải.

A
1. Phương trình ban đầu tương đương với
p p
x2 + 1 − x x2 + 1 + 2x − 2 x2 + 1 = 0.

n
p p
⇐⇒ ( x2 + 1 − x)( x2 + 1 − 2) = 0.
√ √ √
á
Từ đó ta suy ra x2 + 1 = x√hoặc x2 + 1 = 2. Giải phương
√ trình x2 + 1 = x suy ra vô
To
nghiệm. Giải phương trình x2 + 1 = 2 suy ra x = ± 3.

Vậy S = {± 3}.
2. Xét số hạng tổng quát

1 (n + 1) − n

cb

√ =p √ √
(n + 1) n + n n + 1 n(n + 1)( n + n + 1)
√ √
n+1− n 1 1
= √ √ = √ −√ .
n n+1 n n+1
lạ

Vì vậy,
1 1 1 1 1 1 1
u

VT = √ − √ + √ − √ +···+ √ −√ = 1− √ .
1 2 2 3 2020 2021 2021

Câu 2
1. Chứng minh rằng với mọi số nguyên dương n, số A = 59n − 17n − 9n + 2n chia hết
cho 35.

2. Tìm tất cả các số nguyên x, y thỏa mãn điều kiện x2 y − 3y − 4x − 1 = 0.

.
Lời giải. 1. Chú ý rằng (an − bn ) .. (a − b) với a, b, n ∈ Z+ nên ta có
.
A = (59n − 9n ) − (17n − 2n ) .. 5,
.
A = (59n − 17n ) − (9n − 2n ) .. 7.
Từ đó ta có điều phải chứng minh.
CÂU LẠC BỘ TOÁN A1, HOTLINE: 034 761 1986 - 035 290 3286 95

2. Phương trình ban đâu tương đương với y(x2 − 3) = 4x + 1. Chú ý rằng x2 − 3 6= 0 với mọi
x ∈ Z. Do đó, ta được
4x + 1
y= 2 .
x −3
16x2 − 1 47
Ta có y ∈ Z =⇒ 2
= 16 + 2 ∈ Z =⇒ x2 − 3 | 47. Do đó, chú ý rằng x2 − 3 ≥
x −3 x −3
−3, ta được x2 − 3 ∈ {±1, 47}. Vì x ∈ Z, ta thu được x = ±2.
Đáp số. (x, y) = (2, 9); (−2, −7).

Câu 3
1. Tìm tất cả các số thực a, b, c thỏa mãn đồng thời các điều kiện a2 + b2 + c2 =

1
38, a + b = 8 và b + c ≥ 7.

A
2. Với a, b, c là các số thực không√âm và luôn thỏa mãn a2 +b2 +c2 = 2ab+2bc+2ca,
chứng minh rằng a + b + c ≥ 3 2abc.

Lời giải. á n
To
1. 
2 2 2
a + b + c = 38
 (1)
a+b = 8 (2)


b+c ≥ 7 (3)

cb

Từ (1) và (2) ta suy ra a = 8 − b và c ≥ 7 − b. Thay vào (1) ta có

38 ≥ (8 − b)2 + b2 + (7 − b)2 = 3b2 − 30b + 113 = 3(b − 5)2 + 38 ≥ 38.


lạ

Suy ra b = 5. Từ (2) ta suy ra a = 3 và từ (1) ta suy ra c = 2. Vậy (a, b, c) = (3, 5, 2).


u

2. Từ a2 + b2 + c2 = 2ab + 2bc + 2ca ta suy ra


c2 − 2c(a + b) + (a2 + b2 − 2ab) = 0.


√ √ √ 2
Ta có ∆0 = (a + b)2 − (a2 + b2 − 2ab) = 4ab. Suy ra c = (a + b) + 2 ab = a+ b
√ √ √ 2
hoặc c = (a + b) − 2 ab = a− b .
√ √ 2
• c= a + b . Bất đẳng thức tương đương với
r
√ √ 2 3
√ √ 2
a+b+ a + b ≥ 3 2ab a+ b .

√ √ √
Đặt s = a + b, p = ab. Bất đẳng thức trở thành
p
2s2 − 2p ≥ 3 3 2s2 p2
96 TUYỂN TẬP ĐỀ THI VÀO LỚP 10 CHUYÊN TOÁN

Lập phương hai vế, chuyển vế và phân tích nhân tử, bất đẳng thức trở thành

2(s2 − 4p)(2s2 + p)2 ≥ 0.

Bất đẳng thức trên hiển nhiên đúng do s2 ≥ 4p (AM-GM) và (2s2 + p2 )2 ≥ 0.


√ √ 2
• c= a − b . Bất đẳng thức tương đương với
r
√ √ 2 3
√ √ 2
a+b+ a − b ≥ 3 2ab a− b .

√ √ √
Đặt s = a + b, p = ab. Bất đẳng thức trở thành

1
q
2s2 − 6p ≥ 3 3 2(s2 − 4p)p2 .

A
Lập phương hai vế, chuyển vế và phân tích nhân tử, bất đẳng thức trở thành

n
2s2 (2s2 − 9p)2 ≥ 0.
á
To
Bất đẳng thức trên luôn đúng

Vậy ta đã chứng minh được bất đẳng thức. Dấu bằng xảy ra tại ví dụ a = b = 1 và c = 4.

cb

Câu 4
Cho tam giác ABC có ba góc nhọn, AB < AC và ba đường cao AD, BE,CF cùng đi qua
điểm H. Gọi (S) là đường tròn ngoại tiếp của tam giác DEF.
lạ

1. Chứng minh rằng đường tròn (S) đi qua trung điểm của đoạn thẳng AH.
u

2. Gọi M và N lần lượt là giao điểm của đường tròn (S) với các đoạn thẳng BH và CH.
Tiếp tuyến tại D của đường tròn (S) cắt MN tại điểm T . Chứng minh rằng đường

thẳng HT song song với đường thẳng EF.

3. Gọi P là giao điểm của hai đường thẳng BH và DF, Q là giao điểm của hai đường
thẳng CH và DE. Chứng minh rằng ba điểm T, P, Q là ba điểm thẳng hàng.

Lời giải.
1. Gọi X là trung điểm của AH. Xét tam giác AFH vuông tại F với FX là trung tuyến nên
ta được XF = XA = XH và ∠FXH = 2∠FAH. Mặt khác, từ các tứ giác nội tiếp AEHF,
CDHE và AFDC ta có ∠BED = ∠BCH = ∠FAH = ∠FEB, dẫn tới EB là phân giác của
∠DEF. Vì vậy nên ∠FED = 2∠FEB = 2∠FAH = ∠FXD. Do đó, tứ giác DEXF nội
tiếp. Hay nói cách khác, đường tròn (S) ≡ (DEF) luôn đi qua trung điểm X của đoạn
thẳng AH.
CÂU LẠC BỘ TOÁN A1, HOTLINE: 034 761 1986 - 035 290 3286 97

X
E
F
H
S
P Q
T
M N

B D C

1
A
2. Chứng minh tương tự câu 1, ta suy ra M, N tương ứng là trung điểm của BH và CH.
Do MN là đường trung bình của tam giác HBC và HD ⊥ BC nên ta được MN chính là

n
trung trực của HD. Như vậy, các điểm T, M nằm trên trung trực của đoạn thẳng HD, kết
hợp với DT là tiếp tuyến của (S), ta suy ra ∠T HM = ∠T DM = ∠BED = ∠FEB. Vì vậy
HT k EF. á
To
MP
3. Từ ∠MDF = ∠MEF = ∠MED, ta suy ra 4MPD ∼ 4MDE (gg), dẫn tới MD = MD
ME .
MP MH MP MH
Chú ý rằng MD = MH nên ta được MH = ME =⇒ PH = HE . Chứng minh hoàn toàn
NQ
= NH

tương tự ta cũng được QH HF . Mặt khác, chú ý rằng tứ giác EFMN nội tiếp nên
4HEF ∼ 4HNM (gg) và kéo theo HF HM
HE = HN . Do đó, chú ý rằng HM = DM và HN = DN
cb

ta được
HM 2 DM 2
   
PM QH HM HF
· = · = = .
PH QN HE HN HN DN
lạ

TM
Trong khi đó, do DT là tiếp tuyến tại D của (S) nên 4T MD ∼ 4T DN, kéo theo TD =
TD DM
T N = DN . Do đó,
u

DM 2
 
TM TM TD

= · = .
TN TD TN DN
QH PM QH T N
Như vậy, PM TM
PH · QN = T N hay viết theo cách khác, PH · QN · T M = 1. Áp dụng định lý
Menelaus cho tam giác HMN ta suy ra ba điểm T, P, Q thẳng hàng.

Câu 5
Trên bàn có 6 hộp kẹo, mỗi hộp có 5 viên kẹo. An và Bình cùng chơi một trò chơi như
sau: mỗi lượt chơi, An sẽ chọn một hộp tùy ý và lấy ít nhất 1 viên kẹo trong hộp đó; còn
Bình thì chọn một số hộp và trong các hộp đã chọn, mỗi hộp lấy đúng 1 viên kẹo. Hai
bạn luân phiên thực hiện lượt chơi của mình. Bạn đầu tiên không thể thực hiện được lượt
chơi của mình là người thua cuộc. Nếu An là người lấy kẹo trước, hãy chỉ ra chiến thuật
chơi để Bình là người thắng cuộc.
98 TUYỂN TẬP ĐỀ THI VÀO LỚP 10 CHUYÊN TOÁN

Lời giải. Ta cho Bình bốc như sau: Sau mỗi lần An bốc, Bình sẽ bốc 1 viên từ những hộp còn
có nhiều hơn 1 viên kẹo. Sau 4 lượt, An chỉ có thể làm rỗng tối đa 4 hộp kẹo, tức là vẫn còn ít
nhất 2 hộp kẹo còn kẹo. Điều này cũng đảm bảo cho Bình có thể bốc theo cách nói trên trong
vòng 4 lượt này. Hơn nữa trong mỗi hộp kẹo còn có kẹo thì chỉ còn đúng 1 viên do cách bốc của
Bình. Khi đó Bình sẽ thắng ở lượt thứ 5 (sau khi Bình bốc 1 viên ở một hộp bất kỳ thì Bình sẽ
bốc nốt chỗ kẹo còn lại)

1
A
á n
To

cb
lạ
u

CÂU LẠC BỘ TOÁN A1, HOTLINE: 034 761 1986 - 035 290 3286 99

2.13. ĐỀ THI TUYỂN SINH VÀO LỚP 10 TRƯỜNG PHỔ THÔNG


NĂNG KHIẾU ĐHQG TPHCM NĂM 2020, VÒNG 2
Câu 1
Cho các phương trình x2 + ax + 3 = 0 và x2 + bx + 5 = 0 với a, b là các tham số

(a) Chứng minh rằng nếu ab ≥ 16 thì trong hai phương trình có ít nhất một phương trình
có nghiệm.

(b) Giả sử hai phương trình trên có nghiệm chung x0 . Tìm a, b sao cho |a| + |b| có giá trị
nhỏ nhất.

1
Lời giải.

A
(a) Với phương trình x2 + ax + 3 = 0 ta có ∆1 = a2 − 12. Với phương trình x2 + bx + 5 = 0 ta
có ∆2 = b2 − 20. Vì ab ≥ 16 nên ta có ∆1 + ∆2 = a2 + b2 − 32 ≥ 2ab − 32 ≥ 0. Từ đó suy ra

n
∆1 ≥ 0 hoặc ∆2 ≥ 0, dẫn đến trong hai phương trình có ít nhất một phương trình có nghiệm.
á x02 + 3
To
(b) Chú ý rằng x0 6= 0. Ta có x02 + ax0 + 3 = 0 và x02 + bx0 + 5 = 0, dẫn đến a = − và
x0
x02 + 5
b=− . Khi đó ta được
x0

x02 + 3 x02 + 5 2x02 + 8 2x02 + 8 8|x0 |


cb

|a| + |b| = + ≥ = ≥ = 8.
x0 x0 x0 |x0 | |x0 |

Dấu bằng xảy ra chẳng hạn khi x0 = 2 hoặc x0 = −2. Thay x0 = 2 vào hai phương trình thì
lạ

7 9 7
ta thu được a = − và b = − . Thay x0 = −2 vào hai phương trình thì ta thu được a =
2 2 2
9
u

và b = .
2

Kết luận. Giá trị nhỏ nhất của |a| + |b| là 8.

Câu 2
Cho phương trình 3x2 − y2 = 23n với n là số tự nhiên.

(a) Chứng minh rằng nếu n chẵn thì phương trình đã cho không có nghiệm nguyên (x, y).

(b) Chứng minh rằng nếu n lẻ thì phương trình đã cho có nghiệm nguyên (x, y).

Lời giải.
(a) Chú ý rằng y2 chia 3 dư 0 hoặc 1, nên 3x2 − y2 chia 3 dư 0 hoặc 2. Tuy nhiên nếu n chẵn thì
23n ≡ 2n ≡ 1 (mod 3), khi đó phương trình vô nghiệm.
100 TUYỂN TẬP ĐỀ THI VÀO LỚP 10 CHUYÊN TOÁN

(b) Ta dùng quy nạp ở câu này. Với n = 1 phương trình 3x2 − y2 = 23 có một nghiệm (x, y) =
(3, 2) là thỏa mãn. Giả sử bài toán đúng đến n = k lẻ, tức là tồn tại cặp số nguyên (xk , yk )
thỏa mãn 3xk2 − y2k = 23k . Khi đó ta có

23k+2 = (3xk2 − y2k ) · 232


= (3xk2 − y2k )(262 − 3.72 )
= 3 · 262 · xk2 − 32 · 72 xk2 − 262 y2k + 3 · 72 · y2k
= 3(26xk − 7yk )2 − (21xk − 26yk )2

Khi đó phương trình 3x2 − y2 = 23k+2 có một cặp nghiệm là (x, y) = (26xk − 7yk , 21xk −
26yk ), tức là bài toán cũng đúng với n = k + 2. Quy nạp hoàn tất.

1
Câu 3

A
Cho đường tròn (O), dây cung BC không chứa tâm O và điểm A thay đổi trên cung lớn
BC. Lấy các điểm E, F thỏa mãn ∠ABE = ∠CAE = ∠ACF = ∠BAF = 90◦ .

n
(a) Chứng minh rằng AE · AC = AF · AB và O là trung điểm của EF.
á
(b) Hạ AD vuông góc với EF (D ∈ EF). Chứng minh rằng các tam giác DAB và DCA
To
đồng dạng và D thuộc một đường tròn cố định.

(c) Gọi G là giao điểm của AD và đường tròn (O) (G 6= A). Chứng minh rằng AD đi qua
một điểm cố định và GB · AC = GC · AB.

cb

(d) Gọi K là tâm đường tròn ngoại tiếp của tam giác AEF. Chứng minh rằng AK đi qua
một điểm cố định.
lạ

Lời giải.
(a) Ta có ∠EAB = ∠FAC(= 90◦ − ∠BAC). Từ đó suy ra 4EAB ∼ 4FAC và AE AF
AB = AC , vậy nên
u

AE · AC = AF · AB. Dựng đường kính AX của đường tròn (O). Vì XE k AF và XF k AE nên


AEXF là hình bình hành. Do đó, EF đi qua trung điểm O của AX.
(b) Từ các tứ giác nội tiếp ADBE và ADCF ta suy ra ∠DAB = ∠BED = ∠AFD = ∠DCA.
Chứng minh tương tự ta cũng có ∠DBA = ∠DAC. Như vậy 4DAB ∼ 4DCA. Hơn nữa ta
cũng có
∠BDC = ∠BAC + ∠DBA + ∠DCA = ∠BAC + ∠DAC + ∠DAB = 2∠BAC = ∠BOC.
Do đó, tứ giác BDOC nội tiếp, dẫn tới D luôn thuộc đường tròn (BOC) cố định.
(c) Dựng đường kính OT của đường tròn (BOC). Khi đó ta có T là điểm cố định. Mặt khác, do
∠ODT = ∠ADO = 90◦ nên ba điểm A, D, T thẳng hàng. Như vậy đường thẳng AD luôn đi
qua điểm T cố định.
Chú ý rằng T chính là giao điểm hai tiếp tuyến tại B,C của đường tròn (O). Từ đó ta
thu được hai cặp tam giác đồng dạng 4T GB ∼ 4T BA và 4T GC ∼ 4TCA. Như vậy,
GB TG TG GC
AB = T B = TC = AC , vì vậy GB · AC = GC · AB.
CÂU LẠC BỘ TOÁN A1, HOTLINE: 034 761 1986 - 035 290 3286 101

Q F

D O
J
E I

H M
B C

1
X

A
G
K

T á n
To

(d) Bổ đề. Cho tam giác ABC với đường cao AH (H ∈ BC) và tâm ngoại tiếp O. Khi đó,
∠HAB = ∠OAC.
cb

Cách 1. Xét tam giác AEF có AD là đường cao và K là tâm ngoại tiếp. Khi đó ta có
∠DAE = ∠KAF (theo bổ đề), lại chú ý rằng ∠EAB = ∠FAC, ta thu được ∠DAB = ∠KAC
lạ

(1).

Mặt khác, gọi M là trung điểm của BC. Khi đó ta có OA2 = OB2 = OC2 = OM · OT , dẫn
u

đến 4OAM ∼ 4OTA. Gọi H là hình chiếu của A trên BC. Từ đó ta thu được ∠OAM =

∠OTA = ∠HAT . Kết hợp với ∠HAB = ∠OAC (theo bổ đề) suy ra ∠TAB = ∠MAC (2).

Từ (1) và (2) suy ra ∠KAC = ∠MAC, dẫn tới A, K, M thẳng hàng. Vậy đường thẳng AK
luôn đi qua điểm M cố định.

Cách 2. Gọi Q, R tương ứng là trung điểm của AE, AF. KQ, KR tương ứng cắt AB, AC tại
các điểm I, J. Khi đó ta có AIKJ là hình bình hành và AK đi qua trung điểm của IJ. Trong
khi đó, từ 4AQI ∼ 4ARJ ta được AJ AI
= AQ AE AB
AR = AF = AC . Do đó, IJ k BC. Kết hợp với việc
AK đi qua trung điểm của IJ, ta suy ra AK luôn đi qua trung điểm M cố định của BC.
102 TUYỂN TẬP ĐỀ THI VÀO LỚP 10 CHUYÊN TOÁN

Câu 4
Cho số tự nhiên a = 313 .57 .720 .

(a) Gọi A là tập hợp các số nguyên dương k sao cho k là ước của a và k chia hết cho 105.
Hỏi tập A có bao nhiêu phần tử?

(b) Giả sử B là một tập con bất kỳ của A có 9 phần tử. Chứng minh ta luôn có thể tìm
được 2 phần tử của B sao cho tích của chúng là số chính phương.

Lời giải. (a) Ta có 105 = 3.5.7. Do đó ta có thể viết lại tập A dưới dạng sau:

A = {3x .5y .7z , 1 ≤ x ≤ 13, 1 ≤ y ≤ 7, 1 ≤ z ≤ 20}.

1
Do đó số phần tử của tập A là số các bộ (x, y, z) thỏa mãn điều kiện trên, mà theo quy tắc

A
nhân thì chính là: 13.7.20 = 1820.
Kết luận: |A| = 1820.

n
(b) 9 phần tử của B có dạng 3xi .5yi .7zi , với 1 ≤ i ≤ 9. Trước hết, trong 9 số x1 , . . . , x9 , theo
á
Dirichlet tổn tại ít nhất 5 số có cùng tính chẵn lẻ, không mất tính tổng quát giả sử đó là
x1 , . . . , x5 .
To
Tiếp theo, trong 5 số y1 , . . . , y5 tồn tại ít nhất 3 số có cùng tính chẵn lẻ, không mất tính tổng
quát giả sử đó là y1 , y2 , y3 .

Cuối cùng, trong 3 số z1 , z2 , z3 tồn tại ít nhất 2 số có cùng tính chẵn lẻ, giả sử đó là z1 , z2
cb

Khi đó x1 + x2 , y1 + y2 , z1 + z2 là các số chẵn, nên tích

(3x1 .5y1 .7z1 ) . (3x2 .5y2 .7z2 )


lạ

là số chính phương.
u

Câu 5

Cho hệ phương trình với k là tham số:


 r r
x x x
√ + + =k





 yz y z





 y r r
y y
√ + + =k

 zx z x




 r r

 z z z
√ + + =k


xy x y

(a) Giải hệ với k = 1.

(b) Chứng minh hệ vô nghiệm với k ≥ 2 và k 6= 3.


CÂU LẠC BỘ TOÁN A1, HOTLINE: 034 761 1986 - 035 290 3286 103

Lời giải. (a) Điều kiện của nghiệm là x, y, z là các số cùng dương hoặc cùng âm.

• Trường hợp 1: x, y, z cùng dương. Hệ phương trình tương đương với


r  r 
x x
+1 +1 = 2





 y z





r  r 
y y
+1 +1 = 2

 x z




 r  r 

 z z

 +1 +1 = 2
y x

1
Nhân cả hai vế của ba phương trình lại với nhau ta được

A
r  r 
x y
∏ +1 + 1 = 8.

n
cyc y x

á
To
Tuy nhiên
r  r  r r
x y x y
+1 +1 = 2+ + >2
y x y x

và tương tự với các tích còn lại. Khi đó


cb

r  r 
x y
∏ +1 + 1 > 2.2.2 = 8.
cyc y x
lạ

Suy ra hệ vô nghiệm trong trường hợp này.


• Trường hợp 2: x, y, z cùng âm. Hệ phương trình tương đương với
u

r  r 
x x
−1 −1 = 0





 y z





r  r 
y y
−1 −1 = 0

 x z




 r  r 

 z z

 −1 −1 = 0
y x

Khi đó x = y = z.

Vậy các nghiêm của hệ phương trình là x = y = z = a với a < 0.

(b) Điều kiện của nghiệm là x, y, z là các số cùng dương hoặc cùng âm.
104 TUYỂN TẬP ĐỀ THI VÀO LỚP 10 CHUYÊN TOÁN

• Trường hợp 1: x, y, z cùng dương. Hệ phương trình tương đương với


r  r 
x x
+1 +1 = k+1





 y z





r  r 
y y
+1 +1 = k+1

 x z




r  r 

 z z

 +1 +1 = k+1
y x

x y x y
Không mất tính tổng quát giả sử x = min(x, y, z). Ta có ≤ và ≤ . Dấu bằng xảy
y x z z

1
ra khi và chỉ khi x = y. Khi đó

A
r  r  r  r 
x x y y
k+1 = +1 +1 ≤ +1 + 1 = k + 1.
y z x z

n
Dấu bằng ở giữa buộc phải xảy ra, dẫn đến x = y. Tương tự ta cũng có x = z, dẫn đến
á
x = y = z. Khi đó thay vào phương trình thứ nhất của hệ ta thu được k = 3, loại do
To
k 6= 3. Vậy hệ vô nghiệm trong trường hợp này.
• Trường hợp 2: x, y, z cùng âm. Hệ phương trình tương đương với
r  r 

x x
−1 −1 = 1−k



y z
cb









r  r 
y y
−1 −1 = 1−k
x z
lạ







 r  r 

 z z

 −1 −1 = 1−k
u

y x

Không mất tính tổng quát, giả sử x = min(x, y, z). Khi đó −x ≥ −y và −x ≥ −z, khi
đó r  r 
x x
−1 − 1 ≥ 0.
y z
Tuy nhiên 1 − k < 0, cho nên hệ phương trình vô nghiệm trong trường hợp này.

Vậy hệ phương trình vô nghiệm với k ≥ 2, k 6= 3.


CÂU LẠC BỘ TOÁN A1, HOTLINE: 034 761 1986 - 035 290 3286 105

2.14. ĐỀ THI TUYỂN SINH VÀO LỚP 10 CHUYÊN TOÁN THÀNH


PHỐ H Ồ C HÍ M INH , NĂM 2020

Câu 1
a b c
Cho ba số dương a, b, c thỏa mãn điều kiện + + = 2020.
 2 b + c c +a a + b
a b2 c2
Tính giá trị của biểu thức P = + + : (a + b + c).
b+c c+a a+b

Lời giải. Ta có

1
 
a b c
+ + (a + b + c) = 2020(a + b + c)

A
b+c c+a a+b
a2 b2 c2
=⇒ +a+ +b+ + c = 2020(a + b + c)
b+c c+a a+b

n
a2 b2 c2
=⇒ + + = 2019(a + b + c).
b+c c+a a+b á
To
Từ đó ta suy ra P = 2019.

Câu 2
√ √

a) Giải phương trình: 2x2 + x + 9 + 2x2 − x + 1 = x + 4.


cb

(
y2 − 2xy = 8x2 − 6x + 1
b) Giải hệ phương trình 2
y = x3 + 8x2 − x + 1.
lạ

Lời giải.
√ √
u

a) Đặt a = 2x2 + x + 9 và b = 2x2 − x + 1. Khi đó a2 − b2 = 2(x + 4). Phương trình trở


thành
1 1
a + b = (a2 − b2 ) = (a − b)(a + b)
2 2
hay
(a + b)(a − b − 2) = 0.
Lại có a > 0, b >√0 nên a + b > 0, dẫn đến a − b = 2. Kết hợp với a + b = x + 4 dẫn đến
2a = x + 6, hay
 2  2x2 + x + 9 = x + 6. Bình phương hai vế và giải phương trình bậc hai, ta
8
thu được x ∈ 0, . Thử lại vào phương trình ban đầu thì cả hai nghiệm đều thỏa mãn. Vậy
7  
8
phương trình có hai nghiệm 0, .
7
b) (
y2 − 2xy = 8x2 − 6x + 1, (1)
y2 = x3 + 8x2 − x + 1. (2)
106 TUYỂN TẬP ĐỀ THI VÀO LỚP 10 CHUYÊN TOÁN

Phương trình (1) tương đương với (3x − 1)2 = (x − y)2 .

• Trường hợp 1: x − y = 3x − 1. Suy ra y = 1 − 2x. Thay vào phương trình (2) ta có

(1 − 2x)2 = x3 + 8x2 − x + 1.

Giải phương trình trên ta thu được x ∈ {−3, −1, 0}. Kết hợp với y = 1 − 2x ta thu được
ba bộ nghiệm (x, y) = (−3, 7), (−1, 3), (0, 1).
• Trường hợp 2: x − y = 1 − 3x. Suy ra y = 4x − 1. Thay vào phương trình (2) ta có

(4x − 1)2 = x3 + 8x2 − x + 1.

Giải phương trình trên ta thu được x ∈ {0, 1, 7}. Kết hợp với y = 4x − 1 ta thu được ba

1
bộ nghiệm (x, y) = (0, −1), (1, 3), (7, 27).

A
Vậy hệ phương trình có 6 nghiệm (x, y) = (−3, 7), (−1, 3), (0, 1), (0, −1), (1, 3), (7, 27).

n
Câu 3
á
Cho tam giác nhọn ABC (AB < BC < CA) nội tiếp đường tròn (O). Từ A kẻ đường thẳng
To
song song với BC cắt (O) tại A1 . Từ B kẻ đường thẳng song song với AC cắt (O) tại B1 .
Từ C kẻ đường thẳng song song với AB cắt (O) tại C1 . Chứng minh rằng các đường thẳng
qua A1 , B1 ,C1 lần lượt vuông góc với BC,CA, AB đồng quy.

cb

A A1
lạ
u

K

O
H

B C

Lời giải. Gọi H là trực tâm của tam giác ABC và K là điểm đối xứng với H qua O. Dựng đường
kính AX của đường tròn (O). Khi đó, ta có ∠AA1 X = 90◦ , hay A1 X ⊥ AA1 , kết hợp AA1 k BC
dẫn đến A1 X ⊥ BC (1). Mặt khác, ta cũng có AHXK là hình bình hành nên XK k AH, kết hợp
với AH ⊥ BC dẫn đến XK ⊥ BC (2). Từ (1) và (2) suy ra A1 , X, K thẳng hàng và A1 K ⊥ BC.
Chứng minh tương tự B1 K ⊥ CA,C1 K ⊥ AB. Như vậy, các đường thẳng qua A1 , B1 ,C1 lần lượt
vuông góc với BC,CA, AB đồng quy tại K.
CÂU LẠC BỘ TOÁN A1, HOTLINE: 034 761 1986 - 035 290 3286 107

Câu 4
a2 + b2 (a − b)2
a) Cho 2 số thực a, b. Chứng minh rằng: ≥ ab + 2 .
2 a + b2 + 2
b) Cho 2 số dương a, b thỏa mãn điều kiện a + b ≤ 3.
20 7
Tìm giá trị nhỏ nhất của biểu thức: Q = b − a + + .
a b

Lời giải. a) Ta có

a2 + b2 (a − b)2 (a − b)2 (a − b)2 (a − b)2 (a2 + b2 )


− ab − 2 = − = ≥0

1
2 a + b2 + 2 2 a2 + b2 + 2 2(a2 + b2 + 2)

A
a2 + b2 (a − b)2
cho nên ≥ ab + 2 .
2 a + b2 + 2

n
b) Ta phân tích Q như sau:
á
To
   
20 7
Q = 5a + + 7b + − 6(a + b).
a b

Áp dụng bất đẳng thức AM-GM ta có



cb

r
20 20
5a + ≥ 2 5a · = 20,
a a
lạ

r
7 7
7b + ≥ 2 7b · = 14.
b b
u

Dấu bằng xảy ra khi và chỉ khi a = 2, b = 1. Kết hợp với a + b ≤ 3 ta suy ra

Q ≥ 20 + 14 − 6 · 3 = 16.

Dấu bằng xảy ra khi và chỉ khi a = 2, b = 1.

Câu 5
Đường tròn (I) nội tiếp tam giác ABC tiếp xúc với các cạnh AB, BC,CA lần lượt tại
D, E, F. Kẻ đường kính EJ của đường tròn (I). Gọi d là đường thẳng qua A song song với
BC. Đường thẳng JD cắt d, BC lần lượt tại L, H.

a) Chứng minh rằng E, F, L thẳng hàng.

b) JA, JF cắt BC lần lượt tại M, K. Chứng minh rằng MH = MK.


108 TUYỂN TẬP ĐỀ THI VÀO LỚP 10 CHUYÊN TOÁN

Lời giải. a) Do AL k BC nên ∠ABC = 180◦ − ∠DAL. Từ EJ là đường kính của (I) dẫn đến
∠EDJ = 90◦ . Kết hợp với BI ⊥ DE ta suy ra DJ k BI. Như vậy ∠ADL = ∠ABI = 12 ∠ABC.
Từ đó suy ra ∠ALD = 180◦ − ∠DAL − ∠ADL = ∠ABC − 21 ∠ABC = 12 ∠ABC = ∠ADL và
tam giác ADL cân tại A. Vì thế AL = AD = AF và do đó ∠AFL = 90◦ − 21 ∠LAC = 90◦ −
1 ◦ ◦
2 ∠FCE = ∠CFE = 180 − ∠AFE, dẫn tới ∠AFL + ∠AFE = 180 . Như vậy, E, F, L thẳng
hàng.

b) Gọi N là giao điểm của JF và đường thẳng d. Chứng minh tương tự ở câu a) ta được AN =
AF = AD = AL. Như vậy, A là trung điểm NL. Trong khi đó, sử dụng định lý Thales ta được
MH JM MK
AL = JA = AN . Từ đó suy ra MH = MK.

N A L

1
d

A
J

n
F
D
I á
To
H B E M C K

cb

Câu 6
Tìm tất cả các số nguyên dương x, y thỏa mãn phương trình 3x − y3 = 1.
lạ

Lời giải. Chuyển vế ta có


u

3x = y3 + 1 = (y + 1)(y2 − y + 1).

Suy ra (
y + 1 = 3a ,
y2 − y + 1 = 3b
với a, b ∈ N.
Đặt d = (y + 1, y2 − y + 1). Ta có d | y2 − y + 1 = (y + 1)(y − 2) + 3, và d | y + 1 dẫn đến d | 3.
Suy ra trong hai số tự nhiên a, b tồn tại ít nhất một số không vượt quá 1 (nếu cả a và b đều lớn
hơn 1, y + 1 và y2 − y + 1 cùng chia hết cho 9, dẫn đến d chia hết cho 9, trái với d | 3).

• a = 0. Khi đó y + 1 = 1, dẫn đến y = 0, loại do y nguyên dương.

• a = 1. Khi đó y + 1 = 3, dẫn đến y = 2. Suy ra 3x = 23 + 1 = 9, dẫn đến x = 2. Ta có


nghiệm (x, y) = (2, 2).

• b = 0. Khi đó y2 − y = 0, dẫn đến y = 1. Suy ra 3x = 13 + 1 = 2, loại.


CÂU LẠC BỘ TOÁN A1, HOTLINE: 034 761 1986 - 035 290 3286 109

• b = 1. Khi đó y2 − y = 3, dẫn đến y = 2. Ta lại có bộ nghiệm (x, y) = (2, 2).

Vậy phương trình có nghiệm (x, y) = (2, 2).

1
A
á n
To

cb
lạ
u

110 TUYỂN TẬP ĐỀ THI VÀO LỚP 10 CHUYÊN TOÁN

2.15. ĐỀ THI TUYỂN SINH VÀO LỚP 10 TRƯỜNG THPT CHUYÊN


KHOA HỌC TỰ NHIÊN NĂM 2021, VÒNG 1
Câu 1
Giải phương trình
√ √ p
13 5 − x + 18 x + 8 = 61 + x + 3 (5 − x)(x + 8)

√ √
Lời giải. Điều kiện xác định: −8 ≤ x ≤ 5. Đặt a = 5 − x, b = x + 8 = b thì a, b ≥ 0 và
a2 + 2b2 = 5 − x + 2x + 16 = 21 + x
Phương trình trở thành

1
13a + 18b = 40 + a2 + 2b2 + 3ab.

A
hay tương đương
(a + b − 5)(a + 2b − 8) = 0.

n
Từ đây, ta có hai trường hợp
• Trường hợp 1: a + b = 5 và a2 + b2 = 13
á
Rút a = 5 − b và thay vào phương trình thứ hai ta được (5 − b)2 + b2 = 13 hay
To
b2 − 5b + 6 = 0.
Giải ta tìm được b = 2 và b = 3. Tương ứng thì x = −4 và x = 1.
• Trường hợp 2: a + 2b = 8 và a2 + b2 = 13.

Tương tự, ta cũng có(8 − 2b)2 + b2 = 13 hay tương đương


cb

5b2 − 32b + 51 = 0.
17 89
Giải ta tìm được b = 3 và b = . Tương ứng x = 1 và x = .
lạ

5 25
89
Thử lại các nghiệm đều thỏa mãn. Vậy phương trình có các nghiệm là −4, 1, .
25
u

Câu 2
Giải hệ phương trình
x4 + y4 + 6x2 y2 = 1


x(x + y)4 = x − y

Lời giải. Ta sẽ trình bày hai cách giải.


Cách 1. Nhân chéo 2 vế của hệ phương trình ta thu được
x(x + y)4 = (x − y)(x4 + y4 + 6x2 y2 ).
Khai triển và rút gọn ta được

y(5x2 + 10x2 y2 + y4 ) = 0.
Từ đây ta có hai trường hợp
CÂU LẠC BỘ TOÁN A1, HOTLINE: 034 761 1986 - 035 290 3286 111

• Với y = 0 thì từ phương trình đầu tiên suy ra x4 = 1 hay x = 1 hoặc x = −1. Các nghiệm
này đều thoả mãn hệ.

• Với 5x4 + 10x2 y2 + y4 = 0 ⇒ x = y = 0 (do x2 > 0, y2 > 0).


Ta loại nghiệm này do khi đó x4 + y4 + 6x2 y2 = 0.

Vậy hệ phương trình có các nghiệm (x, y) là (−1, 0); (1, 0).
Cách 2. Nếu x = 0 thì từ phương trình thứ hai suy ra y = 0 nhưng khi đó không thoả mãn phương
trình đầu tiên nên x 6= 0. Từ phương trình thứ hai suy ra

x−y
x4 + y4 + 6x2 y2 + 4xy(x2 + y2 ) = .
x

1
Trừ phương trình trên cho phương trình đầu tiên, ta được

A
y
4xy(x2 + y2 ) = −
x

n
hay tương đương y(4x2 (x2 + y2 ) + 1) = 0. Từ đây ta tìm được y = 0 và x = ±1. Vậy hệ đã cho
có hai nghiệm (x, y) là (−1, 0); (1, 0). á
To
Câu 3
Tìm số nguyên dương n nhỏ nhất biết rằng khi chia n cho 7, 9, 11, 13 ta nhận được các số

dư là 3, 4, 5, 6.
cb

Lời giải. Vì n chia cho 7,9,11,13 được số dư tương ứng là 3,4,5,6. Nên ta suy ra 2n chia cho
lạ

7,9,11,13 được số dư tương ứng là : 6,8,10,12. Suy ra 2n + 1 chia hết cho cả 7,9,11 và 13. Chú
ý là 7,9,11 và 13 đôi một nguyên tố cùng nhau nên số nguyên dương nhỏ nhất chia hết cho cả
7,9,11,13 là : 7.9.11.13= 9009 nên 2n + 1 = 9009 kéo theo
u

9009 − 1
n= = 4504.
2

Câu 4
Cho tam giác nhọn ABC có điểm P nằm trong tam giác (P không nằm trên các cạnh). Gọi
J, K, L lần lượt là tâm đường tròn nội tiếp của các tam giác PBC, PCA, PAB.
d + CKA
1. Chứng minh rằng BJC d = 90◦ .
d + ALB

2. Giả sử PB = PC và PC < PA. Gọi X,Y, Z lần lượt là hình chiếu vuông góc của
J, K, L trên các cạnh BC,CA, AB. Dựng hình bình hành XYW Z. Chứng minh rằng
W nằm trên phân giác BAC.
d
112 TUYỂN TẬP ĐỀ THI VÀO LỚP 10 CHUYÊN TOÁN

A A

E
W
F

L K Y
Z
P L K
J P
J
B C B C

1
X

A
Lời giải.
d = 90◦ + 1 BPC,
1. Ta có BJC d CKAd = 90◦ + 1 CPA d = 90◦ + 1 APB.
d và ALB d Từ đó ta được
2 2 2

n
d + CKA
BJC d = 270◦ + (BPC
d + ALB 1 d + CPA d = 270◦ + · 360◦ = 450◦ .
d + APB) 1
2 2

á
2. Lấy các điểm E, F lần lượt đối xứng với C, B qua Y, Z. Chú ý rằng ta có BZ = BA+BP−PA
2
To
và CY = CA+CP−PA
2 , vì thế BF = BA + BP − PA và CE = CA +CP − PA. Như vậy, AF =
BA − BF = PA − PB và AE = CA −CE = PA − PC, để ý rằng PB = PC nên ta thu được
AF = AE. Gọi W1 là trung điểm của EF. Do 4PBC cân tại P nên X là trung điểm của

BC. Từ đó ta được XY,YW1 ,W1 Z và ZX lần lượt là đường trung bình của 4CBE, 4ECF,
4FBE và 4BFC. Do đó, XY k ZW1 (cùng k BE) và XZ k YW1 (cùng k CF), vì thế XYW1 Z
cb

là hình bình hành và do vậy W1 ≡ W . Mặt khác, 4AEF cân tại A và có W ≡ W1 là trung
điểm của EF, như vậy AW chính là đường phân giác BAC.d Ta có điều phải chứng minh.
lạ

Câu 5
u

Cho tập A = {1, 2, ..., 2021}. Tìm số nguyên dương k lớn nhất (k ≥ 2) sao cho ta có thể
chọn được k số phân biệt từ tập A mà tổng của hai số phân biệt bất kỳ trong k số được

chọn không chia hết cho hiệu của chúng.

Lời giải. Ta chứng minh giá trị lớn nhất của k là 674. Trong k số được chọn thì không thể
chứa cả hai cặp {x, x + 1} và {x, x + 2}. Do đó nếu x1 < x2 < · · · < xk là k số được chọn thì
xi+1 − xi ≥ 3 với mọi i = 1, 2, ..., k − 1. Vì x1 ≥ 1 nên từ đây ta có x2 ≥ 4, x3 ≥ 7, ...., xk ≥ 3k − 2.
Vì xk ≤ 2021 nên k ≤ 674. Ví dụ, ta có thể chọn 674 số sau đây 1, 4, 7, ..., 2020 thì tổng hai số
bất kỳ sẽ chia 3 dư 2, còn hiệu hai số thì chia hết cho 3 vì tất cả các số được chọn đều chia 3 dư
1.
CÂU LẠC BỘ TOÁN A1, HOTLINE: 034 761 1986 - 035 290 3286 113

2.16. ĐỀ THI TUYỂN SINH VÀO LỚP 10 TRƯỜNG THPT CHUYÊN


KHOA HỌC TỰ NHIÊN NĂM 2021, VÒNG 2
Câu 1
1. Với a, b, c là các số thực thoả mãn a + b + c 6= 0 và (a + b)(b + c)(c + a) = 1, chứng
minh rằng

a b 1 + abc + ab(a + b + c)
+ = .
a2 (a + b + c) + 1 + abc b2 (a + b + c) + 1 + abc (a + b + c)2

2. Giải hệ phương trình

1
(
x2 + 4y2 + 4xy + 2x2 y2 = 11

A
.
3xy(x + 2y) + 31 = 9x + 18y + 13xy.

n
Lời giải.
1. Ta sử dụng đẳng thức quan trọng sau á
To
(a + b)(b + c)(c + a) + abc = (a + b + c)(ab + bc + ca)
= ab(a + b) + bc(b + c) + ca(c + a) + 3abc

Vì (a + b)(b + c)(c + a) = 1 nên ta suy ra 1 + abc = (a + b + c)(ab + bc + ca). Do đó,


cb

a2 (a + b + c) + 1 + abc = (a + b + c)(a2 + ab + bc + ca) = (a + b + c)(a + b)(a + c).

Tương tự
lạ

b2 (a + b + c) + 1 + abc = (a + b + c)(b + a)(b + c).


Suy ra vế trái của đẳng thức cần chứng minh bằng
u

a b a(b + c) + b(c + a)
+ = .

(a + b + c)(a + b)(c + a) (a + b + c).(a + b)(b + c) (a + b + c)(a + b)(b + c)(c + a)


ab + ab + bc + ca ab(a + b + c) + (ab + bc + ca)(a + b + c) 1 + abc + ab(a + b + c)
= = = .
a+b+c (a + b + c)2 (a + b + c)2
Ta có điều phải chứng minh.
2. Đặt a = x + 2y, b = xy thì hệ đã cho có thể viết lại thành
(
a2 + 2b2 = 11
.
3ab + 31 = 9a + 13b.

Cộng hai phương trình và chuyển vế, phân tích nhân tử ta có

(a + b − 4)(a + 2b − 5) = 0.

Từ đây ta có hai trường hợp


114 TUYỂN TẬP ĐỀ THI VÀO LỚP 10 CHUYÊN TOÁN

• Nếu a + b = 4 thì a = 4 − b và thay vào phương trình a2 + 2b2 = 11 ta được

3b2 − 8b + 5 = 0.
5 7
Giải tìm được b = 1, a = 3 và b = , a = .
3 3  
1
+) Nếu a = 3, b = 1 thì x + 2y = 3, x · 2y = 2 tìm được (x, y) là (1, 1), 2, .
2
7 5 7 10
+) Nếu a = , b = thì x + 2y = , x · 2y = không tồn tại x, y.
3 3 3 3
• Nếu a + 2b = 5 thì a = 5 − 2b và thay vào phương trình a2 + 2b2 = 11 ta được

3b2 − 10b + 7 = 0.

1
7 1

A
Giải tìm được b = 1, a = 3 và b = , a = .
3 3  
1
+) Nếu a = 3, b = 1 thì x+2y = 3, x·2y = 2 tương tự như trên thì (x, y) là (1, 1), 2, .

n
2
7 1 1 14
+) b = , a = thì x + 2y = , x · 2y =
3 3 3 3 á
thì không tồn tại x, y.
To
 
1
Vậy hệ đã cho có 2 nghiệm (1, 1), 2, .
2

cb

Câu 2
1. Tìm x, y nguyên dương thoả mãn 3x + 29 = 2y .
lạ

2. Với a, b, c là các số thực dương thoả mãn điều kiện 2(a + b + c) + ab + bc + ca = 9,


tìm giá trị lớn nhất của biểu thức
u

a+1 b+1 c+1


M= + + .
a2 + 10a + 21 b2 + 10b + 21 c2 + 10c + 21

Lời giải.
1. Với x = 1 thì y = 5 và ta có nghiệm là (1; 5). Với x > 1 thì y > 5 xét theo mod9 vì
x > 1, ta suy ra 2 ≡ 2y (mod 9) nên 1 ≡ 2y−1 (mod 9). Mặt khác 26 ≡ 1 (mod 9) nên
y = 6n + 1, n ≥ 1.
Xét theo mod 32 vì y > 5 ta suy ra 3x − 3 ≡ 0 (mod 32) kéo theo 3x ≡ 3 (mod 32) dẫn
tới 3x−1 ≡ 1 (mod 32). Lại có 38 ≡ 1 (mod 32), nên x = 8m + 1, m ≥ 1.
Cuối cùng xét theo mod 7 cho ta 38m+1 + 1 ≡ 26n+1 (mod 7). Lại có 26 ≡ 1 (mod 7)
nên 26n+1 ≡ 2 (mod 7). Suy ra 38m+1 ≡ 1 (mod 7), điều này là mâu thuẫn với sự kiện
36 ≡ 1 (mod 7), và 8m + 1 không thể là bội của 6. Suy ra phương trình không có nghiệm
x > 1, y > 5.
Vậy phương trình đã cho có nghiệm duy nhất là x = 1, y = 5.
CÂU LẠC BỘ TOÁN A1, HOTLINE: 034 761 1986 - 035 290 3286 115

x
2. Đặt x = a + 1, y = b + 1, z = c + 1 thì xy + yz + zx = 12 và M = ∑ . Áp dụng
x2 + 8x + 12
bất đẳng thức AM-GM, ta có x2 + 4 ≥ 4x, y2 + 4 ≥ 4y, z2 + 4 ≥ 4z. Suy ra
x y z
M≤ + + .
12x + 8 12y + 8 12z + 8
3
Ta sẽ chứng minh vế trái của bất đẳng thức trên không vượt quá . Quy đồng và biến
16
đổi tương đương, ta cần chứng minh

20(x + y + z) + 96 ≥ 27xyz.
p
Theo bất đẳng thức AM-GM thì xy + yz + zx ≥ 3 3 (xyz)2 dẫn tới 27xyz ≤ 216, trong khi
đó vì (x + y + z)2 ≥ 3(xy + yz + zx) = 36 nên 20(x + y + z) + 96 ≥ 216 ≥ 27xyz. Dấu bằng

1
3
xảy ra khi và chỉ khi x = y = z = 2 hay a = b = c = 1 và giá trị lớn nhất của M là .
16

A
Câu 3

n
Cho hình thoi ABCD (BADd < 90◦ ) có đường tròn nội tiếp (O). Các điểm M, N lần lượt
á
thuộc các cạnh CB,CD sao cho MN tiếp xúc (O) tại P, và tam giác CMN nhọn không
To
cân. Đường thẳng MN lần lượt cắt các đường thẳng AB, AD tại E, F. Gọi K, L theo thứ tự
là trực tâm của các tam giác BME và DNF.

1. Chứng minh rằng OP đi qua trung điểm I của KL.



cb

2. Gọi H là trực tâm của tam giác CMN. Chứng minh rằng
OI EF 1
− =− .
CH 2MN 2
lạ

3. Gọi giao điểm của EK, FL với BD lần lượt là S, T . Gọi giao điểm của NS và MT
là Q. Đường tròn nội tiếp của tam giác CMN tiếp xúc với MN tại G. Chứng minh
u

rằng hai đường thẳng PQ và GH song song.


Lời giải.
1. Do K, L tương ứng là trực tâm của 4BME và 4DNF nên BK ⊥ ME và DL ⊥ NF. Hơn
nữa, ta có OP ⊥ MN. Như vậy, ba đường thẳng BK, DL và OP đôi một song song với
nhau. Vì O là trung điểm của BD nên theo tính chất của đường trung bình của hình thang
ta suy ra OP đi qua trung điểm I của KL.
2. Do BE k CN và MH ⊥ CN (do H là trực tâm của 4CMN) nên ta thu được MH ⊥ BE.
Trong khi đó, lại chú ý rằng MK ⊥ BE do K là trực tâm của 4BME, từ đó ta suy ra
M, H, K thẳng hàng. Chứng minh tương tự ta cũng được N, H, L thẳng hàng. Mặt khác, vì
OI là đường trung bình của hình thang BDLK nên áp dụng định lý Thales ta có
2OI BK DL BM DN ME NF ME + NF
= + = + = + = .
CH CH CH CM CN MN MN MN
116 TUYỂN TẬP ĐỀ THI VÀO LỚP 10 CHUYÊN TOÁN

B O D

I
E K
L
P
M

1
H N

A
F

Do đó,
á n
To
2OI EF ME + NF − EF ME + NF − (ME + MN + NF) −MN
− = = = = −1.
CH MN MN MN MN
Vì vậy,

 
OI EF 1 2OI EF 1
− = − =− .
cb

CH 2MN 2 CH MN 2

A
lạ
u

X
B S T O D

E Q
K
P L
M G

J N
H
F
C

3. Gọi X là giao điểm của ES và FT . Khi đó ta có X chính là trực tâm của 4AEF.
CÂU LẠC BỘ TOÁN A1, HOTLINE: 034 761 1986 - 035 290 3286 117

Ta sẽ chứng minh XP k GH. Thật vậy, vì AE k CN và AF k CM nên 4AEF ∼ 4CNM


(g.g). Khi đó, chú ý rằng P, G tương ứng là tiếp điểm của đường tròn nội tiếp 4AEF và
PE GN
4CNM với các cạnh EF và NM, vì thế ta suy ra được PF = GM . Ngoài ra, do X
[ EF =
H
\ NM (XE k HN) và X [FE = H \ MN (XF k HM) nên 4XEF ∼ 4HNM (g.g), kết hợp với
PE GN
PF = GM ta chứng minh được 4XPF ∼ 4HGM (c.g.c). Do vậy, X PF = HGM và vì thế
[ \
XP k GH.
Như vậy, bây giờ ta chỉ cần chứng minh X, P, Q thẳng hàng. Gọi Q1 là giao điểm của
NS và XP. Áp dụng định lý Menelaus cho 4XPE với N, S, Q1 thẳng hàng ta được
Q1 X NP SE Q1 X NE SX
· · = 1 =⇒ = · .
Q1 P NE SX Q1 P NP SE
Gọi Q2 là giao điểm của MT và XP. Chứng minh tương tự như trên ta cũng được

1
Q2 X MF T X
= · .

A
Q2 P MP T F
d = BSE
Ta có XST d − AEX
d = ABD d − AFX
d = ADB [ = DT
[ dS và vì thế 4XST cân tại
F = XT
X và SX = T X.

n
Ngoài ra ta có 4BSE ∼ 4DT F (g.g) nên
SE BE BE DA ME NE ME NE á
To
= = · = · = · .
TF DF BA DF MF NF NF MF
Mặt khác, (O) cũng chính là đường tròn bàng tiếp ứng với đỉnh C của 4CNM nên theo
kết quả quen thuộc ta được MP = NG và NP = MG. Do đó,

MP GN PE PE − MP ME
= = = = .
cb

NP GM PF PF − NP NF
Vì thế ta được
SE ME NE MP NE NE 1 MF 1
= · = · =⇒ · = · .
lạ

TF NF MF NP MF NP SE MP T F
Do đó,
Q1 X NE SX MF T X Q2 X
u

= · = · = .
Q1 P NP SE MP T F Q2 P

Từ đó ta suy ra Q1 ≡ Q2 = NS ∩ MT và vì thế Q ≡ Q1 ≡ Q2 ∈ XP. Vậy X, P, Q thẳng hàng.


Kết hợp với XP k GH ta được PQ k GH.

Câu 4
Giả sử a1 , a2 , ..., a2021 là các số thực thoả mãn
a1 a2 a2021
2
+ 2
+···+ 2
= 0.
a1 + 1 a2 + 1 a2021 + 1

Chứng minh rằng tồn tại số nguyên k (1 ≤ k ≤ 2021) sao cho

a1 2a2 kak 2k + 1
2
+ 2
+···+ 2
≤ .
a1 + 1 a2 + 1 ak + 1 8
118 TUYỂN TẬP ĐỀ THI VÀO LỚP 10 CHUYÊN TOÁN

2ai
Lời giải. Đặt xi = thì |xi | ≤ 1 với mọi i = 1, 2, ..., 2021 và
1 + a2i

x1 + x2 + · · · + x2021 = 0.
k
sk − sk−1
Lại đặt s0 = 0, s1 = x1 và sk = ∑ ixi với k = 1, 2, ..., 2021. Ta có xk = với k =
i=1 k
1, 2, ..., 2021 nên ta có
1 1 1 1
s1 + s2 + · · · + s2020 + s2021 = 0.
2 6 2020.2021 2021
Như vậy tồn tại k ∈ {1, 2, ..., 2021} để sk−1 sk ≤ 0. Lúc này nếu điều cần chứng minh không
đúng thì có mâu thuẫn là

1
2k + 1 2(k − 1) + 1

A
|kxk | = |sk − sk−1 | = |sk | + |sk−1 | > + ≥ k.
4 4

á n
To

cb
lạ
u

CÂU LẠC BỘ TOÁN A1, HOTLINE: 034 761 1986 - 035 290 3286 119

2.17. ĐỀ THI TUYỂN SINH VÀO LỚP 10 TRƯỜNG THPT CHUYÊN


SƯ PHẠM HÀ NỘI NĂM 2021, VÒNG 2
Câu 1

1+ 5
Cho α = .
2
a) Tìm một đa thức bậc hai Q(x) với hệ số nguyên sao cho α là nghiệm của Q(x).

b) Cho đa thức P(x) = x5 − x4 − x + 1. Tính giá trị của P(α).

1
Lời giải.

A
1. Từ đề bài suy ra 2α − 1 = 5. Bình phương hai vế ta được

4α 2 − 4α − 4 = 0

hay tương đương


á n
To
α 2 − α − 1 = 0.
Từ đó chọn Q(x) = x2 − x − 1 là đa thức thoả mãn đề bài.

2. Ta viết
cb

P(x) = (x5 −x4 −x3 )+(x3 −x2 −x)+(x2 −x−1)+x+2 = x3 .Q(x)+x.Q(x)+Q(x)+x+2.



5+ 5
Vì α là nghiệm của Q(x) nên P(α) = α + 2 = .
lạ

2
u

Câu 2

Cho A, B là hai điểm cố định nằm trên đường tròn tâm O, bán kính R. Giả sử C là điểm
cố định trên tia đối của tia BA. Một cát tuyến thay đổi qua C cắt đường tròn (O) tại D và
E (D nằm giữa C và E). Các đường tròn ngoại tiếp các tam giác BCD và ACE cắt nhau
tại giao điểm thứ hai M. Biết rằng bốn điểm O, B, M, E tạo thành tứ giác OBME. Chứng
minh rằng

a) Tứ giác OBME nội tiếp.

b) CD ·CE = CO2 − R2 .

c) M luôn di chuyển trên một đường tròn cố định.

Lời giải.
120 TUYỂN TẬP ĐỀ THI VÀO LỚP 10 CHUYÊN TOÁN

O D

1
A B C

A
á n
To
d = EAC
a) Do các tứ giác ACME, BCMD và ABDE là các tứ giác nội tiếp nên ta được EAB d =

180 − EMC,
[ BMC [ = BDCd = EAB.
d Vì vậy, ta được

[ = 180◦ − EAB d = 180◦ − 2EAB


d = 180◦ − EOB.

[ − BMC
[ = EMC
EMB d − BDC [
cb

[ = 180◦ , do đó tứ giác OBME là tứ giác nội tiếp.


[ + EOB
Vậy EMB

b) Gọi N là trung điểm của DE. Do 4ODE cân tại O nên ON ⊥ DE, đồng thời ON là phân
lạ

giác của DOE.


[ Vì vậy ta được

CO2 − R2 = CO2 − OD2 = (ON 2 +CN 2 ) − (ON 2 + DN 2 ) = CN 2 − DN 2


u

= (CN − DN)(CN + DN) = CD · (CN + NE) = CD ·CE.


c) Vì O là tâm ngoại tiếp của 4BDE nên ta được E


[ BD = 12 EOD
[ = EON.
[ Kết hợp với việc
[ = MOE
các tứ giác OBME và BCMD là cá tứ giác nội tiếp, ta suy ra MON [ − EON [ =
[ −E
MBE [ BD = MBD
[ = MCD [ = MCN.[ Từ đó suy ra tứ giác MCON là tứ giác nội tiếp. Vì
[ = ONC
thế OMC ◦
[ = 90 . Chú ý rằng O,C đều là các điểm cố định. Như vậy M luôn chạy
trên đường tròn cố định với đường kính OC.

Câu 3
Tìm tất cả các số nguyên dương N sa cho N có thể biểu diễn một cách duy nhất ở dạng
x2 + y
với x, y là hai số nguyên dương.
xy + 1
CÂU LẠC BỘ TOÁN A1, HOTLINE: 034 761 1986 - 035 290 3286 121

x2 + y
Lời giải. +) Với N = 1, ta chỉ ra cặp có hai cặp (x, y) = (1, 1), (1, 2) để N = .
xy + 1
+) Với N ≥ 2, vì N là số nguyên dương bất kỳ lớn hơn 1 thì N có thể biểu diễn thành N =
N4 + N 2
2 , y = N thì N = x + y . Vậy với mọi N > 1 đều có ít nhất một
nên nếu chọn x = N
N2 · N + 1 xy + 1
cách biểu diễn dưới dạng đã cho.
Ta có
xy + 1 | x2 + y
x2 + y
nên xy + 1 | y(x2 + y) − x(xy + 1) = y2 − x. Từ đây lưu ý rằng nếu x ≤ y thì ≤ 1. Do đó
xy + 1
x2 + y
để ≥ 2 thì x > y. Khi đó có
xy + 1

1
−xy − 1 < −x < y2 − x < y2 < xy + 1

A
Từ đây ta phải có x = y2 . Vậy ứng với mỗi N là số tự nhiên, N > 1 thì chỉ có đúng một cách chọn
là y = N, x = N 2 .
Vậy đáp số là các số nguyên dương N ≥ 2.

Câu 4 á n
To
Cho a, b, c là ba số nguyên dương sao cho mỗi số đó đều biểu diễn được ở dạng luỹ thừa
của 2 với số mũ tự nhiên.
Biết rằng phương trình bậc hai ax2 − bx + c = 0 (1) có cả hai nghiệm đều là số nguyên.

Chứng minh rằng hai nghiệm của phương trình (1) bằng nhau.
cb

Lời giải.
Cách 1. Ta viết a = 2m , b = 2n , c = 2k trong đó m, n, k là các số tự nhiên và gọi hai nghiệm đã
lạ

cho là x1 , x2 . Theo định lý Vieta, ta có x1 + x2 = 2n−m và x1 x2 = 2k−m . Từ đây suy ra x1 , x2 > 0


và đều là luỹ thừa của 2. Từ đó ta có thể viết x1 = 2 p và x2 = 2q với p, q đều là các số tự nhiên
u

và không mất tính tổng quát giả sử p ≥ q. Thế thì ta có 2 p + 2q = 2n−m hay tương đương
2q (2 p−q + 1) = 2n−m .

Nếu p > q thì 2 p−q + 1 là số lẻ lớn hơn 1, trong khi đó vế phải của đẳng thức trên là luỹ thừa
của 2, mâu thuẫn. Suy ra p = q và ta có điều phải chứng minh.
Cách 2. Ta đặt a = 2x , b = 2y , c = 2z , với x, y, z là các số tự nhiên
Xét phương trình (1), ta có:

∆ = b2 − 4ac = 22y − 22+x+z = 22+x+z (22y−x−z−2 − 1)


Do phương trình (1) có 2 nghiệm đều là số nguyên nên ∆ là số chính phương. Dẫn tới số mũ của
2 trong ∆ là chẵn, tức là 2 + x + z chẵn. Ta đặt x + z + 2 = 2k. Khi đó:
∆ = 22k (22y−2k − 1).
Mà cả ∆ và 22k đều là số chính phương, 22k > 0 cho nên 22y−2k − 1 cũng phải là số chính
phương. Mà 22y−2k = 22(y−k) là số chính phương, cho nên 22y−2k = 1, và ∆ = 0 khiến 2 nghiệm
của phương trình trùng nhau.
122 TUYỂN TẬP ĐỀ THI VÀO LỚP 10 CHUYÊN TOÁN

2.18. ĐỀ THI TUYỂN SINH VÀO LỚP 10 CHUYÊN TOÁN HÀ


NỘI, NĂM 2021
Câu 1

1. Giải phương trình x2 + x + 2 − 2 x + 1 = 0.

2. Cho ba số thực a, b và c thoả mãn ab + bc + ca = 1. Chứng minh


a−b b−c c−a
2
+ 2
+ = 0.
1+c 1+a 1 + b2

1
Lời giải.
1. Điều kiện xác định: x ≥ −1.

A
Ta viết lại phương trình dưới dạng

x2 + ( x + 1 − 1)2 = 0.

n
√ √
á
Vì x2 ≥ 0, ( x + 1 − 1)2 ≥ 0 nên ta phải có x2 = ( x + 1 − 1)2 = 0 và từ đó suy ra x = 0.
To
2. Gọi vế trái của đẳng thức cần chứng minh là P. Vì ab + bc + ca = 1 nên a2 + 1 = a2 +
ab + bc + ca = a.(a + b) + c.(a + b) = (a + b)(a + c).
Tương tự b2 + 1 = (b + a)(b + c) và c2 + 1 = (c + a)(c + b).

Từ đó suy ra
cb

a−b b−c c−a (a − b) b−c c−a


P= 2
+ 2
+ 2
= + + .
1+c 1+a 1+b (c + a)(c + b) (a + b)(a + c) (b + a)(b + c)
lạ

Quy đồng, ta được


(a − b)(a + b) + (b − c)(b + c) + (c − a)(c + a) a2 − b2 + b2 − c2 + c2 − a2
P= = = 0.
(a + b)(b + c)(c + a) (a + b)(b + c)(c + a)
u

Câu 2
1. Tìm tất cả các cặp số nguyên (x, y) thoả mãn x2 + 5xy + 6y2 + x + 2y − 2 = 0.

2. Chứng minh rằng với mỗi số nguyên n, số n2 + n + 16 không chia hết cho 49.

Lời giải.
1. Ta viết x2 + 5xy + 6y2 = x2 + 2xy + 3xy + 6y2 = x(x + 2y) + 3y(x + 2y) = (x + 3y)(x + 2y).
Từ đó phương trình đã cho có thể viết lại thành
(x + 2y)(x + 3y + 1) = 2.
Vì x, y là các số nguyên, nên x + 2y và x + 3y + 1 là các số nguyên. Nên ta chỉ cần xét 4
trường hợp sau:
CÂU LẠC BỘ TOÁN A1, HOTLINE: 034 761 1986 - 035 290 3286 123

• Trường hợp 1. x + 2y = 1 và x + 3y + 1 = 2 và ta tìm được (x, y) = (1, 0).


• Trường hợp 2. Với x + 2y = 2 và x + 3y + 1 = 1 thì y = −2 và x = 6.
• Trường hợp 3. Với x + 2y = −1 và x + 3y + 1 = −2 thì y = −2 và x = 3.
• Trường hợp 4. Với x + 2y = −2 và x + 3y + 1 = −1 thì y = 0 và x = −2.
Vậy các cặp (x, y) cần tìm là (1, 0); (6, −2); (3, −2); (−2, 0).
2. Giả sử tồn tại số nguyên n sao cho n2 + n + 16 chia hết cho 49. Suy ra 4.(n2 + n + 16) chia
hết cho 49. Mà
4 · (n2 + n + 16) = 4n2 + 4n + 1 + 63 = (2n + 1)2 + 63
Vì 4 · (n2 + n + 16) chia hết cho 49 nên (2n + 1)2 + 63 chia hết cho 7, kéo theo (2n + 1)2
chia hết cho 7. Do đó, (2n + 1)2 chia hết cho 49. Nhưng khi đó, vì 63 không chia hết cho

1
49 nên (2n + 1)2 + 63 không chia hết cho 49. Điều mâu thuẫn này chứng tỏ giả sử là sai

A
và từ đó ta có điều phải chứng minh.

n
Câu 3

1. Cho số thực x khác 0 thoả mãn x +


2
á
và x3 là số hữu tỉ. Chứng minh x là số hữu tỉ.
To
x
2. Cho các số thực không âm a, b và c thoả mãn a + b + c = 5. Chứng minh rằng
2a + 2ab + abc ≤ 18.

cb

Lời giải.
2 x2 + 2 x4 + 2x2
1. Ta có: x + = = ∈ Q ⇒ x4 + 2x2 ∈ Q.
x x x3
lạ

2 4
Mặt khác do x + ∈ Q nên x2 + 2 ∈ Q. Suy ra
x x
x6 − 8 x7 − 8x
u

4 8
x4 + 2x2 − 2(x2 + ) = x 4
− = = ∈ Q.
x2 x2 x2 x3

Từ đó x7 − 8x ∈ Q hay x(x6 − 8) ∈ Q.
Chú ý x6 − 8 6= 0 do x3 ∈ Q nên x6 − 8 ∈ Q nên x ∈ Q.
(b + c + 2)2 (7 − a)2
2. Theo BĐT AM-GM, ta có 2ab + abc = ab(2 + c) 6 a =a . Do đó,
4 4
(7 − a)2
2a + 2ab + abc 6 2a + a .
4
(7 − a)2
Giờ, ta sẽ chứng minh 2a + a 6 18. Bất đẳng này thức tương đương với
4
(a − 8)(a − 3)2 6 0
luôn đúng. Dấu bằng xảy ra khi a = 3, b = 2, c = 0.
124 TUYỂN TẬP ĐỀ THI VÀO LỚP 10 CHUYÊN TOÁN

Câu 4
d = 60◦ và AB < AC. Các đường
Cho tam giác nhọn ABC nội tiếp đường tròn (O), với BAC
thẳng BO,CO lần lượt cắt các đoạn thẳng AC, AB tại M, N. Gọi F là điểm chính giữa của
cung BC lớn.

(a) Chứng minh năm điểm A, N, O, M và F cùng thuộc một đường tròn.

(b) Gọi P, Q lần lượt là các giao điểm thứ hai của hai tia FN, FM với đường tròn (O).
Gọi J là giao điểm của đường thẳng BC và đường thẳng PQ. Chứng minh tia AJ là
tia phân giác của góc BAC.
d

(c) Gọi K là giao điểm của đường thẳng OJ và đường thẳng CF. Chứng minh AB vuông
góc với AK.

1
A
F

n
A

á K
To
D
N

cb

P O
lạ
u

B J C

Lời giải.
(a) Do O là tâm ngoại tiếp của 4ABC nên MON
[ = BOC
d = 2BACd = 120◦ . Như vậy, MAN
[+
◦ ◦ ◦
[ = 60 + 120 = 180 và vì thế A, M, O, N cùng thuộc một đường tròn.
MON
Trong khi đó, vì F là trung điểm cung lớn BC nên FB = FC, hơn nữa kết hợp với BFC
d =
d = 60◦ ta thu được 4BFC đều. Từ đó, FO là phân giác BFC
BAC [ = 30◦ . Như
d và do đó BFO
[ = MBC
vậy, AMO [ + ACB d = 30◦ + AFBd = BFO [ + AFBd = AFO.
[ Từ đó suy ra A, F, M, O
cùng thuộc một đường tròn. Vì vậy năm điểm A, N, O, M và F cùng thuộc một đường tròn.
CÂU LẠC BỘ TOÁN A1, HOTLINE: 034 761 1986 - 035 290 3286 125

[ = AOF
(b) Ta có ANF d = 2ABF,d suy ra 4NBF cân tại N. Do đó, APF
d = ABFd = PFB,
d từ đó ta
được AP k FB và APBF là hình thang cân. Vì vậy, MQJ
[ = FQP
[ = FBP d = BFA
d = BCA d =
[ = CQM
[ từ đó suy ra tứ giác CMJQ nội tiếp và vì thế CJM
JCM, [ = CQF d = 60◦ =
[ = CBF
d = JBM
d và tứ giác ABJM nội tiếp. Vậy JAM
BAC d = CBO d = 30◦ = 1 BAC
d và ta suy ra được
2
AJ là phân giác của BAC.
d

(c) Gọi D là giao điểm của AJ và FB. Vì các tứ giác NOJB, AMON, AMJB là các tứ giác
d = CJO
nội tiếp nên ta được CJK d = BNO[ = AMO [ = AMB
[ = AJBd = BJD.d Lại chú ý rằng
d d ◦
JBD = JCK = 60 nên 4BJD ∼ 4CJK (g.g), kết hợp với tính chất phân giác ta suy ra được
BD BJ AB ◦
CK = CJ = AC . Do đó, 4ABD ∼ 4ACK (c.g.c), như vậy, CAK = BAD = BAJ = 30 . Vì
d d d
d = BAC
vậy, BAK d + CAKd = 60◦ + 30◦ = 90◦ .

1
Câu 5

A
Cho A là một con có 100 phần tử của tập hợp {1, 2, 3, ..., 178}.

n
1. Chứng minh A chứa hai số tự nhiên liên tiếp.

á
2. Chứng minh với mọi số tự nhiên n thuộc tập hợp {2, 3, ..., 22}, tồn tại hai phần tử
To
của A có hiệu bằng n.

Lời giải.

1. Ta chia tập {1, 2, ..., 178} thành 89 cặp liên tiếp {1, 2}, {3, 4}, ..., {177, 178}. Vì A chứa
cb

100 phần tử mà chỉ có 89 cặp nên theo nguyên lý Dirichlet phải có hai phần tử cùng thuộc
một cặp và hai phần tử này là hai số liên tiếp.
lạ

2. Xét số tự nhiên n, với 2 ≤ n ≤ 22. Giả sử 178 chia cho 2n dư k với k < 2n, ta đặt 178 =
2an + k.
u

Ta chia tập hợp {1, 2, ..., 178} thành các cặp dưới đây

(1, 1 + n), (2, 2 + n), ...(n, 2n), (2n + 1, 3n + 1), (2n + 2, 3n + 2), ..., (2an − n, 2an),

và thêm các nhóm

• {2an + 1}, {2an + 2}, ..., {2an + k} nếu k < n.


• (2an + k, 2an + k − n), (2an + k − 1, 2an + k − 1 − n), ..., (2an + k − n, 2an + k − 2n)
nếu k ≥ n.

Do 2an + k − 2an ≤ 2an nên các cặp trên sẽ phủ hết tập A. Và ta có

• an + k cặp nếu k < n


• an + n cặp nếu k ≥ n
n+k
Số cặp luôn nhỏ hơn hoặc bằng an + , từ đó ta có
2
126 TUYỂN TẬP ĐỀ THI VÀO LỚP 10 CHUYÊN TOÁN

• Nếu n = 22, ta có 178 = 8.22 + 2 nên số cặp là 4.22 + 2 = 90 < 100, theo nguyên
lý Dirichlet, có 2 số thuộc cùng 1 cặp, vậy có 2 số có hiệu là 22
n+k
• Nếu n ≤ 21, ta có 2an + n + k = 178 + n ≤ 178 + 21 = 199 dẫn tới an + < 100
2
nên số cặp nhỏ hơn hẳn 100. Do đó, theo nguyên lý Dirichlet thì sẽ có 2 số thuộc
cùng 1 cặp, do đó chúng có hiệu là n.

1
A
á n
To

cb
lạ
u

CÂU LẠC BỘ TOÁN A1, HOTLINE: 034 761 1986 - 035 290 3286 127

2.19. ĐỀ THI TUYỂN SINH VÀO LỚP 10 CHUYÊN TIN HÀ


NỘI, NĂM 2021
Câu 1

1. Giải phương trình 4 + 2x − x2 = x − 2.
(
x3 + 2 = 3y
2. Giải hệ phương trình 3 .
y + 2 = 3x

Lời giải.

1
1. Điều kiện xác định : 4 + 2x − x2 ≥ 0. Với x ≥ 2, bình phương hai vế của phương trình đã
cho, ta có

A
4 + 2x − x2 = (x − 2)2
hay tương đương

n
2x2 − 6x = 0.

á
Giải ta tìm được x = 0 và x = 3. Loại nghiệm x = 0 nên phương trình đã cho có đúng một
To
nghiệm x = 3.
2. Trừ vế với vế tương ứng của hệ phương trình ta được và phân tích thành thừa số ta được
(x − y)(x2 − xy + y2 + 3) = 0.

y 2 3 2
cb


Vì x2 −xy+y2 +3 = x − + y +3 > 0 nên ta phải có x = y. Từ đây thay vào phương
2 4
trình đầu tiên của hệ, ta được
x3 + 2 = 3x
lạ

hay tương đương


(x − 1)2 (x + 2) = 0.
u

Giải phương trình trên, suy ra x = 1, x = −2. hệ đã cho có hai nghiệm (x, y) là (1, 1) và

(−2, −2).

Câu 2
1. Chứng minh rằng với mỗi số nguyên n, số n2 + 3n + 16 không chia hết cho 25.

2. Tìm tất cả các số nguyên x và y thoả mãn x2 − xy − 2y2 + x + y − 5 = 0.

Lời giải.
1. Giả sử phản chứng n2 + 3n + 16 chia hết cho 25, thế thì 4(n2 + 3n + 16) = (2n + 3)2 + 55
cũng chia hết cho 25. Suy ra (2n + 3)2 + 55 chia hết cho 5. Vì 55 chia hết cho 5 nên
(2n + 3)2 chia hết cho 5, kéo theo 2n + 3 chia hết cho 5. Do đó, (2n + 3)2 chia hết cho 25,
nhưng khi đó (2n + 3)2 + 55 lại không chia hết cho 25, mâu thuẫn. Vậy, điều giả sử là sai
và từ đó ta có điều phải chứng minh
128 TUYỂN TẬP ĐỀ THI VÀO LỚP 10 CHUYÊN TOÁN

2. Viết lại phương trình đã cho dưới dạng


(x + y)(x − 2y + 1) = 5.
Từ đây, ta có 4 trường hợp
• Với x + y = 1 và x − 2y + 1 = 5 thì x = 2, y = −1.
• Với x + y = 5 và x − 2y + 1 = 1 thì 3y = 5 loại vì y nguyên.
• Với x + y = −1 và x − 2y + 1 = −5 thì 3y = 5 loại vì y nguyên.
• Với x + y = −5 và x − 2y + 1 = −1 thì y = −1 và x = −4.
Vậy (x, y) là (2, −1) và (−4, −1).

1
Câu 3

A
1. Cho a, b, c thực đôi một phân biệt. Chứng minh:
        
a+b b+c b+c c+a c+a a+b
+ + = −1.

n
a−b b−c b−c c−a c−a a−b

a b c á
To
2. Cho biểu thức P = √ +√ +√ với a, b, c không âm và
1 + 2bc 1 + 2ca 1 + 2ab
a2 + b2 + c2 = 1.
Tìm giá trị lớn nhất của P.

cb

Lời giải.
a+b b+c c+a
1. Đặt x = ,y = ,z = . Cần chứng minh: xy + yz + zx = −11.
a−b b−c c−a
lạ


2a 2b 2c
 x+1 =
 ,y+1 = ,z+1 =
Khi đó a−b b−c c−a
2b 2c 2a
u

 x−1 =
 ,y−1 = ,z−1 =
a−b b−c c−a

Suy ra (x + 1)(y + 1)(z + 1) = (x − 1)(y − 1)(z − 1). Khai triển phá ngoặc, ta được xy +
yz + zx = −1.
1
2. Áp dụng bất đẳng thức Bunhiacovksy, ta có a2 + (b + c)2 > (a + b + c)2 . Suy ra
2

a a a 2a
√ =√ =p 6 .
1 + 2bc a2 + b2 + c2 + 2bc a2 + (b + c)2 a + b + c
√ √
b 2b c 2c
Tương tự ta có √ 6 và √ 6 .
1 + 2ca a + b + c 1 + 2ab a + b + c

 
1 1
Từ đó, P 6 2 và dấu bằng xảy ra khi (a, b, c) là hoán vị của √ , √ , 0 . Vậy, giá trị
√ 2 2
lớn nhất của P là 2.
CÂU LẠC BỘ TOÁN A1, HOTLINE: 034 761 1986 - 035 290 3286 129

Câu 4
Cho tam giác nhọn ABC nội tiếp đường tròn (O) và AB < AC. Gọi I là tâm đường tròn
nội tiếp của tam giác ABC. Đường thẳng AI cắt đường tròn (O) tại điểm thứ hai M (M
khác A). Gọi D, E và F lần lượt là các hình chiếu của điểm I trên các cạnh BC,CA và AB.

(a) Chứng minh tam giác MBI là tam giác cân.

(b) Đường tròn ngoại tiếp tam giác AEF cắt đường tròn (O) tại điểm thứ hai P (P khác
A). Chứng minh P, M và D là ba điểm thẳng hàng.

(c) Gọi H là giao điểm của đường thẳng IP và đường thẳng EF. Chứng minh HD song
song với AM.

1
A
A

n
P

á
To
E
H
F

I
cb

B D C
lạ
u

M

Lời giải.
d = MBC
(a) Ta có phép biến đổi góc MBI [ + IBC
d = MAC
[ + IBA
d = IAB
d + IBA
d = MIB,
d vì thế
4MBI cân tại M.
d = AEP
(b) Do tứ giác AEFP nội tiếp nên AFP d hay PFB d = PEC. d Kết hợp với PBF d = PBA d =
PCA = PCE, nên ta suy ra được 4PFB ∼ 4PEC (g.g). Chú ý rằng BF = BD và CE = CD,
d d
PB BF BD
ta thu được PC = CE = CD . Theo tính chất phân giác ta suy ra PD là phân giác của BPC.
d Mà
M là trung điểm của cung BC không chứa A, do vậy ta suy ra được ba điểm P, D, M thẳng
hàng.

(c) Từ 4PFB ∼ 4PEC ta suy ra FPE


d = FPCd + CPEd = FPC d + BPFd = BPC d và PF = PB , vì
PE PC
thế 4PFE ∼ 4PBC (c.g.c). Ta có IE = IF nên PI là phân giác của FPE.
d Ta chứng minh
130 TUYỂN TẬP ĐỀ THI VÀO LỚP 10 CHUYÊN TOÁN

PH HF IH
được 4PFH ∼ 4PBD (g.g) và 4IFH ∼ 4MBD (g.g). Từ đó suy ra PD = BD = DM hay
PH PD
HI = DM . Theo định lý Thales đảo ta suy ra HD k AM.

Câu 5
Trên bàn có n viên kẹo. Hai bạn An và Bình cùng chơi một trò chơi như sau: Hai bạn luân
phiên lấy kẹo trên bàn, mỗi lần chỉ được lấy 1, 2, 3, 4 hoặc 5 viên kẹo và phải lấy số viên
kẹo khác với số viên kẹo của bạn còn lại vừa lấy ngay trước đó. Bạn đầu tiên không thể
thực hiện được lượt chơi của mình là người thua cuộc. Nếu An là người lấy kẹo trước,

1. Với n = 7, hãy chỉ ra chiến thuận chơi của Bình khiến An là người thua cuộc.

2. Với n = 22, hãy chỉ ra chiến thuận chơi của An khiến là người thua cuộc.

1
A
Lời giải.
1. Chiến thuật chơi của Bình

n
• Nếu An lấy 1 viên kẹo trong lượt đầu thì Bình lấy 3 viên, lúc này còn 3 viên kẹo.
á
Lượt sau An không thể lấy 3 viên nên chỉ có thể lấy k viên với 1 ≤ k ≤ 2, khi đấy
To
Bình sẽ lấy 3 − k viên còn lại và thắng (lưu ý 3 − k 6= k ∀k ∈ Z và 1 ≤ 3 − k ≤ 2 nên
chiến thuật này có thể làm)
• Nếu An lấy ` viên kẹo trong lượt đầu (2 ≤ ` ≤ 5) thì Bình lấy nốt 7 − ` viên còn lại.

Lưu ý ` 6= 7 − ` ∀` ∈ Z và 2 ≤ 7 − ` ≤ 5 nên chiến thuật này có thể làm.


cb

2. Chiến thuật chơi của An như sau

• Lượt đầu, An bốc 2 viên, còn 20 viên.


lạ

• Tại lượt sau nếu Bình bốc k viên với k ≥ 2 thì An bốc 7 − k viên, còn 13 viên. Nếu
Bình bốc 1 viên thì An bốc 3 viên. Khi đó nếu Bình bốc ` viên với 1 ≤ ` ≤ 2 thì An
bốc 3 − ` viên. còn 13 viên. Nếu Bình bốc ` viên với 4 ≤ ` ≤ 5 thì An bốc 9 − ` viên
u

(lưu ý khi này 4 ≤ 9 − ` ≤ 5), còn lại 7 viên. Như vậy sau bước này thì số bi còn lại

là 13 viên hoặc 7 viên và đến lượt Bình bốc.


• Nếu số bi còn lại 7 viên thì An chỉ cần dựa theo chiến thuật tại câu 1) thì sẽ chiến
thắng. Nếu số bi còn lại 13 viên thì An sẽ bốc tiếp như sau. Nếu Bình bốc m viên
với m 6= 3 thì An bốc 6 − m viên, lúc này còn lại 7 viên và đến lượt Bình bốc, khi
đó An chỉ cần dựa theo chiến thuật câu 1) là thắng. Nếu Bình bốc 3 viên thì An sẽ
bốc 5 viên, còn lại 5 viên và đến lượt Bình bốc. Tuy nhiên Bình chỉ có thể bốc t viên
với 1 ≤ t ≤ 4 (do t 6= 5 do An bốc 5 viên ở lượt trước), khi đó tại lượt sau An sẽ bốc
5 − t viên còn lại và chiến thắng.
CÂU LẠC BỘ TOÁN A1, HOTLINE: 034 761 1986 - 035 290 3286 131

2.20. ĐỀ THI TUYỂN SINH VÀO LỚP 10 TRƯỜNG THPT CHUYÊN


KHOA HỌC TỰ NHIÊN NĂM 2022, VÒNG 1

Câu 1: (4 điểm)
1. Giải hệ phương trình
(
6(xy + 5) + x3 y + 5x2 = 42
.
x3 + 5x2 y + 6x + 30y = 42

2. Giải phương trình


√ √

1
p
( 3 x + 6 + 3 3 − x)(2 + 3 3 (x + 6)(3 − x)) = 24.

A
Lời giải.

n
1. Trừ vế với vế của 2 phương trình ta được

á
(x3 y − x3 ) + (6xy − 6x) − (5x2 y − 5x2 ) − (30y − 30) = 0
To
tương đương với
(y − 1)(x − 5)(x2 + 6) = 0.

Từ đây ta có y = 1 hoặc x = 5.
cb

• Với y = 1 thì x = 1.
−113
• Với x = 5 thì y = .
155
lạ

 
−113
Vậy hệ phương trình có nghiệm (x, y) là (1, 1); 5, .
155
u

√ √
2. Đặt 3 x + 6 = a, 3 3 − x = b.

(
(a + b)(2 + 3ab) = 24
Khi đó ta có hệ phương trình . Cộng hai phương trình lại ta
a3 + b3 = 9
suy ra (a + b)3 + 2(a + b) − 33 = 0 hay tương đương

(a + b − 3)((a + b)2 + 3(a + b) + 11) = 0.

Chú ý là (a + b)2 + 3(a + b) + 11 > 0 nên ta phải có a + b = 3, kéo theo ab = 2. Giải ta


tìm được (a, b) là (1, 2) hoặc (2, 1). Xét hai trường hợp

• a = 1, b = 2. Giải tìm được x = −5.


• a = 2, b = 1. Giải tìm được x = 2.

Vậy hệ phương trình có hai nghiệm là 2 và −5.


132 TUYỂN TẬP ĐỀ THI VÀO LỚP 10 CHUYÊN TOÁN

Câu 2: (2 điểm)

1) Tìm tất cả các cặp số nguyên (x, y) thoả mãn đẳng thức

25y2 + 354x + 60 = 36x2 + 305y + (5y − 6x)2022 .

2) Trên bàn có 8 hộp rỗng (trong các hộp không có viên bi nào). Người ta thực hiện
các lần thêm bi vào các hộp theo quy tắc sau: mỗi lần ta chọn ra 4 bất kỳ và bỏ vào
một hộp 1 viên, một hộp 2 viên, hai hộp còn lại mỗi hộp 3 viên. Hỏi số lần thêm bi
ít nhất có thể nhận được số bi ở 8 hộp trên là 8 số tự nhiên liên tiếp?

Lời giải.

1
1) Phương trình đã cho có thể viết lại dưới dạng

A
(5y − 6x − 1)(5y + 6x − 60) = (5y − 6x)2022 .

n
Rõ ràng (5y − 6x − 1, 5y − 6x) = 1 nên (5y − 6x − 1, (5y − 6x)2022 ) = 1, từ đó ta phải có
5y − 6x − 1 = ±1. Xét hai trường hợp á
To
– Nếu 5y − 6x − 1 = 1 thì 5y + 6x − 60 = 22022 . Cộng lại, ta có 10y = 22022 + 62. Tuy
nhiên 22022 = (24 )505 .22 = 16505 .4 có tận cùng là 4 nên 22022 + 62 có tận cùng là 8
không chia hết cho 10. Trường hợp này phương trình vô nghiệm.

– Nếu 5y − 6x − 1 = −1 thì 5y + 6x − 60 = 0. Từ đây tìm được y = 6, x = 5.


cb

Vậy phương trình đã cho có đúng một nghiệm là (5; 6).


lạ

2) Gọi số lần thêm ít nhất là n thì các hộp sẽ có số bi tương ứng là a, a + 1, a + 2, . . . , a + 7.


Mỗi lần thêm tất cả là 9 bi nên ta có
u

a + (a + 1) + (a + 2) + · · · + (a + 7) = 9n

hay tương đương


8a + 28 = 9n.
Suy ra 4(2a + 7) chia hết cho 9, kéo theo 2a + 7 chia hết cho 9. Từ đó ta phải có 2a + 7 ≥ 9
dẫn tới n ≥ 4. Ta chứng minh n = 4 thoả mãn.
Gọi các hộp tương ứng là 1, 2, 3, 4, 5, 6, 7, 8 và thực hiện thêm bi ở các lần như sau

– Lần 1: 1, 2, 0, 0, 0, 0, 3, 3;
– Lần 2: 0, 0, 1, 0, 2, 3, 0, 3;
– Lần 3: 0, 0, 2, 1, 3, 0, 3, 0;
– Lần 4: 0, 0, 0, 3, 0, 3, 1, 2.
CÂU LẠC BỘ TOÁN A1, HOTLINE: 034 761 1986 - 035 290 3286 133

Câu 3: (3 điểm)

Cho hình chữ nhật ABCD (AB < AD) nội tiếp trong đường (O). Trên cạnh AD lấy hai điểm
d + DCF
E và F (E, F không trùn với A, D) sao cho E nằm giữa A và F, đồng thời ABE [=
1 d
2BOC.

1) Chứng minh BE và CF cắt nhau tại một điểm nằm trên đường tròn (O).

2) Đường thẳng qua O song song với BC cắt BE,CF theo thứ tự tại M, N. Chứng minh
[ + ADN
rằng DAM [ + 1 AOD[ = 180◦ .
2

3) Dựng hình chữ nhật MNPQ sao cho NQ song song với BD, đồng thời MP song
song với AC. Chứng minh rằng đường tròn ngoại tiếp hình chữ nhật MNPQ tiếp

1
xúc với đường tròn (O).

A
K

A E F á n
D
To
O
M N

cb

J X
B C
Q P
lạ

T
u

Lời giải.
1) Đặt BE ∩CF = K. Gọi H là hình chiếu của K trên AD. Để ý rằng KH k AB avà KH k CD
d = BKH
nên ta được BKC [ + CKH [ = ABE d + DCF[ = 1 BOC.d Từ đó suy ra K ∈ (O).
2

[ = KBC
2) Trước hết để ý rằng KMO d = KAC d = KAO[ ta thu được tứ giác KAMO là tứ giác
nội tiếp. Chứng minh tương tự, ta cũng có KDNO là tứ giác nội tiếp. Đặt AM ∩ DN = T .
d = MAO
Khi đó, ta có TAC [ = MKO [ = BKC d − NKO
[ = BDC d − NDO[ = T[ DC. Từ đó suy ra
T ∈ (O). Vì vậy,

[ + 1 AOD
[ + ADN
DAM dD + 1 AOD
[ = 180◦ − AT [ = 180◦ .
2 2

3) Gọi MP ∩ NQ = J. Để ý rằng JMN


[ = OCB d = OBC d = JNM[ nên JM = JN. Như vậy, ta
quy về chứng minh đường tròn (J, JM) tiếp xúc với (O).Thật vậy, ta cần chứng minh
134 TUYỂN TẬP ĐỀ THI VÀO LỚP 10 CHUYÊN TOÁN

OM XB
JM + OJ = R với R là bán kính của (O). Đặt KO ∩ BC = X. Khi đó, để ý rằng ON = XC ,
ta thấy 4JMN ∪ O ∼ 4OBC ∪ X. Do đó, ta được
KO OM MN JO JM JM JM + JO JM + JO
= = = = = = = .
KX BX BC OX OB OK OK + OX KX
Suy ra , JM + JO = OK = R. Như vậy, (MNPQ) ≡ (J, JM) tiếp xúc trong với (O).

Câu 4: (1 điểm)

Cho a, b, c là các số thực dương. Chứng minh rằng


2a a + b 6a + 2c 4a + 3b + c 32a
+ + + ≥ .

1
a+b a+c 3b + c b+c 2a + b + c

A
Lời giải.

n
Ta viết lại vế trái dưới dạng

P=
2a
+

2a
+
b+c
 
−1 +
2(a + c)
+á 4a
 
+
4a
+
3b + c
+
3b + c

To
a+b a+c a+c 3b + c 3b + c b + c 2(b + c) 2(b + c)
 
2a 2a 4a 4a 3b + c b + c 2(a + c) 3b + c
= + + + + + + + − 1.
a + b a + c 3b + c b + c 2(b + c) a+c 3b + c 2(b + c)

b + c 2(a + c) 3b + c
cb

Áp dụng bất đẳng thức AM-GM thì + + ≥ 3 nên


a+c 3b + c 2(b + c)

2a 2a 4a 4a 3b + c
P≥ + + + + + 2.
lạ

a + b a + c 3b + c b + c 2(b + c)
r r
2a 2a 8a 4a 3b + c 2a 4a 2a
u

Lưu ý là + ≥ , + ≥2 và +2 ≥ 4
a + b a + c 2a + b + c 3b + c 2(b + c) b+c b+c b+c

nên
r
8a 2a 8a 12a 8a 24a 32a
P≥ +6 = +p ≥ + = .
2a + b + c b + c 2a + b + c 2a(b + c) 2a + b + c 2a + b + c 2a + b + c

Phép chứng minh hoàn tất.


CÂU LẠC BỘ TOÁN A1, HOTLINE: 034 761 1986 - 035 290 3286 135

2.21. ĐỀ THI TUYỂN SINH VÀO LỚP 10 TRƯỜNG THPT CHUYÊN


KHOA HỌC TỰ NHIÊN NĂM 2022, VÒNG 2

Câu 1: (3.5 điểm)


1 1 1
1) Với a, b, c là những số thực dương thỏa mãn điều kiện + + = 1. Chứng minh
a b c
rằng s
 
1 1 1 1 abc
+ + = .
2 a + bc b + ca c + ab (a + bc)(b + ca)(c + ab)

2) Giải hệ phương trình

1
(
2x2 + 3xy + y2 = 6

A

3x + 2y + 1 = 2 2x + y + 6.

n
Lời giải.
á
To
1) Từ điều kiện ta có được ab + bc + ca = abc. Từ đó
 
1 1 1 1
VT = + +
2 a + bc b + ca c + ab

 
1 a b c
= + +
cb

2 a2 + ab + bc + ca b2 + ab + bc + ca c2 + ab + bc + ca
ab + bc + ca
=
(a + b)(b + c)(c + a)
lạ

s
a2 b2 c2
=
(a2 + ab + bc + ca) (b2 + ab + bc + ca) (c2 + ab + bc + ca)
u

s
a2 b2 c2
=

(a2 + abc) (b2 + abc) (c2 + abc)


s
abc
= .
(a + bc)(b + ca)(c + ab)

2) Điều kiện xác định: 2x + y + 6 ≥ 0. Hệ đã cho có thể viết lại dưới dạng

(x + y)(2x + y) = 6 p
(x + y) + (2x + y) − 2 2x + y + (x + y)(2x + y) + 1 = 0

(x + y)(2x + y)p=6
⇐⇒
(x + y + 1) − 2 (x + y + 1)(2x + y) + (2x + y) = 0.

Từ trên suy ra (x + y + 1) + (2x + y) và (x + y + 1)(2x + y) đều không âm nên (x + y +


√ (2x + y) là√các số không âm. Do đó, phương trình thứ hai của hệ có thể viết lại thành
1),
( x + y + 1 − 2x + y)2 = 0. Từ đây ta có ngay x + y + 1 = 2x + y hay x = 1. Từ đó ta giải
136 TUYỂN TẬP ĐỀ THI VÀO LỚP 10 CHUYÊN TOÁN

ra được x = 1, y = 1 hoặc x = 1, y = −4. Ta loại nghiệm (1, −4) vì 3 · 1 + 2 · (−4) + 1 < 0.


Vậy hệ đã cho có đúng một nghiệm (x, y) là (1, 1).

Câu 2: (2.5 điểm)

1) Tìm tất cả các cặp số nguyên dương (x, y) thỏa mãn đẳng thức

(x + y)(5x + y)3 + xy3 = (5x + y)3 + x2 y3 + xy4 .

2) Với a, b, c là những số thực dương thoả mãn các điều kiện sau
(
c ≤ b ≤ a ≤ 3, b2 + 2a ≤ 10, b2 + 2a + 2c ≤ 14,

1
(a2 + 1)(b2 + 1) + 4ab ≤ 2a3 + 2b3 + 2a + 2b.

A
Tìm giá trị lớn nhất của biểu thức

P = 4a2 + b4 + 2b2 + 4c2 .

Lời giải. á n
To
1) Ta viết lại phương trình thành (5x + y)3 (x + y − 1) − xy3 (x + y − 1) = 0, tức là
(x+y−1)[(5x+y)3 −xy3 ] = 0. Vì x, y nguyên dương nên x+y−1 > 0 nên xy3 = (5x+y)3 ,
5x + y 3
 

3 3
suy ra (5x + y) chia hết cho y dẫn đến 5x + y chia hết cho y. Vì vậy x = là lập
y
cb

phương của một số nguyên dương. Đặt x = a3 , với a nguyên dương. Ta suy ra ay = 5a3 + y
tương đương với 5a3 = y(a − 1), do đó 5a3 = 5(a3 − 1) + 5 chia hết cho (a − 1) nên a − 1
là ước dương của 5. Dẫn đến a ∈ {2, 6}.
lạ

Từ đó ta tìm được (x, y) = (8, 40), (216, 216).


2) Điều kiện thứ hai tương đương với
u

(a2 + 1 − 2b)(b2 + 1 − 2a) ≤ 0.


Tuy nhiên do a2 + 1 − 2b ≥ 2a − 2b ≥ 0 nên ta có b2 + 1 ≤ 2a.


Đặt x = 2a, y = b2 + 1, z = 2c. Ta suy ra x ≥ y từ nhận xét trên. Từ điều kiện thứ nhất ta
suy ra x + y = b2 + 2a + 1 ≤ 11, x + y + z = 2a + b2 + 1 + 2c ≤ 15, x = 2a ≤ 6. Ta cũng
có y = b2 + 1 ≥ 2b ≥ 2c = z, hay y ≥ z.
Ta có
P = 4a2 + (b2 + 1)2 + 4c2 − 1
= x2 + y2 + z2 − 1
= x(x − 6) + y(y − 5) + z(z − 4) + 6x + 5y + 4z − 1
= (x − y)(x − 6) + (y − z)(x + y − 11) + z(x + y + z − 15) + 4(x + y + z) + (x + y) + x − 1
≤ 0 + 0 + 0 + 4 · 15 + 11 + 6 − 1
= 76
CÂU LẠC BỘ TOÁN A1, HOTLINE: 034 761 1986 - 035 290 3286 137

Vậy Max P = 76 đạt được x = 6, y = 5, z = 4 hay a = 3, b = c = 2.

Câu 3: (3 điểm)

Cho tam giác ABC nhọn, không cân, nội tiếp đường tròn (O). Điểm P nằm trong tam giác
ABC. Gọi E, F lần lượt là hình chiếu vuông góc của P trên các cạnh CA, AB. Giả sử tứ
giác BCEF nội tiếp trong đường tròn (K).

1) Chứng minh rằng AP vuông góc với BC.

2) Chứng minh rằng AP = 2OK.

3) Đường thẳng qua P vuông góc với AP cắt đường tròn (O) tại hai điểm Q và R. Chứng

1
minh rằng đường tròn tâm A bán kính AP tiếp xúc với đường tròn ngoại tiếp tam giác
KQR.

A
X á n
To
A
t

D
J
cb

E
F
Q R
lạ

P
O
u

K

B C

Lời giải.
d = PAF
1) Để ý rằng các tứ giác AEPF và BCEF là các tứ giác nội tiếp nên ta được PAB d =
◦ ◦ ◦
90 − APF = 90 − AEF = 90 − ABC. Từ đó suy ra AP ⊥ BC.
d d d

2) Trước hết, ta có OK là đường trung trực của BC, vì thế OK ⊥ BC và kéo theo OK k AP. Để
d = 180◦ −AOC
ý rằng OAC
d
= 90◦ − ABC
d = 90◦ − AEF,d ta thu được OA ⊥ EF. Mặt khác, gọi
2
J là trung điểm của AP. Khi đó, vì J, K tương ứng là tâm của các đường tròn (AEPF) và
(BCEF) nên JK ⊥ EF, từ đó suy ra JK k AO. Kết hợp AP k OK đã chứng minh ở trên ta suy
ra AJKO là hình bình hành. Vì vậy OK = AJ = 21 AP hay AP = 2OK.
138 TUYỂN TẬP ĐỀ THI VÀO LỚP 10 CHUYÊN TOÁN

3) Dựng đường kính AT của (O), vì OJ là đường trung bình của 4APT nên PT k OJ. Do đó,
theo tiên đề Euclid thì P, K, T thẳng hàng và hơn nữa PT = 2OJ = 2PK nên K cũng là trung
điểm PT . Gọi D là giao điểm thứ hai của PT với đường tròn (O). Khi đó, vì AT là đường
kính của (O) nên ADP
d = ADT [ = 90◦ .
d = 90◦ nên 4AXP cân tại A, kéo theo
Gọi X là điểm đối xứng với P qua D. Khi đó, vì ADP
AX = AP hay X ∈ (A, AP). Mặt khác, ta có PX · PK = 2PD · PK = PD · 2PK = PD · PT =
PQ · PR, kéo theo tứ giác XQKR là tứ giác nội tiếp. Như vậy X ∈ (KQR).
Dựng tia tiếp tuyến Xt của (A, AP), khi đó Xt ⊥ AX. Để ý rằng vì OK k AP, AP ⊥ QR nên
OK ⊥ QR, dẫn đến OK chính là trung trực của QR. Như vậy KQ = KR, kéo theo XK là phân
[ 4XQP ∼ 4XKR và dẫn đến XPQ
giác của QXR, [=X [ d = PXt
RK. Từ đó ta được KXt d=

90 − AXP ◦
d = 90 − APX d = XPQ [ =X [ d =K
RK. Như vậy, KXt [ RX, từ đó kéo theo Xt cũng là
một tiếp tuyến của (KQR). Do vậy, (A, AP) tiếp xúc với (KQR) tại điểm X.

1
A
Câu 4: (1 điểm)

n
Cho các điểm A1 , A2 , ..., A30 theo thứ tự nằm trên một đường thẳng sao cho độ dài các đoạn
Ai Ai+1 bằng k (đơn vị dài), với k = 1, 2, ..., 29. Ta tô màu mỗi đoạn thẳng A1 A2 , A2 A3 , ...,
á
A29 A30 bởi một trong ba màu (mỗi đoạn được tô bởi đúng một màu). Chứng minh rằng
To
với mọi cách tô màu, ta luôn chọn được hai số nguyên dương 1 ≤ i ≤ j ≤ 29 sao cho hai
đoạn Ai Ai+1 , A j A j+1 được tô cùng màu và i − j là bình phương của một số nguyên dương.

Lời giải. Giả sử phản chứng, gọi các màu tương ứng là 1, 2, 3. Giả sử đoạn thẳng Ai Ai+1 được
cb

tô màu ai với ai ∈ {1, 2, 3}. Thế thì

• a1 , a10 , a26 đôi một phân biệt;


lạ

• a1 , a17 , a26 đôi một phân biệt;

Suy ra a10 = a17 = x1 . Chứng minh tương tự ta có


u

a11 = a18 = x2 , a12 = a19 = x3 , a13 = a20 = x4 .

Do 19 − 10 = 9 là số chính phương nên x1 6= x3 . Tương tự thì x1 6= x2 , x2 6= x3 . Suy ra x1 , x2 , x3


đôi một khác nhau. Tương tự thì x2 , x3 , x4 đôi một khác nhau. Do đó, x1 = x4 hay a13 = a17
nhưng 17 − 13 = 4 là số chính phương nên ta có điều vô lí. Vậy điều giả sử là sai và do đó ta có
điều phải chứng minh.
CÂU LẠC BỘ TOÁN A1, HOTLINE: 034 761 1986 - 035 290 3286 139

2.22. ĐỀ THI TUYỂN SINH VÀO LỚP 10 TRƯỜNG THPT CHUYÊN


SƯ PHẠM HÀ NỘI NĂM 2022, VÒNG 2

Câu 1: (2,5 điểm)


p
3 √
a) Không sử dụng máy tính, hãy tính giá trị của biểu thức P = 7+5 2 +
p
3 √
7 − 5 2.

b) Cho đa thức P(x) = ax2 + bx + c. Chứng minh rằng nếu P(x) nhận giá trị nguyên
với mỗi số nguyên x thì ba số 2a, a + b, c đều là những số nguyên. Sau đó, chứng tỏ
rằng nếu 2a, a + b, c là những số nguyên thì P(x) cũng nhận giá trị nguyên với mỗi
số nguyên x.

1
A
Lời giải.
a) Sử dụng hằng đẳng thức (a + b)3 = a3 + b3 + 3ab(a + b), ta có

n
3
q √ √
P = 14 + 3 (7 + 5 2)(7 − 5 2).P
kéo theo á
To
P3 + 3P − 14 = 0.
Phân tích nhân tử phương trình trên ta được (P − 2)(P2 + 2P + 7) = 0. Lưu ý là P2 + 2P +
7 = (P + 1)2 + 6 > 0 nên ta phải có P = 2 là số nguyên.

b) Vì P(x) ∈ Z với mọi x ∈ Z nên P(0), P(1), P(−1) là các số nguyên. Suy ra c, a + b + c, a −
cb

b + c là các số nguyên. Từ đó a + b, a − b cũng nguyên. Suy ra 2a = (a + b) + (a − b) ∈ Z.


Vậy, 2a, a + b, c là các số nguyên.
Đảo lại nếu 2a, a + b, c nguyên thì ta viết
lạ

x(x − 1)
P(x) = 2a. + (a + b)x + c
2
u

và lưu ý rằng x(x − 1) là tích hai số nguyên liên tiếp nên x(x − 1) chia hết cho 2. Từ đây

thì P(x) ∈ Z với mọi x nguyên.

Câu 2: (3,0 điểm)

Cho tam giác đều ABC ngoại tiếp đường tròn (O). Cung nhỏ OB của đường tròn ngoại
tiếp tam giác tam giác OBC cắt đường tròn (O) tại điểm E. Tia BE cắt đường tròn (O) tại
điểm thứ hai F.

a) Chứng minh tia EO là tia phân giác của góc CEF.

b) Chứng minh tứ giác ABOF nội tiếp.

c) Gọi D là giao điểm thứ hai của CE với đường tròn (O). Chứng minh ba điểm A, F, D
thẳng hàng.
140 TUYỂN TẬP ĐỀ THI VÀO LỚP 10 CHUYÊN TOÁN

E
D

1
B C

A
Lời giải.
a) Để ý rằng O vừa là tâm nội tiếp, vừa là tâm ngoại tiếp của tam giác đều ABC. Vì thế OB = OC

n
hay 4OBC cân tại O. Từ đó, để ý rằng tứ giác OEBC là tứ giác nội tiếp nên OEC[ = OBC d =
d = OEF.
OCB
á
[ Vì thế, tia EO là tia phân giác của CEF. [
To
b) Vì AO,CO tương ứng là các tia phân giác của BAC,
d ACBd nên OAB d = 30◦ và OCB
d = 1 BAC d =
2
1d ◦
2 ACB = 30 . Mặt khác, vì 4OEF cân tại O và tứ giác OEBC nội tiếp nên ta thu được
[ = OFE
OFB [ = OEF[ = OCB d = 30◦ . Như vậy, OFB d = 30◦ , vì vậy tứ giác AFOB nội
[ = OAB

tiếp.
cb

c) Vì 4OEF và 4OED cân tại O và OEF [ = OED [ = EOD


[ nên ta được EOF [ và 4OEF =
4OED. Như vậy, EF = ED hay 4DEF cân tại E. Hơn nữa, DEF = 180◦ − BEC
[ d = 180◦ −
d = 180◦ −120◦ = 60◦ . Như vậy, 4DEF là tam giác đều và do đó DFE
BOC [ = 60◦ . Mặt khác,
lạ

ta lại có AFB d = 120◦ (vì tứ giác ABOF nội tiếp). Do vậy, AFD
d = AOB [ = AFB
d + DFE[=
◦ ◦ ◦
120 + 60 = 180 và từ đó suy ra A, F, D thẳng hàng.
u

Câu 3: (2,0 điểm)

Cho a, b, c, d là các số nguyên dương thoả mãn ab = cd. Chứng minh rằng

N = a2022 + b2022 + c2022 + d 2022 là hợp số.

Lời giải. Đây là bài toán quen thuộc và có nhiều cách giải. Từ giả thiết, ta suy ra
a d
=
c b
a b
và ta viết hai phân số , dưới dạng phân số tối giản, tức là
c d
a d x
= = ,
c b y
CÂU LẠC BỘ TOÁN A1, HOTLINE: 034 761 1986 - 035 290 3286 141

trong đó x, y là các số nguyên dương nguyên tố cùng nhau. Suy ra tồn tại các số nguyên dương
m, n sao cho a = mx, c = my, d = nx, b = ny. Từ đó

N = (mx)2022 + (ny)2022 + (my)2022 + (nx)2022 = (m2022 + n2022 )(x2022 + y2022 ).

Lưu ý là do m, n, x, y nguyên dương nên m2022 + n2022 > 1 và x2022 + y2022 > 1, kéo theo ngay N
là hợp số.

Câu 4: (2,0 điểm)

Ta viết 10 số 0, 1, 2, . . . , 9 vào mười ô tròn trong hình bên, mỗi số được viết đúng một lần.
Sau đó ta tính tổng ba số trên mỗi đoạn thẳng để nhận được 6 tổng. Có hay không một
cách viết 10 số như thế sao cho 6 tổng nhận được là bằng nhau.

1
A
á n
To

cb

Lời giải. Phản chứng, giả sử có một cách viết thoả mãn đề bài. Gọi số ở tâm và ở 3 đỉnh của
tam giác đều lần lượt là a, b, c, d. Ta xét các số hạng trong 6 tổng được nhắc đến trong đề bài, ta
lạ

thấy các số a, b, c, d mỗi số xuất hiện đúng 3 lần trong 6 tổng này, còn 6 số còn lại mỗi số xuất
hiện một lần. Khi đó nếu gọi giá trị của 6 tổng này là x ∈ N, ta cộng vế cả 6 tổng lại thì có
u

3(a + b + c + d) + Tổng 6 số còn lại = 6x


Mà tổng của cả 10 số bằng 0 + 1 + · · · + 9 = 45 nên ta có

2(a + b + c + d) + 45 = 6x,

vô lý do vế trái là số lẻ còn vế phải là số chẵn.

Câu 5: (1,0 điểm)

a) Trong mặt phẳng cho 5 điểm sao cho không có ba điểm nào thẳng hàng. Chứng
minh rằng tồn tại ít nhất một tam giác tù có các đỉnh được lấy từ 5 điểm đã cho.

b) Trong mặt phẳng cho 2022 điểm sao cho không có ba điểm nào thẳng hàng. Chứng
minh rằng tồn tại ít nhất 2018 tam giác tù mà mỗi tam giác tù đó có các đỉnh được
lấy từ 2022 điểm đã cho.
142 TUYỂN TẬP ĐỀ THI VÀO LỚP 10 CHUYÊN TOÁN

Lời giải.
B A
C
A D

E
C E
D
E A B
B C
D

a) Gọi 5 điểm đã cho là A, B,C, D, E. Xét bao lồi của 5 điểm này. Có các khả năng sau đây

• Bao lồi là một ngũ giác lồi, giả sử là ABCDE. Thế thì tổng các góc trong ngũ giác

1
lồi này bằng 540◦ nên trong các góc này tồn tại một góc lớn hơn 90◦ . Không mất

A
d > 90◦ , tức là tam giác EAB là tam giác tù.
tính tổng quát giả sử EAB
• Bao lồi là một tứ giác lồi, giả sử là tứ giác ABCD. Nếu tồn tại một đỉnh của tứ
d > 90◦ , tức là tam giác DAB
giác mà góc ở đỉnh đó là góc tù, giả sử giả sử DAB

n
là tam giác tù. Nếu không thì tất cả các góc của tứ giác này đều bằng 90◦ , vì nếu
á
không sẽ có một góc lớn hơn 90◦ do tổng các góc trong tứ giác bằng 360◦ . Khi
To
đó tứ giác ABCD là hình chữ nhật. Điểm E nằm trong tứ giác ABCD và gọi α =
max{AEB,
d BEC, [ thì do tổng bốn góc này bằng 360◦ nên ta có ngay
[ DEA}
d CED,
α ≥ 90◦ . Không mất tính tổng quát, giả sử α = AEB.d Nếu α = 90◦ thì AEB d =
[ = 90◦ . Nhưng khi ấy thì các điểm A, E,C lại thẳng hàng, trái

d = CED
BEC [ = DEA
với giả thiết nên α > 90◦ thì tam giác AEB là tam giác tù.
cb

• Bao lồi là một tam giác ABC. Khi đó hai điểm D, E nằm trong tam giác. Ta thấy
d + BDC
ADB d + CDA d = 360◦ nên trong ba góc ADB, d BDC, d có ít nhất một góc
d CDA
lớn hơn 90◦ . Không mất tính tổng quát, giả sử là ADB
d > 90◦ thì tam giác ADB là
lạ

tam giác tù.


u

Tóm lại trong mọi trường hợp, đều tồn tại ít nhất một tam giác tù.

b) Ta chứng minh bằng quy nạp nếu có n ≥ 5 điểm thoả mãn thì sẽ tạo ra được n − 4 tam giác
tù có các đỉnh các là các điểm đã cho. Với n = 5, đã chứng minh ở câu a). Giả sử khẳng
định đúng đến n −1, ta chứng minh khẳng định cũng đúng với n. Xét n điểm A1 , A2 , ..., An .
Xét 5 điểm đầu tiên A1 , A2 , A3 , A4 , A5 ta tìm được một tam giác tù, giả sử là A1 A2 A3 với
đỉnh A1 tù. Bỏ đi điểm A1 , trong n − 1 điểm còn lại thì theo giả thiết quy nạp ta tìm được ít
nhất n − 5 tam giác tù và các tam giác tù này có các đỉnh thuộc tập {A2 , A3 , ..., An } không
trùng với tam giác tù đỉnh A1 . Do đó, có ít nhất n − 4 tam giác tù. Theo nguyên lý quy
nạp, khẳng định được chứng minh. Áp dụng bài toán với n = 2022 ta thu được điều phải
chứng minh.
CÂU LẠC BỘ TOÁN A1, HOTLINE: 034 761 1986 - 035 290 3286 143

2.23. ĐỀ THI TUYỂN SINH VÀO LỚP 10 CHUYÊN TOÁN HÀ


NỘI, NĂM 2022

Câu 1: (2.0 điểm)



1. Giải phương trình x2 − 4x + 2 2x − 1 + 1 = 0.

2. Cho các số thực a, b và c thỏa mãn điều kiện ab + bc + ca = 1. Tính giá trị của biểu
thức
a b c 2
P= 2
+ 2
+ 2

1+a 1+b 1+c a + b + c − abc

1
Lời giải.

A
1
1. Điều kiện xác định: x ≥ .
2
Phương trình đã cho tương đương với

n

x2 − 4x + 2 2x − 1 + 1 = 0
á √
⇔ x2 − 2x + 1 = 2x − 1 − 2 2x − 1 + 1
To
√ 2
2
⇔ (x − 1) = 2x − 1 − 1 .

Tới đây có hai trường hợp


√ √
cb

• Trường hợp 1. x − 1 = 2x − 1 − 1, khi đó ta có x = 2x − 1 tương đương với x2 −


2x + 1 = 0, vậy x = 1 và nghiệm này thoả mãn.
√ √
• Trường hợp 2. 1 − x = 2x − 1 − 1, khi đó ta có 2 − x = 2x − 1, với điều kiện x ≤ 2,
lạ

bình phương ta có x2 − 4x + 4 = 2x − 1 ⇔ x2 − 6x + 5 = 0. Chú ý là x ≤ 2 nên ta tìm


được x = 1.
u

Vậy phương trình đã cho có đúng một nghiệm x = 1.


2. Do ab + bc + ca = 1, ta có
a b c 2
P= 2
+ 2
+ 2

1+a 1+b 1+c a + b + c − abc
a b c 2
= 2 + 2 + 2 −
a + ab + bc + ca b + ab + bc + ca c + ab + bc + ca a + b + c − abc
a b c 2
= + + −
(a + b)(a + c) (b + c)(b + a) (c + a)(c + b) a + b + c − abc
2 2
= − .
(a + b)(b + c)(c + a) a + b + c − abc
Lại có
(a + b)(b + c)(c + a) = (a + b + c)(ab + bc + ca) − abc
Mà do ab + bc + ca = 1, thay vào ta có P = 0.
144 TUYỂN TẬP ĐỀ THI VÀO LỚP 10 CHUYÊN TOÁN

Câu 2: (2.0 điểm)

1) Chứng minh rằng nếu n là số tự nhiên lẻ thì 32n+1 − 7 chia hết cho 20.

2) Tìm tất cả cặp số nguyên dương (x, y) sao cho y(x2 + x + 1) = (x + 1)(y2 − 1)

Lời giải.
1) Ta có 32n+1 ≡ (−1)2n+1 ≡ −1 (mod 4). Do đó 32n+1 − 7 ≡ −1 − 7 ≡ 0 (mod 4)
Vì n lẻ, đặt n = 2a + 1 với a là số tự nhiên. Khi đó 32n+1 = 34a+3 = 81a .27 ≡ 1.2 ≡ 2
(mod 5), vì thế 32n+1 − 7 ≡ 0 (mod 5). Mặt khác (4, 5) = 1, do đó 32n+1 − 7 chia hết cho
20.

1
2) Đặt (y, y2 − 1) = d, khi đó y và y2 − 1 đều chia hết cho d, dẫn đến 1 chia hết cho d, nên d = 1.
Vậy (y, y2 − 1) = 1. Tương tự ta có (x2 + x + 1, x + 1) = 1

A
Ta có y(x2 + x + 1) = (x + 1)(y2 − 1) và x, y nguyên dương nên y(x2 + x + 1) chia hết cho
x + 1 và (x + 1)(y2 − 1) chia hết cho y. Mà (x2 + x + 1, x + 1) = (y2 −
(1, y) = 1, suy ra y chia

n
x+1 = y
hết cho x + 1 và x + 1 chia hết cho y, dẫn đến x + 1 = y. Vì vậy ta có 2 ,
á y − 1 = x2 + x + 1
To
từ đây ta tìm được x = 1, y = 2.

Câu 3: (2.0 điểm)


m3 n3
cb

1) Tìm hai số nguyên dương m, n sao cho và đều là các số nguyên tố.
m+n m+n
2) Với a, b và c là các số thực không âm thoả mãn điều kiện a + b + c = 3, tìm giá trị
lạ

lớn nhất của biểu thức P = ab + 2bc + 3ca − 3abc.


u

Lời giải.
m3

1. Đặt = p là số nguyên tố thì m3 = p(m + n). Suy ra m3 chia hết cho p, kéo theo m chia
m+n
m
hết cho p. Do đó, m + n = .m2 . Suy ra m + n chia hết cho m, kéo theo n chia hết cho m.
p
m3 m2
Chứng minh tương tự thì m cũng chia hết cho n nên m = n. Suy ra = = p là số
m+n 2
nguyên tố. Vì m2 = 2p nên m2 chia hết cho 2 kéo theo m chia hết cho 2. Suy ra m2 chia hết
cho 4 và do đó p chia hết cho 2. Do p là số nguyên tố nên p = 2 và từ đó tìm được m = n = 2.
2. Áp dụng bất đẳng thức AM-GM, ta có

(a + b + c)2 (a + b + c)2 27
P ≤ a(b + c) + 2c(a + b) ≤ +2 = .
4 4 4
3 27
Dấu bằng xảy ra khi và chỉ khi a = c = , b = 0. Vậy giá trị lớn nhất của P là .
2 4
CÂU LẠC BỘ TOÁN A1, HOTLINE: 034 761 1986 - 035 290 3286 145

Câu 4: (3.0 điểm)

Cho tam giác ABC nhọn với AB < AC. Đường tròn (I) nội tiếp tam giác ABC, tiếp xúc
với ba cạnh BC,CA và AB lần lượt tại ba điểm D, E, F.

1) Gọi M là giao điểm của hai đường thẳng AI và DF. Chứng minh đường thẳng CM
vuông góc với đường thẳng AI.

2) Gọi N là giao điểm của hai đường thẳng AI và DE. Gọi K là trung điểm của đoạn
thẳng BC. Chứng minh tam giác KMN là tam giác cân.

3) Các tiếp tuyến tại M và N của đường tròn (K; KM) cắt nhau tại điểm S. Chứng
minh đường thẳng AS song song với đường thẳng ID.

1
A
A

áE

n
To
F
I

J

N
cb

B K C
S D

M
lạ

Q P
u

Lời giải.

d = 90◦ + A . Suy ra MIC


b
1. Ta có I là tâm đường tròn nội tiếp tam giác ABC nên AIC d = 180◦ −
2
A
b
d = 90◦ − . Tam giác BDF cân tại B nên B A
b
AIC [ DF = 90◦ − . Do đó, MIC
d =B [DF = CDM.
[
2 2
[ = IMC
Từ đó suy ra tứ giác DICM là tứ giác nội tiếp. Vì thế AMC d = IDC d = 90◦ .

2. Gọi J là trung điểm của CI. Ta có JK là đường trung bình của tam giác IBC nên JK k IB. Để
ý, DM ≡ DF ⊥ IB, ta thu được JK ⊥ DM. Vì C, I, D, M cùng thuộc một đường tròn đường
kính CI nên J thuộc trung trực của DM. Suy ra JK chính là trung trực của DM. Do đó
KD = KM. Chứng minh tương tự ta cũng được KD = KN. Vì vậy, KM = KN = KD.

3. Gọi P là giao điểm của MS và AC, còn Q là giao điểm của NS và AB. Để ý rằng K chính là
tâm ngoại tiếp của 4DMN, ta suy ra NKD
[ = 2NMD [ = 2IMD d = 2ICDd = ACB.
d Từ đó thu
được KN k AC và kéo theo NS ⊥ AC. Chứng minh tương tự thì MS ⊥ AB. Chú ý rằng hai
tam giác ANQ và AMP đồng dạng nên AQ AN
AP = AM . Hai tam giác ANB và AMC đồng dạng nên
146 TUYỂN TẬP ĐỀ THI VÀO LỚP 10 CHUYÊN TOÁN

AB
AC
AN
= AM . Suy ra AQ AB
AP = AC nên theo định lý Thales đảo thì BC k PQ. Lại có trong tam giác
APQ có QS ⊥ AC, PS ⊥ AB nên S là trực tâm tam giác APQ. Suy ra AS ⊥ PQ. Mà BC k PQ
nên AS ⊥ BC. Lại có BC ⊥ ID nên AS k ID.

Câu 5: (1.0 điểm)

Cho tập hợp A gồm 70 số nguyên dương không vượt quá 90. Gọi B là tập hợp gồm các số
có dạng x + y với x, y ∈ A và x, y không nhất thiết phân biệt,

1) Chứng minh 68 ∈ B.

2) Chứng minh B chứa 91 số nguyên liên tiếp.

1
Lời giải. Ta có các nhận xét sau đây:

A
Nhận xét 1. Các số lẻ từ 43 tới 139 xuất hiện trong B.
Giả sử a lẻ và 43 ≤ a ≤ 139.

n
 
a−1 a+1
+) Nếu a < 90 xét các cặp (1, a − 1), (2, a − 2), . . . , , và các số a, a + 1, . . . , 90 thì
2 2
a−1
cặp và 91 − a số. Nếu a ∈
á
To
có / B thì A chỉ chứa tối đa một số từ các cặp trên. Suy ra số phần
2
tử của A không vượt quá
a−1 181 − a 181 − 43
+ 91 − a = ≤ = 69,

2 2 2
cb

mâu thuẫn với giả thiết.


+) Nếu a > 90. Xét các cặp
lạ

 
a−1 a+1
(a − 90, 90), (a − 89, 89), . . . , ,
2 2
u

và các số
1, 2, . . . , a − 91

181 − a
thì có tất cả là cặp và a − 91 số. Nếu a ∈
/ B thì A chỉ chứa tối đa một số từ các cặp trên,
2
suy ra số phần tử của A không vượt quá
181 − a a − 1 139 − 1
+ a − 91 = ≤ = 69,
2 2 2
mâu thuẫn với giả thiết.
Từ các lập luận trên thì a ∈ B.
Nhận xét 2. Các số chẵn từ 44 đến 138 đều thuộc B.
Xét a chẵn và 44 ≤ a ≤ 138.
+) nếu a ≤ 90 thì xét các cặp
 
a−2 a+2
(1, a − 1), (2, a − 2), . . . , ,
2 2
CÂU LẠC BỘ TOÁN A1, HOTLINE: 034 761 1986 - 035 290 3286 147

a a−2
và các số , a, . . . , 90 thì có cặp và 92 − a số. Nếu a ∈
/ B thì A chỉ chứa tối đa một số trong
2 2
mỗi cặp nên số phần tử của A không vượt quá
a−2 182 − a 182 − 44
+ 92 − a = ≤ = 69,
2 2 2
mâu thuẫn với giả thiết.
+) Nếu a > 90 thì tương tự xét các cặp
 
a−2 a+2
(a − 90, 90), (a − 89, 89), . . . , ,
2 2

và các số
a

1
1, 2, . . . , a − 91,
2

A
thì ta cũng chỉ ra được nếu a ∈
/ B thì số phần tử của A không vượt quá 69.
Như vậy các số lẻ từ 43 tới 139 và các số chẵn từ 44 đến 138 đều thuộc B.

n
1. Theo chứng minh trên thì hiển nhiên 68 ∈ B.

á
2. Các số từ 43 tới 139 thuộc B và đây là 97 số tự nhiên liên tiếp.
To

cb
lạ
u

148 TUYỂN TẬP ĐỀ THI VÀO LỚP 10 CHUYÊN TOÁN

2.24. ĐỀ THI TUYỂN SINH VÀO LỚP 10 CHUYÊN TIN HÀ


NỘI, NĂM 2022

Câu 1: (2.0 điểm)


p
1) Giải phương trình x2 − 2x + 2 = (x2 + 4)(x + 1).

2) Với a, b và c là các số thực dương thỏa mãn abc = 3, tính giá trị của biểu thức
1 1 1
P= + + .
a2 (b + c) + 3 b2 (c + a) + 3 c2 (a + b) + 3

1
Lời giải.

A
1. Điều kiện xác định là x ≥ −1. Đặt x2 + 4 = a2 và x + 1 = b2 với a > 0 và b ≥ 0, ta có phương
trình đã cho tương đương

á n
q
2
(x + 4) − 2 (x + 1) − (x2 + 4) (x + 1) = 0,
To
a2 − 2b2 − ab = 0,

(a − 2b)(a + b) = 0.
cb

Lưu ý là a + b > 0, ta suy ra a = 2b, hay

x2 + 4 = 4 (x + 1),
lạ

tương đương với


x (x − 4) = 0.
u

Giải ta tìm được x = 0, 4 đều thoả mãn điều kiện bài toán. Vậy phương trình có hai nghiệm
x = 0 và x = 4.

2. Ta có
1 1 1
P= + +
a2 (b + c) + 3 b2 (c + a) + 3 c2 (a + b) + 3
1 1 1
= + +
a2 (b + c) + abc b2 (c + a) + abc c2 (a + b) + abc
bc ca ab
= + +
abc (ab + bc + ca) abc (ab + bc + ca) abc (ab + bc + ca)
bc + ca + ab 1 1
= = = .
abc (ab + bc + ca) abc 3
CÂU LẠC BỘ TOÁN A1, HOTLINE: 034 761 1986 - 035 290 3286 149

Câu 2: (2.0 điểm)

1) Với p là số nguyên tố lớn hơn 3 , chứng minh số A = 5 p + p2 chia hết cho 6.

2) Tìm tất cả cặp số nguyên (x, y) thỏa mãn x3 − x2 y + 2x = 5x2 − 2y − 1.

Lời giải.
1. Vì p là số nguyên tố lớn hơn 3 nên p không chia hết cho 3 và p lẻ. Vì 5 p và p2 đều lẻ nên
5 p + p2 chia hết cho 2.
Lại có 5 p ≡ (−1) p ≡ −1 (mod 3) (vì p lẻ) và p2 là số chính phương không chia hết cho 3
nên p2 ≡ 1 (mod 3). Từ đây ta suy ra 5 p + p2 chia hết cho 3.

1
Chú ý (2, 3) = 1, từ các lý luận trên ta suy ra A chia hết cho 6.

A
2. Ta có x3 − 5x2 + 2x + 1 = y(x2 − 2). Vì x là số nguyên nên x2 6= 2. Khi đó

n
x3 − 5x2 + 2x + 1 x(x2 − 2) − 5(x2 − 2) + 4x − 9 4x − 9
y= = = x−5+ 2
2
x −2
á
2
x −2 x −2
To
là số nguyên. Suy ra 4x − 9 chia hết cho x2 − 2, kéo theo (4x + 9)(4x − 9) = 16x2 − 81 =
16(x2 − 2) − 49 chia hết cho x2 − 2. Từ đây ta tìm được x2 ∈ {1, 9}. Thử các trường hợp của
x, khi đó (x, y) là (−1, 7); (1, 1); (−3, −11).

cb

Câu 3: (2.0 điểm)

1) Với a, b, c là các số thực dương thỏa mãn abc = 2, chứng minh


lạ

9
a2 + b2 + c2 − 3 (a + b − c) ≥ − .
4
u

2) Tìm tất cả các số nguyên dương a, b và c sao cho các phương trình x2 − 2ax + b = 0,
x2 − 2bx + c = 0 và x2 − 2cx + a = 0 đều có nghiệm là các số nguyên dương

Lời giải.
1. Bất đẳng thức cần chứng minh tương đương với

3 2
 
2 4
a − 3a + + b − c − ≥ 0.
a 2

4 (a − 2)2 (a + 1)
Chú ý là a2 − 3a + = ≥ 0 nên bất đẳng thức trên luôn đúng. Dấu bằng xảy
a a
1
ra khi và chỉ khi a = b = 2, c = .
2
150 TUYỂN TẬP ĐỀ THI VÀO LỚP 10 CHUYÊN TOÁN

2. Ta có biệt thức ∆0 của các phương trình tương ứng là a2 − b, b2 − c, c2 − a. Phương trình
muốn có nghiệm nguyên dương thì các biệt thức đó phải là các số chính phương. Không mất
tính tổng quát có thể giả sử a = max(a, b, c). Ta có a2 − b < a2 nên a2 − b ≤ (a − 1)2 từ đó
2a ≤ b + 1 ≤ a + 1 hay a ≤ 1. Vì a nguyên dương nên a = 1, kéo theo a = b = c = 1. Kiểm
tra lại thì các giá trị này thoả mãn. Vậy, a = b = c = 1.

Câu 4: (3.0 điểm)

Cho tam giác ABC với AB < AC nội tiếp đường tròn (O). Ba đường cao AD, BE và CF
của tam giác ABC cùng đi qua điểm H. Gọi I và K lần lượt là trung điểm của EF và BC.

AI HI
1) Chứng minh = .
AK HK

1
2) Chứng minh đường thẳng AH là tiếp tuyến của đường tròn ngoại tiếp tam giác HIK.

A
3) Gọi P là chân đường vuông góc kẻ từ điểm H đến đường thẳng EF. Chứng minh
rằng đường thẳng DP song song với đường thẳng AI.

A á n
To

E
cb

G I
O
P
lạ

H
u

L B D C

Lời giải.
1. Dễ thấy hai tam giác AEF và ABC đồng dạng theo trường hợp góc-góc và AI, AK là hai trung
tuyến tương ứng nên hai tam giác AEI và ABK đồng dạng. Suy ra
AI EI EF
= = .
AK BK BC
Chứng minh tương tự thì
HI EF
=
HK BC
nên
AI HI
= .
AK HK
CÂU LẠC BỘ TOÁN A1, HOTLINE: 034 761 1986 - 035 290 3286 151

2. Ta có tam giác HEF đồng dạng với tam giác HCB do tứ giác EFBC nội tiếp. Suy ra hai tam
giác HEI và HKC đồng dạng. Do B,C, E, F cùng thuộc đường tròn đường kính BC nên K
thuộc trung trực của EF. Suy ra IK ⊥ EF, dẫn tới

d − 90◦ = HKC
d = HIE
HIK [ − 90◦ = KHD.
[

Suy ra HD là tiếp tuyến của đường tròn ngoại tiếp tam giác HIK, tức là HA là tiếp tuyến của
đường tròn ngoại tiếp tam giác HIK.

3. Gọi L là giao điểm của EF và BC, còn G là giao điểm của AL và đường tròn ngoại tiếp tam
giác ABC, với G 6= A. Kẻ đường kính AV của đường tròn ngoại tiếp tam giác ABC.
Ta có LE · LF = LB · LC = LA · LG nên AGFE là tứ giác nội tiếp. Do đó G thuộc đường tròn
[ = 90◦ . Vì AGV
đường kính AH nên AGH d = 90◦ nên G, H,V thẳng hàng. Do V, H đối xứng

1
qua K nên G, H, K thẳng hàng. Suy ra HK ⊥ AL. Mà AH ⊥ KL nên H là trực tâm tam giác

A
AKL. Do đó, LH ⊥ AK.
Vì tam giác AKB đồng dạng với tam giác AIE nên ta có

n
d = 90◦ + HPD
IPD [ = 90◦ + H
[ LD = 90◦ + 90◦ − AKL
= 180◦ − AKL
d = 180◦ − AKBá
d = 180◦ − AIE
d
d = AIP.
To
d

Từ đây suy ra DP k AI.



cb

Câu 5: (1.0 điểm)

Trên bảng có hai số tự nhiên m và n . An và Bình chơi một trò chơi như sau : Mỗi lượt
chơi, một bạn chọn một trong hai số trên bảng để xóa và viết lên bảng một số mới là hiệu
lạ

không âm của số vừa xóa với một ước số tự nhiên bất kỳ của số vừa xóa. Hai bạn luân
phiên thực hiện lượt chơi. Bạn đầu tiên không thể thực hiện được lượt chơi của mình là
người thua cuộc , người còn lại là người thắng cuộc . Biết rằng An là người thực hiện lượt
u

chơi đầu tiên.


1) Với m = 2022, n = 2023 hãy chỉ ra chiến thuật chơi của An để An là người thắng
cuộc.

2) Vơi m = 2022, n = 1981 hãy chỉ ra chiến thuật chơi của An để An là người thắng
cuộc.

Lời giải.
1. Bước đầu tiên , An chọn số 2023, xóa số đó đi , và viết vào bảng số 2022 (chú ý 2022=2023-
1). Trên bảng lúc này có hai số 2022. Sau đó, ở các bước tiếp theo, nếu Bình thực hiện bất
cứ tác động nào lên 1 số trên bảng thì thì An chỉ cần làm giống như Bình đối với số còn lại.
Như vậy, An luôn bảo đảm sau mỗi lượt chơi của An thì trên bảng luôn có hai số bằng nhau.
Các bước diễn ra như vậy, khi đó An luôn có thể thực hiện lượt chơi của mình, nên An không
thể thua, tức là đến một thời điểm nào đó, Bình sẽ thua cuộc.
152 TUYỂN TẬP ĐỀ THI VÀO LỚP 10 CHUYÊN TOÁN

2. Đầu tiên An chọn số 2022 , xóa nó đi và viết thêm vào bảng số 2021. Khi đó trên bảng có
hai số lẻ. Chú ý mọi ước tự nhiên của 1 số lẻ luôn là số lẻ, vì thế sau lượt chơi của Bình thì
trên bảng có 2 số gồm 1 số chẵn và 1 số lẻ. giả sử số chẵn này lớn hơn 0, bởi nếu số chẵn này
bằng 0 thì An sẽ thắng ở ngay lượt chơi tiếp theo. Khi đó, ở lượt tiếp theo, An sẽ xóa số chẵn
đó đi và thay bởi hiệu của số chẵn đó và 1. Như vậy, sau lượt chơi của An , trên bảng luôn có
hai số lẻ. Ở đây nếu Bình tạo ra trạng thái, sau lượt chơi của Bình thì trên bảng có một số 0
và một số lẻ lớn hơn 1 thì ngay lập tức An sẽ thắng cuộc ở bước tiếp theo. Trường hợp còn
lại , quá trình trên tiếp tục diễn ra đến khi sau lượt chơi của An trên bảng chỉ còn 2 số 1 và
1. Tại lượt chơi tiếp theo của Bình thì trên bảng có 2 số 1 và 0. Và An sẽ thắng cuộc sau lượt
chơi tiếp theo.

1
A
á n
To

cb
lạ
u

CÂU LẠC BỘ TOÁN A1, HOTLINE: 034 761 1986 - 035 290 3286 153

2.25. ĐỀ THI TUYỂN SINH VÀO LỚP 10 CHUYÊN TOÁN THÀNH


PHỐ H Ồ C HÍ M INH , NĂM 2022

Câu 1
p
Cho x, y là hai số thực thỏa mãn xy + (1 + x2 ) (1 + y2 ) = 1. Tính giá trị của biểu thức
 p  p 
M = x + 1 + y2 y + 1 + x2 .

Lời giải. Ta có:


 q 2
xy + (1 + x ) (1 + y ) = 12
2 2

1
A
q
2 2 2 2
⇔ 2x y + x + y + 2xy (1 + x2 ) (1 + y2 ) = 0
 p p 2
2
⇔ x 1+y +y 1+x 2 =0

n
p p
á
⇔ x 1 + y2 + y 1 + x2 = 0
To
Từ đó suy ra M = 1.

Câu 2

a) Giải phương trình x + 4 + |x| = x2 − x − 4.


cb

 x

 = 2x − 1
y + z


y
b) Giải hệ phương trình = 3y − 1
z+x
lạ


 z
= 5z − 1.


x+y

u

Lời giải.
a) ĐKXĐ: x > −4.

• Nếu −4 6 x < 0, khi đó ta có



x + 4 − x = x2 − x − 4

⇔ x + 4 = x2 − 4

⇔ x + 4 + x + 4 = x2 + x
 √  √ 
⇔ x− x+4 x+ x+4+1 = 0

TH1: x = x + 4; khi đó x > 0, loại.
√ 2
TH2: x + 4 = −1 − x; khi đó −4 6 x 6 −1. √ Khi đó bình phương lên ta được x + x −
−1 − 13
3 = 0. Ta giải ra và nhận nghiệm x = . (Thỏa mãn ĐKXĐ)
2
154 TUYỂN TẬP ĐỀ THI VÀO LỚP 10 CHUYÊN TOÁN

• Nếu x > 0, khi đó



x + 4 = x2 − 2x − 4

⇔ x + 4 + x + 4 = x2 − x
 √  √ 
⇔ x+ x+4 x− x+4−1 = 0


TH1: x + 4 = −x, khi đó x 6 0, loại.

TH2: x − 1 = √ x + 4. Khi đó x > 1. Bình phương lên ta được x2 − 3x − 3 = 0. Giải ra
3 + 21
ta được x = . (Thỏa mãn ĐKXĐ)
2

1
( √ √ )

A
−1 − 13 3 + 21
Vậy S = ; .
2 2

á n
b) Từ hệ phương trình ta có được 2x(y + z) = 3y(z + x) = 5z(x + y) = x + y + z; và x, y, z khác 0.
To
Từ 2x(y + z) = 3y(z + x), suy ra 2xz = 3yz + xy.

Lại từ 3y(z + x) = 5z(x + y), suy ra 3xy = 5xz + 2yz.



cb

Từ hai điều trên ta có xz = 11yz, lại do z khác 0 nên x = 11y. Khi đó 22y(y + z) = 60yz, từ
đó ta cũng có được 22y = 38z hay 11y = 19z. Từ đó
lạ

x = 11y = 19z.
u

 
19z 239 239 239
Khi đó = 38z − 1. Giải ra ta được (x, y, z) = , , .

19z 60 660 1140


+z
11

Câu 3
Cho hình vuông ABCD. Trên các cạnh BC,CD lần lượt lấy các điểm M và N sao cho
[ = 45◦ .
MAN

a) Chứng minh rằng MN tiếp xúc với đường tròn tâm A bán kính AB.

b) Kẻ MP k AN (P ∈ AB) và kẻ NQ k AM (Q ∈ AD). Chứng minh rằng AP = AQ.


CÂU LẠC BỘ TOÁN A1, HOTLINE: 034 761 1986 - 035 290 3286 155

A P B

Q
E

F D N C

1
A
Lời giải.
a) Gọi E là hình chiếu của A trên MN. Đường thẳng đi qua A vuông góc với AM cắt đường

n
thẳng CD tại F. Ta có AB = AD và BAM
[ = DAFd = (90◦ − DAM)
[ nên 4ABM = 4ADF
á
(g.c.g) và kéo theo AM = AF. Hơn nữa lại để ý rằng MAN
[ = NAF(=
[ 45◦ ) nên ta được
To
4AMN = 4AFN(c.g.c). Từ đó suy ra ANE[ = AND[ và kéo theo 4ADN = 4AEN(ch.gn).
Do đó AB = AD = AE hay E ∈ (A, AB). Từ AE ⊥ MN tại E, ta thu được MN tiếp xúc với
(A, AB) tại E.

b) Để ý MP k AN nên ANMP là hình thang. Mặt khác, vì ANM


[ = AND [ = NAP d nên ANMP là
cb

hình thang cân, vì thế AP = MN. Chứng minh tương tự như trên ta cũng có AMNQ là hình
thang cân, kéo theo AQ = MN. Như vậy AP = AQ(= MN).
lạ

Câu 4
u

Cho ba số thực dương a, b, c thỏa a + b + c = 3.


a) Chứng minh rằng ab + bc + ca ≤ 3.


a b c
b) Tìm giá trị nhỏ nhất của biểu thức P = + + .
b2 + 1 c2 + 1 a2 + 1

Lời giải.
1
a) Ta có (a + b + c)2 − 3(ab + bc + ca) = [(a − b)2 + (b − c)2 + (c − a)2 ] ≥ 0, vì thế
2

(a + b + c)2
ab + bc + ca ≤ = 3.
3

Đẳng thức xảy ra khi a = b = c = 1.


156 TUYỂN TẬP ĐỀ THI VÀO LỚP 10 CHUYÊN TOÁN

b) Theo bất đẳng thức AM-GM thì b2 + 1 ≥ 2b. Khi đó

a a(b2 + 1) − ab2 ab2 ab2 ab


2
= 2
= a − 2
≥ a − = a− .
b +1 b +1 b +1 2b 2
b bc c ca
Chứng minh tương tự ta có ≥ b− và 2 ≥ c− .
c2 + 1 2 a +1 2
ab + bc + ca
Vì thế P ≥ (a + b + c) − . Mặt khác a + b + c = 3, ab + bc + ca ≤ 3 nên P ≥
3
3 3
3 − = . Đẳng thức xảy ra khi a = b = c = 1.
2 2
3
Vậy giá trị nhỏ nhất của P là đạt được khi a = b = c = 1.
2

1
A
Câu 5
Cho tam giác ABC nhọn (AB < AC) có các đường cao AD, BE,CF cắt nhau tại H. Đường

n
thẳng EF cắt đường thẳng BC tại I. Đường thẳng qua A vuông góc với IH tại K và cắt BC
tại M.
á
To
BI CI
a) Chứng minh rằng tứ giác IKFC nội tiếp và BD = CD .

b) Chứng minh M là trung điểm của BC.



cb

A
lạ

E
u

F K

I B D M C

Lời giải.
d = ADI
a) Vì AKI d = 90◦ nên tứ giác AKDI nội tiếp. Vì thế KIC
d = KID
d = KAD
[ = KAH.
[ Ngoài
ra ta cũng có AFH [ = 90◦ nên tứ giác AFHK nội tiếp, vì thế KFC
[ = AKH [ = KFH
[ = KAH
[=
d Do đó tứ giác IKFC nội tiếp.
KIC.
IB EC FA
Áp dụng định lý Menelaus cho 4ABC với I, E, F thẳng hàng ta suy ra IC · EA · FB = 1. Áp
DB EC FA
dụng định lý Ceva cho 4ABC với AD, BE,CF đồng quy ta suy ra DC · EA · FB = 1. Như vậy
IB DB BI CI
ta thu được IC = DC hay BD = CD .
CÂU LẠC BỘ TOÁN A1, HOTLINE: 034 761 1986 - 035 290 3286 157

b) Để ý rằng bốn điểm E, F, H, K cùng nằm trên đường tròn đường kính AH và hơn nữa tứ giác
HKMD nội tiếp. Do đó, ID · IM = IH · IK = IE · IF. Từ đó ta được tứ giác DMEF nội tiếp.
[ = MFE
Vì thế ta được MFC [ − CFE [ = MDE [ − HFE [ = BAC d − HAE[ = HAF [ = MCF.
[ Từ đó
4MFC cân tại M nên MF = MC. Hơn nữa ta cũng có MBF [ = 90◦ − MCF [ = 90◦ − MFC
[=
[ nên 4MFB cân tại M. Như vậy MB = MF = MC.
MFB

Câu 6
Số nguyên dương n được gọi là "số tốt" nếu n + 1 và 8n + 1 đều là các số chính phương.

a) Hãy chỉ ra ví dụ ba "số tốt" lần lượt có 1, 2, 3 chữ số.

b) Tìm các số nguyên k thỏa mãn |k| ≤ 10 và 4n + k là hợp số với mọi n là "số tốt".

1
A
Lời giải.
a) Ta có 3 là "số tốt" nhỏ nhất cũng là "số tốt" có 1 chữ số.

n
15 là "số tốt" có 2 chữ số, vì 15 + 1 = 16 là số chính phương và 8 · 15 + 1 = 121 là số chính
phương. á
To
120 là "số tốt" có 3 chữ số vì 120 + 1 = 121 là số chính phương và 8 · 120 + 1 = 961 = 312
là số chính phương.

b) Vì 3 "số tốt" nhỏ nhất nên với n là "số tốt" thì 4n + k ≥ 12 + k.


Xét n là một "số tốt" bất kỳ. Vì số chính phương chia cho 3 chỉ có thể dư 0 hoặc 1, mà
cb

n + 1, 8n + 1 là số chính phương nên n không thể chia cho 3 dư 1, 2. Vì vậy n phải chia hết
cho 3.
Với k ∈ {−8, −6, −4, −2, 0, 2, 4, 6, 8, 10} thì 4n + k luôn là số nguyên lớn hơn 2 và chia hết
lạ

hết cho 2 với mọi n là "số tốt", nên 4n + k là hợp số với mọi n là số tốt.
Với k ∈ {−3, 0, 3, 9} thì 4n + k luôn là số nguyên lớn hơn 3 và chia hết cho 3 với mọi n là số
u

tốt, nên 4n + k là hợp số với mọi n là "số tốt" .


Với k = −10, khi n = 3 thì 4n + k = 2 là số nguyên tố nên k = −10 loại.


Với k = −9, khi n = 3 thì 4n + k = 3 là số nguyên tố nên k = −9 loại.
Với k = −7, khi n = 3 thì 4n + k = 5 là số nguyên tố nên k = −7 loại.
Với k = −5, khi n = 3 thì 4n + k = 7 là số nguyên tố nên k = −5 loại.
Với k = −1, khi n = 3 thì 4n + k = 11 là số nguyên tố nên k = −1 loại.
Với k = 1, khi n = 3 thì 4n + k = 13 là số nguyên tố nên k = 1 loại.
Với k = 5, khi n = 3 thì 4n + k = 17 là số nguyên tố nên k = 5 loại.
Với k = 7 khi n = 3 thì 4n + k = 19 là số nguyên tố nên k = 7 loại.
Vậy k ∈ {−8, −6, −4, −3, −2, 0, 2, 3, 4, 6, 8, 9, 10}.
158 TUYỂN TẬP ĐỀ THI VÀO LỚP 10 CHUYÊN TOÁN

2.26. ĐỀ THI TUYỂN SINH VÀO LỚP 10 TRƯỜNG THPT CHUYÊN


KHOA HỌC TỰ NHIÊN NĂM 2023, VÒNG 1

Câu 1: (3,5 điểm)


1. Giải phương trình
p √ p √
x2 + 6x + 2023 + x + 3 = x2 + 5x + 2025 + 5.

2. Giải hệ phương trình



(x + 6y) (3x + 2y) = 12

1
2x3 + 6y3 + 15x2 y + 19y2 x + x + 6y = 12.

A
Lời giải.

n
1. Điều kiện xác định: x ≥ −3.

á
Cách 1. Xét x > 2 thì vì (x2 + 6x + 2023) − (x2 + 5x + 2025) = x − 2 > 0 và (x + 3) − 5 =
x − 2 > 0 nên suy ra
To
p √ p √
x2 + 6x + 2023 + x + 3 ≥ x2 + 5x + 2025 + 5.

Tương tự nếu −3 ≤ x < 2 thì


cb

p √ p √
x2 + 6x + 2023 + x + 3 ≤ x2 + 5x + 2025 + 5.

Do đó, ta phải có x = 2 và nghiệm này thoả mãn điều kiện xác định. Vậy x = 2 là nghiệm
lạ

duy nhất của phương trình


Cách 2. Phương trình đã cho có thể viết lại thành
u

p √ p √
x2 + 6x + 2023 − 5 = x2 + 5x + 2025 − x + 3.

Giả sử hai vế cùng dấu, bình phương hai vế và rút gọn, ta được
q q
5(x2 + 6x + 2023) = (x2 + 5x + 2025)(x + 3).

Bình phương một lần nữa, khai triển và rút gọn, ta được

(x − 2)(x2 + 5x + 2020) = 0.

Vì x2 + 5x + 2020 = 0 vô nghiệm nên x = 2 và nghiệm này thoả mãn.


Vậy, phương trình đã cho có nghiệm duy nhất x = 2.

2. Biến đổi phương trình thứ hai thành

(x + 6y)(2x2 + 3xy + y2 + 1) = 12.


CÂU LẠC BỘ TOÁN A1, HOTLINE: 034 761 1986 - 035 290 3286 159

Kết hợp với phương trình thứ nhất ta suy ra


(x + 6y)(3x + 2y) = (x + 6y)(2x2 + 3xy + y2 + 1).
Mà ta thấy x + 6y 6= 0 nên chia cả hai vế phương trình trên cho x + 6y ta được
2x2 + 3xy + y2 + 1 = 3x + 2y.
Biến đổi phương trình trên ta được
0 = 2x2 + 3xy + y2 − 3x − 2y + 1
= (2x + y)(x + y) − (2x + y) − (x + y) + 1
= (2x + y − 1)(x + y − 1)

nên suy ra 2x + y − 1 = 0 hoặc x + y − 1 = 0.

1
• Nếu 2x + y − 1 = 0 thì ta có y = 1 − 2x. Thay vào phương trình thứ nhất ta được

A
(x + 6 − 12x)(3x + 2 − 4x) = 12.
 
28 −45
Giải phương trình ta thu được các nghiệm (x, y) = (0, 1), , .

n
11 11

á
• Nếu x + y − 1 = 0 thì ta có y = 1 − x. Thay voà phương trình thứ nhất ta được
To
(x + 6 − 6x)(3x + 2 − 2x) = 12.
 
−4 9
Giải phương trình ta thu được các nghiệm (x, y) = (0, 1), , .
5 5

   
28 −45 −4 9
cb

Vậy, hệ đã cho có ba nghiệm (x, y) = (0, 1), , , , .


11 11 5 5

Câu 2: (2,5 điểm)


lạ

1) Giả sử n là số nguyễn sao cho 3n3 − 1011 chia hết cho 1008. Chứng minh rằng n − 1
chia hết cho 48 .
u

2) Với a, b, c là các số dương thỏa mãn điều kiện ab + bc + ca = 1. Chứng minh rằng
   
1 1 1
1+ 1+ 1+ > 4.
1 + a2 1 + b2 1 + c2

Lời giải.
1. Ta có các biến đổi sau 1008 | 3n3 − 1011 = 3n3 − 3 − 1008 khi và chỉ khi 1008 | 3n3 − 3 =
3(n3 − 1) tương đương với
336 | n3 − 1 = (n − 1)(n2 + n + 1) = n3 − n + n − 1.

Vì 336 chia hết cho 16 mà n2 + n + 1 = n(n + 1) + 1 lẻ nên n − 1 chia hết cho 16.
Ngoài ra vì 336 chia hết cho 3 mà n3 − n = n(n − 1)(n + 1) tích ba số nguyên liên tiếp
chia hết cho 3 nên n − 1 chia hết cho 3.
Vì (3, 16) = 1 nên n − 1 chia hết cho 3.16 = 48. Phép chứng minh hoàn tất.
160 TUYỂN TẬP ĐỀ THI VÀO LỚP 10 CHUYÊN TOÁN

2. Ta cần chứng minh

   
1 1 1
1+ 1+ 1+ > 4. (1)
1 + a2 1 + b2 1 + c2

Ta biến đổi tương đương bất đẳng thức (1) như sau.

2 + a2 2 + b2 2 + c2
   
(1) ⇔ >4
1 + a2 1 + b2 1 + c2

1
⇔ 2 + a2 2 + b2 2 + c2 > 4 1 + a2 1 + b2 1 + c2
     

A
⇔ 3 (abc)2 + 2 a2 b2 + b2 c2 + c2 a2 < 4

 
⇔ 3 (abc)2 + 2 (ab + bc + ac)2 − 2abc (a + b + c) < 4

n
⇔ 3 (abc)2 + 2 (1 − 2abc (a + b + c)) < 4
á
⇔ 3 (abc)2 − 4abc (a + b + c) < 2. (2)
To

3
Theo bất đẳng thức AM − GM và giả thiết ta có 1 = ab + bc + ac ≥ 3 a2 b2 c2 kéo theo

1
3 (abc)2 ≤ < 2. Suy ra bất đẳng thức (2) đúng và bài toán được chứng minh.
cb

9
lạ
u

Câu 3: (3 điểm)

Cho hai đường tròn (O) và (O0 ) cắt nhau tại A và B sao cho O nằm ngoài (O0 ) và O0 nằm
ngoài (O). Trên đường tròn (O) lấy điểm P di chuyển sao cho P nằm trong đường tròn
(O0 ). Đường thẳng AP cắt (O0 ) tại C khác A.

1) Chứng minh rằng hai tam giác OBP và O0 BC đồng dạng.

2) Gọi Q là giao điểm của hai đường thẳng OP và O0C. Chứng minh rằng

d = 90◦ .
d + ABP
QBC

3) Lấy điểm D thuộc (O) sao cho AD vuông góc với O0C. Chứng minh rằng trung điểm
của đoạn thẳng DQ luôn nằm trên một đường tròn cố định khi P thay đổi.
CÂU LẠC BỘ TOÁN A1, HOTLINE: 034 761 1986 - 035 290 3286 161

E
A Q

P
M

O X O0
D F

1
A
B

n
C

Lời giải. á
To
0C. Kết hợp với OB O0 B
d = 2BAP
1. Ta có BOP d = 2BAC [
d = BO
OP = O0 C = 1, ta thu được 4OBP ∼
0
4O BC.

2. Vì 4OBP ∼ 4O0 BC nên ta được OPB


d =O[0CB, từ đó kéo theo tứ giác BCQP nội tiếp. Do
d Hơn nữa, vì 4OAP cân tại O nên ta được 1 AOP d = 90◦ .
cb

d = QPC
đó, QBC d = OPA. d + OPA
2
Như vậy,
d + ABP
QBC d + 1 AOP
d = OPA d = 90◦ .
2
lạ

3. Gọi M là trung điểm của DQ. Đặt AD ∩ O0C = E. Vì các tứ giác BCQP và ADBP nội tiếp
d = BPC
nên ta được BQC d = ADBd =E [DB, vì thế tứ giác BDEQ nội tiếp đường tròn đường
u

kính DQ (với tâm đường tròn là M). Do đó, ta được


[ = 1 DMB
BMO [ =D[EB = AEB.
d
2
Mặt khác, ta cũng có
[ = 180◦ − 1 BOD
BOM [ = 180◦ − BAD
d = BAE. d
2
Như vậy, ta được 4OBM ∼ 4ABE (g.g). Gọi F, X,Y lần lượt là trung điểm của AB, OO0 , OB.
Khi đó, MY và EF là các đường trung tuyến tương ứng của 4OBM và 4ABE, vì thế ta
được 4OMY ∼ 4AEF. Kết hợp với việc tứ giác AEO0 F nội tiếp (vì AEO [0 = AFO[0 =
◦ 0
90 ), XY là đường trung bình của 4OBO và sử dụng tính đối xứng, ta được
[ = AEF
OMY [
d = AO0 F = AO
[ 0 O = BO
[ 0 O = OXY
[.
Như vậy, tứ giác OMXY nội tiếp. Mặt khác, vì, O, B, O0 là các điểm cố định nên X,Y cũng
là các điểm cố định. Do vậy, điểm M luôn chạy trên đường tròn (OXY ) cố định.
162 TUYỂN TẬP ĐỀ THI VÀO LỚP 10 CHUYÊN TOÁN

Câu 4: (1 điểm)

Giả sử tập A là tập con của tập hợp gồm 30 số tự nhiên đầu tiên {0, 1, 2, ..., 29} sao cho
với k nguyên bất kỳ và a, b ∈ A bất kỳ (có thể a = b) thì a + b + 30k không là tích của hai
số nguyên liên tiếp. Chứng minh rằng số phần tử của tập hợp A nhỏ hơn hoặc bằng 10.

Lời giải. Ta trình bày hai lời giải


Cách 1. Trước hết ta loại các số mà bản thân nó không thể xuất hiện trong A, bao gồm:
n(n + 1)
• Các số có dạng : 0,1,3,6,10,15,24,28;
2
n(n + 1) + 30
• Các số có dạng : 16,18,25;

1
2

A
n(n + 1) − 30
• Các số có dạng : 13.
2

n
Đối với các số còn lại, ta ghép cặp chúng như sau:

á
(2, 4), (5, 7), (8, 12), (9, 11), (19, 23), (20, 22), (24, 26), (27, 29), 14, 17.
To
(gồm 8 cặp số và 2 số lẻ không trong cặp nào). Ta thấy các số không cùng một cặp không thể
cùng thuộc A, cho nên A chỉ chứa đúng một số trong mỗi cặp. Do đó A chỉ có tối đa 10 phần tử.
Cách 2. Với hai số nguyên liên tiếp n, n + 1, ta có n(n + 1) ≡ 0, 2, 6, 12, 20, 26 (mod 30). Với

a ∈ A, xét b = a và k = 0 ta có 2a không đồng dư với 0, 2, 6, 12, 20, 26 (mod 30) nên a không
cb

đồng dư với 0, 1, 3, 6, 10, 13, 15, 16, 18, 21, 25, 28 (mod 30). Suy ra

A ⊂ B = {2, 4, 5, 7, 8, 9, 11, 12, 14, 17, 19, 20, 22, 23, 24, 26, 27, 29}
lạ

và nếu phân hoạch B thành 10 tập

{2, 4}, {5, 7}, {8, 12}, {11, 9}, {14, 22}, {17, 19}, {20}, {23, 27}, {24, 26}, {29}
u

thì mỗi tập con này chứa nhiều nhất một phần tử của A. Do đó, A chứa tối đa 10 phần tử.
Thực ra ta có thể chứng minh được số phần tử của A nhiều nhất bằng 10, chỉ cần chọn

A = {2, 5, 8, 11, 14, 17, 20, 23, 26, 29}.


CÂU LẠC BỘ TOÁN A1, HOTLINE: 034 761 1986 - 035 290 3286 163

Phân tích hướng giải bài Tổ hợp thi chuyên KHTN năm 2023

Trong đề thi vòng 1, trường THPT chuyên KHTN năm 2023, có bài Tổ hợp sau

Giả sử tập A là tập con của tập hợp gồm 30 số tự nhiên đầu tiên {0, 1, 2, ..., 29} sao cho
với k nguyên bất kỳ và a, b ∈ A bất kỳ (có thể a = b) thì a + b + 30k không là tích của hai
số nguyên liên tiếp. Chứng minh rằng số phần tử của tập hợp A nhỏ hơn hoặc bằng 10.

Trong bài viết này, tôi trình bày một hướng tiếp cận bài toán này. Giả thiết bài toán có thể diễn
giải thành không tồn tại a, b ∈ A và số nguyên n mà a + b ≡ n(n + 1) (mod 30) hay tương đương
4(a + b) + 1 ≡ (2n + 1)2 (mod 120). Vì a có thể bằng b nên nói riêng nếu chọn a = b thì A
không chứa phần tử a nào mà 8a + 1 ≡ (2n + 1)2 (mod 120).
Giờ ta xét xem một số chính phương lẻ chia 120 có những số dư nào? Xét m là số lẻ và m2 ≡ b

1
(mod 120). Ngoài ra chú ý là vì m lẻ nên

A
(m + 30)2 = m2 + 60m + 900 ≡ m2 + 60 + 60 ≡ m2 (mod 120)
Do đó ta chỉ cần xét m = 1, 3, 5, 7, 9, ..., 29. Tính toán trực tiếp thì

n
m2 ≡ 1, 9, 25, 49, 81, 105.
á
To
Thế thì 8a + 1 6≡ 1, 9, 25, 49, 81, 105 (mod 120) hay

a 6≡ 0, 1, 3, 6, 10, 13 (mod 15).


Như vậy là A sẽ không chứa các phần tử 0, 1, 3, 6, 10, 13, 16, 18, 25, 28. Từ đó
cb

A ⊂ B = {2, 4, 5, 7, 8, 9, 11, 12, 14, 17, 19, 20, 22, 23, 24, 26, 27, 29}.

Tuy nhiên liệu A có thể lấy hết các phần tử thuộc B không? Câu trả lời là không. Ta để ý tiếp là
lạ

2, 4 không thể cùng thuộc A vì nếu không thì chọn a = 2, b = 4 thì a + b = 6 = 2.3. Hoặc 14, 22
không thể cùng thuộc A vì nếu chọn a = 14, b = 22, k = −1 thì a + b − 30k = 6 = 2.3. Tương
u

tự như thế thì trong các nhóm sau, mỗi nhóm chỉ chứa tối đa một phần tử của A

(2, 4), (5, 7), (8, 12), (9, 11), (14, 22), (17, 19), (23, 27), (24, 26), 20, 29.
Lập luận trên cho ta A chỉ chứa tối đa 10 phần tử. Một ví dụ về tập A có đúng 10 phần tử thoả
mãn là
A = {2, 5, 8, 11, 14, 17, 20, 23, 26, 29}.
164 TUYỂN TẬP ĐỀ THI VÀO LỚP 10 CHUYÊN TOÁN

2.27. ĐỀ THI TUYỂN SINH VÀO LỚP 10 TRƯỜNG THPT CHUYÊN


KHOA HỌC TỰ NHIÊN NĂM 2023, VÒNG 2

Câu 1: (3 điểm)
1) Giải phương trình p p
2x + 1 + 2 4x2 + 6x = 4 5x − x2 .

2) Giải hệ phương trình


(
xy(x + y) = 30,

x3 + y3 = 30 + 3 x + y + 120.

1
A
Lời giải.
1) Điều kiện: 0 ≤ x ≤ 5. Ta biến đổi phương trình thành

n
√ √ √ √
x + 2 x 4x + 6 + 4x + 6 = 4x + 4 x 5 − x + 5 − x.
á
To
Sử dụng hằng đẳng thức, ta thu được
√ √ √ √
( x + 4x + 6)2 = (2 x + 5 − x)2

√ √ √ √
Suy ra x + 4x + 6 = 2 x + 5 − x (do từng vế đều không âm), hay
cb

√ √ √
4x + 6 = x + 5 − x.

Bình phương hai vế của phương trình này ta có


lạ

p
4x + 6 = x + 5 − x + 2 x(5 − x)
u

p
hay 4x + 1 = 2 x(5 − x). Tiếp tục ta bình phương hai vế với điều kiện 4x + 1 ≥ 0 (đã thoả

mãn) được
16x2 + 8x + 1 = 4x(5 − x).
1 1
Giải phương trình trên ta thu được x = và x = (đều thoả mãn điều kiện).
2 10
1 1
Vậy, phương trình đã cho có đúng hai nghiệm x = , x = .
2 10
2) Đặt S = x + y, P = xy. Ta có

x3 + y3 = (x + y)3 − 3xy(x + y) = S3 − 3SP.

Khi đó hệ phương trình trở thành


(
SP = 30,
3

S − 3SP = 30 + 3 S + 120.
CÂU LẠC BỘ TOÁN A1, HOTLINE: 034 761 1986 - 035 290 3286 165

Thay SP = 30 vào phương trình thứ hai ta có



S3 = 120 + 3 S + 120.

hay S3 + S = (S + 120) + 3 S + 120. Ta nhận thấy

• Nếu S > 3 S + 120 thì S3 > S + 120, suy ra

S3 + S > (S + 120) + 3 S + 120,
loại.

• Nếu S < 3
S + 120 thì S3 < S + 120, suy ra

S3 + S < (S + 120) + 3 S + 120,
loại.

1

Như vậy ta có S = 3 S + 120, hay S3 − S − 120 = 0. Giải phương trình ta thu được S = 5, khi

A
30
đó P = = 6. Vậy ta có
S (
x + y = 5,

n
xy = 6.
á
Theo Vi-ét đảo thì x, y là hai nghiệm của phương trình
To
X 2 − 5X + 6 = 0.
Giải phương trình ta thu được (x, y) = (2, 3) hoặc (3, 2).

Vậy, hệ đã cho có hai nghiệm (x, y) là (2, 3) và (3, 2).


cb

Câu 2: (3 điểm)
lạ

1) Tìm tất cả các cặp số nguyên dương (x; y) thỏa mãn

4x + (1 + 3y ) (1 + 7y ) = 2x (3y + 7y + 2)
u

2) Với x, y, z là những số thực dương, tìm giá trị nhỏ nhất của biểu thức:

x14 − x6 + 3 y14 − y6 + 3 z14 − z6 + 3


M= 2 2 + +
x y + zx + zy y2 z2 + xy + xz z2 x2 + yz + yx

Lời giải.
1) Cách 1. Ta có các biến đổi phương trình sau
4x + (1 + 3y ) (1 + 7y ) = 2x (3y + 7y + 2) ⇔ 22x − 1 + 2 + 3y + 7y + 21y = 2x (3y + 7y + 2)
⇔ (2x − 1) (3y + 7y + 1 − 2x ) = 21y (1)

Ta chứng minh gcd(2x −1, 3y +7y +1−2x ) = 1. Thật vậy, nếu gcd(2x −1, 3y +7y +1−2x ) >
1 thì gọi p là ước nguyên tố chung của 2x − 1, 3y + 7y + 1 − 2x . Suy ra p | 3y + 7y . Chú ý là
3x + 7y đều không chia hết cho 3,7 nên p 6= 3, 7. Lại có p | 21y nên p ∈ {3, 7} mâu thuẫn.
Vậy gcd(2x − 1, 3y + 7y + 1 − 2x ) = 1. Ta xét hai trường hợp sau
166 TUYỂN TẬP ĐỀ THI VÀO LỚP 10 CHUYÊN TOÁN

• Nếu x là số chẵn thì 2x − 1 chia hết cho 3 và 3y + 7y + 1 − 2x chia 3 dư 1.


Khi đó, từ phương trình (1) ta có
(
2x − 1 = 3y
.
3y + 7y + 1 − 2x = 7y .

Suy ra 2x = 3y + 1. Chú ý là 3y ≡ 1, 3 (mod 8) nên 3y + 1 không chia hết cho 8. Từ đó


x = 2 và y = 1. Vậy (x, y) = (2, 1).
• Nếu x là số lẻ thì 2x − 1 chia 3 dư 1 và 3y + 7y + 1 − 2x chia hết cho 3.
Khi đó, từ phương trình (1) ta có
(
2x − 1 = 7y

1
3y + 7y + 1 − 2x = 3y

A
Suy ra 2x = 7y + 1. Vế phải chia 7 dư 1 nên vế trái chia 7 dư 1. Từ đó x = 3k, k ∈ N∗ và
thay vào phương trình được

á n
(2k − 1)(22k + 2k + 1) = 7y .
To
Vì gcd(2k −1, 22k +2k +1) ∈ {1, 3} nên gcd(2k −1, 22k +2k +1) = 1. Vì 22k +2k +1 >
1 nên 2k − 1 = 1 suy ra k = 1 và 7y = 7 nên y = 1 và x = 3k = 3. Vậy (x, y) = (3, 1).

Vậy tất cả các cặp số (x, y) thỏa mãn là (2, 1), (3, 1).
cb

Cách 2. Phương trình đã cho có thể viết lại thành

(2x − 7y − 1)(2x − 3y − 1) = 0.
lạ

Tới đây giải giống hai trường hợp ở trên.


u

2) Ta có 3(x14 − x6 + 3) = (3x14 + 4) − 3x6 + 5 ≥ 7x6 − 3x6 + 5 = 4x6 + 5 theo bất đẳng thức

AM-GM. Lại có cũng theo bất đẳng thức AM-GM, thì

4x6 + 5 = (x6 + x6 + 1) + (x6 + x6 + 1) + 3 ≥ 3(x4 + x4 + 1) ≥ 3(x4 + 2x2 ).

Suy ra
x4 x2
M≥∑ + 2 ∑ x2y2 + xz + yz
x2 y2 + xz + yz
và áp dụng bất đẳng thức cộng mẫu cho vế trái, ta có

(x2 + y2 + z2 )2 + 2(x + y + z)2 3(x2 y2 + y2 z2 + z2 x2 ) + 6(xy + yz + zx)


M≥ ≥ = 3.
x2 y2 + y2 z2 + z2 x2 + 2(xy + yz + zx) x2 y2 + y2 z2 + z2 x2 + 2(xy + yz + zx)

Dấu bằng xảy ra khi và chỉ khi x = y = z = 1. Giá trị nhỏ nhất của M là 3.
CÂU LẠC BỘ TOÁN A1, HOTLINE: 034 761 1986 - 035 290 3286 167

Câu 3: (3 điểm)

Cho tam giác ABC nhọn vói AB < AC nội tiếp trong đường tròn (O) có tiếp tuyến tại A
của (O) cắt BC ở T sao cho T B > BC. Gọi P và E lần lượt là trung điểm của TA và TC.

1) Chứng minh rằng tứ giác APEB nội tiếp.

2) Gọi giao điểm thứ hai của AE với (O) là F. Lấy G thuộc (O) sao cho FG song song
với AC. Chứng minh rằng ATdG = TAF.
d

3) Gọi H là trực tâm của tam giác ABC, D là giao điểm của AH và BC. M là trung điểm
BC. K đối xứng với A qua BC. N thuộc đường thẳng AM sao cho KN song song với
HM. Lấy S thuộc BC sao cho NS ⊥ NK. Dựng R thuộc tia AK sao cho AR · AH = AD2 .
Q là điểm sao cho PQ ⊥ AS và SQ ⊥ AO. Chứng minh rằng điểm đối xứng của A qua

1
QR thuộc đường tròn đường kính DN.

A
n
A
á
To
P
O
F

cb

T E B C
lạ

G
u

Lời giải.

1
1) Vì AT là tiếp tuyến của (O) nên ta được TA2 = T B · TC. Như vậy, ta được T P · TA = TA2 =
2
1
T B · TC = T B · T E và vì thế tứ giác APEB là tứ giác nội tiếp.
2

2) Vì EP là đường trung bình của 4TAC, AFGC là hình thang cân và AT là tiếp tuyến của
(O) nên ta thu được AEP
d = EAC
d = FACd = GCA
d = TAGd và GACd = FCA d = TAF d = PAE.
d
AE AP
Như vậy, ta được 4AEP ∼ 4ACG (g.g) và dẫn đến = . Lại chú ý rằng AT = 2AP
AC AG
AE 2AE 2AP AT
và AC = 2EP, ta thu được = = = d = TAG
. Kết hợp với AEP d ta thu được
EP AC AG AG
4AEP ∼ 4TAG (c.g.c) và vì thế AT
dG = TAF.
d

3)
168 TUYỂN TẬP ĐỀ THI VÀO LỚP 10 CHUYÊN TOÁN

1
A
á n
To
A

cb

P J
H O

W
lạ

T B D M X C S
Y
R
u

Z G

K A'
L

Q A'' N

Y1
CÂU LẠC BỘ TOÁN A1, HOTLINE: 034 761 1986 - 035 290 3286 169

Gọi AY là đường kính của (O), L và Y1 là giao của AY với QS và KN.


Theo tính chất quen thuộc ta có M là trung điểm của HY nên theo bổ đề hình thang ta được
N là trung điểm của KY1 .
Vì SN ⊥ KY1 và SD là trung trực AK nên S là tâm ngoại tiếp của tam giác AKY1 .
Vì QS ⊥ AO nên AY1 ⊥ SL mà S là tâm (AKY1 ) nên L là trung điểm AY1 . Theo tính chất
đường trung bình ta được DN k AO và NL k AD suy ra tứ giác ADNL là hình bình hành. Gọi
Z là giao của DL và AN thì ta được Z là trung điểm AN và DL.
Gọi A00 là đối xứng của A qua R, X là giao của NL với BC, A0 là giao của AX với (R) thì
XA0 ⊥ A0 A00 nên A0 thuộc đường tròn (DN).
Áp dụng định lý Thales ta được
AD AD AH AM AM
= = = = .
AA00 2AR 2AD 2AZ AN
Theo định lý Thales đảo ta suy ra NA00 k DM nên NA00 ⊥ DA00 suy ra DXNA00 là hình chữ

1
nhật và D, X, A00 , N thuộc (DN).

A
Gọi U,V lần lượt là giao của QS và QP với AH; W là giao của AO và BC.
Ta có ∆QUV ∼ ∆AW S(g.g) (do 2 tam giác này có các cặp cạnh tương ứng vuông góc).
Từ V kẻ V G ⊥ QS vì SQ k AT nên V G ⊥ AT .

n
Vì đường qua T vuông góc với AS và đường qua S vuông góc với AT đồng quy tại trực tâm
á
tam giác AT S trên AH nên V G phải đi qua trung điểm J của AS.
To
Mặt khác JG k AL (cùng vuông góc với QS nên G là trung điểm LS.
Theo tính chất đường trung bình của ∆ANA00 , ∆DLS ta được GZ k DS, RZ k NA00 mà NA00 k DS
nên G, Z, R thẳng hàng và GR ⊥ VU.
Ta có V[GU = W LS = 90◦ và VUG
[ = LW dS nên ∆V GU ∼ ∆SLW (g.g).

d
Vì LX và GR là hai đường cao tương ứng của hai tam giác này nên theo tính tương ứng của
cb

VR WX
đồng dạng ta được tỷ lệ thức sau = .
RU XS
Từ đây kết hợp với ∆QUV ∼ ∆AW S(g.g) ta được ∆QUR ∼ ∆AW X(c.g.c) suy ra QR vuông
góc AX. Vì QR ⊥ AA0 mà R thuộc trung trực AA0 nên A và A0 đối xứng nhau qua QR.
lạ

Vậy điểm đối xứng của A qua QR là A0 thuộc (DN).


u

Câu 4: (1 điểm)

Viết 100 số nguyên dương đầu tiên 1, 2, ..., 100 vào một bảng ô vuông kích thước 10 × 10
một cách tuỳ ý sao cho mỗi ô vuông được viết đúng một số. Chứng minh rằng tồn tại hai
ô kề nhau (hai ô có cạnh chung) mà hai số được viết ở hai ô này có hiệu lớn hơn hoặc
bằng 10.

Lời giải.
Cách 1. Ta giải bài toán tổng quát: Điền các số 1, 2, ..., n2 với n > 1 vào các ô vuông của bảng
cỡ n × n. Khi đó tồn tại hai ô vuông kề nhau (chung cạnh) chứa hai số x, y mà |x − y| ≥ n.
Kí hiệu mk , Mk tương ứng là số nhỏ nhất và số lớn nhất của hàng thứ k với k = 1, 2, ..., n. Chú ý
là m1 , m2 , ..., mn , M1 , M2 , ..., Mn đôi một phân biệt. Đặt
m = max(m1 , m2 , ..., mn ) và M = min(M1 , M2 , ..., Mn ).
Xét hai trường hợp
170 TUYỂN TẬP ĐỀ THI VÀO LỚP 10 CHUYÊN TOÁN

• Nếu m < M thì ta có mk ≤ m < Mk với mọi k = 1, 2, ..., n. Điều này suy ra với hàng k bất
kỳ thì tồn tại hai số ak , bk thuộc hàng đó sao cho ak ≤ m < bk và với mỗi hàng k ta chọn
cặp (ak , bk ) thuộc hai ô kề nhau ở hàng k. Vì b1 , b2 , ..., bn lớn hơn m và các số b1 , b2 , ..., bn
là đôi một phân biệt nên tồn tại k ∈ {1, 2, ..., n} sao cho bk ≥ n + m và do đó

bk − ak ≥ (m + n) − m = n.

• Nếu m > M thì gọi i, j ∈ {1, 2, ..., n} là các chỉ số sao cho mi > M = M j . Khi đó tại mỗi
cột luôn tồn tại các số không vượt quá M (ví dụ như số hàng j) và các số lớn hơn M (ví
dụ như số ở hàng i). Khi đó với cột k bất kỳ tồn tại ak , bk sao cho ak ≤ M < bk và ak , bk
thuộc hai ô kề nhau của cột k. Tương tự như trường hợp đầu tiên thì ta cũng có tồn tại
k ∈ {1, 2, ..., n} mà bk ≥ M + n. Suy ra

1
bk − ak ≥ (M + n) − M = n.

A
Trong mọi tình huống ta đều có điều phải chứng minh.

n
Cách 2. (Phạm Việt Hưng) Giả sử phản chứng, tồn tại cách điền để không tồn tại x, y như vậy.
Ta lần lượt điền các số bắt đầu từ 1 vào bảng ô vuông, xét thời điểm đầu tiên mà cả n cột đều đã
được điền số. Ta xét hai trường hợp sau: á
To
• Nếu tồn tại cột nào đó đã được phủ hoàn toàn, ta sẽ đổi vai trò của hàng và cột (xét thời
điểm đầu tiên cả n hàng đều có số). Nếu khi đó lại tồn tại một hàng được phủ hoàn toàn,

thì tức là hai thời điểm đang xét trùng nhau. Điều này là không thể vì ô vừa được thêm
phải là ô cuối cùng của hàng và cột đó, xét thời điểm ngay trước đó cho ta tất cả các hàng
cb

đều đã được điền.

• Nếu không tồn tại cột đã phủ hoàn toàn, do mỗi cột đều có ô đã điền, ta có thể chọn ra ở
cột thứ i được cặp ô Ai , Bi mà Ai được điền còn Bi thì chưa.
lạ

Cần có Bi − Ai ≤ n − 1, kéo theo Bi ≤ max(A1 , A2 , ..., An ) + n − 1, ∀i.


Mà các số từ 1 đến max(Ai ) đều đã được điền nên Bi > max(Ai ), vô lý do n số Bi phân
u

biệt.

Các trường hợp cho ta giả sử sai và ta có điều phải chứng minh.
CÂU LẠC BỘ TOÁN A1, HOTLINE: 034 761 1986 - 035 290 3286 171

2.28. ĐỀ THI TUYỂN SINH VÀO LỚP 10 TRƯỜNG THPT CHUYÊN


SƯ PHẠM HÀ NỘI NĂM 2023, VÒNG 2
Câu 1
a) Chứng minh rằng tích của bốn số nguyên liên tiếp cộng với 1 là bình phương của
một số nguyên.
(
2xy − x = 10,
b) Tìm các cặp số nguyên của hệ phương trình
x + y + xy = 11.

1
Lời giải.

A
a) Ta gọi bốn số nguyên liên tiếp lần lượt là n, n + 1, n + 2, n + 3 với n ∈ Z. Tích của bốn số
này cộng với 1 là

n
n(n + 1)(n + 2)(n + 3) + 1 = (n2 + 3n)(n2 + 3n + 2) + 1 = (n2 + 3n + 1)2

là số chính phương. á
To
b) Theo phương trình x + y + xy = 11 ta có xy = 11 − x − y, thay vào phương trình 2xy −
12 − 3x
x = 10 ta có 3x + 2y = 12 hay y = . Thay vào phương trình đâu tiên ta được
2

(12 − 3x) − x = 10 hay


−3x2 + 11x − 10 = 0.
cb

5
Phương trình có hai nghiệm là x1 = 2, x2 = . Vì x ∈ Z nên x = 2, khi đó y = 3.
3
lạ

Vậy hệ đã cho có nghiệm nguyên duy nhất (x, y) là (2, 3).


u

Câu 2

a) Cho a, b là các số thực không âm và c là số thực dương thoả mãn


√ √ √ √
a − a + b − c = b + c.
√ √ √ √
Chứng minh rằng 3 a + 3 b − 3 c = 3 a + b − c.
√ √
3+ a
b) Tìm các số nguyên dương a, b sao cho √ √ là số hữu tỉ.
5+ b

Lời giải.

a) Nếu b = c thì thay vào bài toán ta có c = 0 mâu thuẫn nên b 6= c. Viết lại giả thiết thành

c−b b−c
√ √ =√ √ .
a+ a+b−c b− c
172 TUYỂN TẬP ĐỀ THI VÀO LỚP 10 CHUYÊN TOÁN

√ √ √ √ √ √
Suy ra a + a + b − c = c √ − b. √ Kết hợp với giả thiết ta suy ra a√= c√hay a√= c.
Thay
√ vào giả thiết thì được − b = b dẫn tới b = 0. Khi đó ta có 3 a + 3 b − 3 c =
3
a + b − c = 0.
√ √ √ √
Vậy, trong mọi tình huống ta luôn có 3 a + 3 b − 3 c = 3 a + b − c.
√ √
b) Bổ đề. Nếu x, y là các số hữu tỉ không âm thoả mãn x − y là số hữu tỉ thì hoặc x = y
hoặc x, y là bình phương các số hữu tỉ.
Bổ đề này đã dạy ở Tổ Kiến nhiều lần
Đặt √ √
3+ a
√ √ = n,
5+ b
trong đó n ∈ Q+ . Viết lại đẳng thức trên dưới dạng

1
√ √ √ √
a − n b = n 5 − 3.

A
Bình phương hai vế, ta suy ra
√ √

n
a + n2 b − 2n ab = 5n2 + 3 − 2n 15,
kéo theo √ √ á
To
2n( ab − 15) = a + n2 b − 5n2 − 3.
√ √
Vế phải là số hữu tỉ nên vế trái cũng hữu tỉ, do n hữu tỉ dương nên ta phải có ab − 15
cũng là một số hữu tỉ. Theo một bổ đề đã dạy ở Tổ Kiến thì ab = 15. Từ đó (a, b) là

(1, 15), (3, 5), (5, 3), (15, 1). Thử trực tiếp thì chỉ có (a, b) = (5, 3) thoả mãn bài toán.
cb

Câu 3
Cho tam giác ABC có đường tròn nội tiếp (I) tiếp xúc với ba cạnh BC, CA, AB tại D, E,
lạ

G. Hai đường thẳng DE, DG lần lượt cắt đường phân giác ngoài BAC
d tại M, N. Hai đường
thẳng MG, NE cắt nhau tại P. Chứng minh rằng
u

a) EG k MN.

b) Điểm P thuộc đường tròn (I).

M
A
N
E
P
G I

B C
D
CÂU LẠC BỘ TOÁN A1, HOTLINE: 034 761 1986 - 035 290 3286 173

Lời giải. a) Ta thấy MN vuông góc với AI và AI vuông góc với EG nên EG k MN.

b) Định nghĩa lại điểm P là giao điểm thứ hai khác A của (AGN) và (AEM).
Ta có
d = AGN
APN [ = BGD
[ = GED [ = 180◦ − APE.
[ = AME d

Suy ra ba điểm E, P, N thẳng hàng. Chứng minh tương tự ta có G, P, M thẳng hàng. Do


đó điểm P định nghĩa lại giống với đề bài ban đầu.
[ = PAN
Lại có PGD d = PEM
[ nên tứ giác GPED nội tiếp hay P cũng thuộc (I).

Câu 4

1
Bảy lục giác đều được sắp xếp và tô màu bằng hai màu trắng, đen như ở Hình 1. Mỗi lần

A
cho phép chọn ra một lục giác đều, đổi màu của lục giác đó và của tất cả các lục giác đều
chung cạnh với lục giác đó (trắng thành đen và đen thành trắng). Chứng minh rằng dù có
thực hiện cách làm trên bao nhiêu lần đi nữa, cũng không thể nhận được các lục giác đều

n
được ô màu như ở Hình 2.

á
To

cb
lạ

Hình 1 Hình 2
u

Lời giải.
Cách 1. Đánh số vào các hình lục giác như hình vẽ.

0
1 1
0
1 1
0
174 TUYỂN TẬP ĐỀ THI VÀO LỚP 10 CHUYÊN TOÁN

Ta xét một hình lục giác được điền số ai thì ai + bi ≡ 0 (mod 2) trong đó bi là tổng các số được
điền trong các hình lục giác chung cạnh với hình lục giác đang xét. Khi đổi màu thì tổng các số
trong các hình màu đen đồng dư với tổng các số trong các hình màu trắng theo modulo 2. Do
đó, sau khi đổi màu thì tổng các số trong các hình màu đen không đổi theo modulo 2.
Đối với hình 1 thì số dư này bằng 0, còn đối với hình 2 thì số dư này bằng 1 nên không thể có
cách đổi màu nào biến hình 1 thành hình 2.
Cách 2.

1
2 6
7

1
A
3 5
4

á n
Xét các ô 2, 3, 5, 6. Mỗi bước ta đổi màu hai hoặc cả bốn ô đó nên số ô đen không thay đổi tính
To
chẵn, lẻ. Ban đầu trong bốn ô nói trên có hai ô đen nên không thể có trạng thái trong bốn ô đó
có đúng một ô đen.
Cách 3. Giả sử phản chứng ta có thể đổi màu để từ hình 1 thu được hình 2. Cũng đánh số như

trong cách 2 và gọi xi là số lần chọn hình thứ i để đổi màu với i = 1, 2, ..., 7. Chú ý là một hình
sẽ không đổi màu nếu số lần đổi màu của hình đó là số chẵn. Ta có số lần đổi màu của các hình
cb

+) Hình 1: số lần là x1 + x2 + x6 + x7 là số lẻ.


+) Hình 2: số lần là x1 + x2 + x3 + x7 là số lẻ
+) Hình 3: số lần là x2 + x3 + x4 + x7 là số lẻ
lạ

+) Hình 4: số lần là x3 + x4 + x5 + x7 là số lẻ
+) Hình 5: số lần là x4 + x5 + x6 + x7 là số chẵn
u

+) Hình 6: số lần là x5 + x6 + x1 + x7 là số chẵn.


Từ hình 1 và hình 4 suy ra x1 + x2 + x3 + x4 + x5 + x6 chẵn, trong khi đó từ hình 2 và 5 lại suy ra

x1 + x2 + x3 + x4 + x5 + x6 lẻ, mâu thuẫn.

Câu 5
Chứng minh rằng tồn tại số nguyên dương n > 102023 sao cho tổng tất cả các số nguyên
tố nhỏ hơn n là một số nguyên tố cùng nhau với n.

Lời giải. Xét một số nguyên tố p > 102023 và gọi p1 , ..., pk là tất cả các số nguyên tố nhỏ hơn p
( p1 < p2 < ... < pk ). Xét hai khả năng

• Nếu gcd(p1 + p2 + ... + pk , p) = 1 thì chọn n = p ta có điều phải chứng minh.

p(p + 1)
• Nếu p | p1 + p2 + ..pk thì ta viết kp = p1 + ... + pk < với k ∈ N∗ .
2
CÂU LẠC BỘ TOÁN A1, HOTLINE: 034 761 1986 - 035 290 3286 175

p+1
Suy ra k < . Chọn q là số nguyên tố nhỏ nhất lớn hơn p tức là p, q là hai số nguyên
2
tố liên tiếp. Thế thì tổng tất cả các số nguyên tố nhỏ hơn q là

S = p1 + p2 + · · · + pk + p = p(k + 1).

p+1 p+3
Chú ý là k + 1 < +1 = < q nên q nguyên tố cùng nhau với p, k + 1 và do đó
2 2
(q, S) = 1. Chọn n = q ta có ngay điều phải chứng minh.

1
A
á n
To

cb
lạ
u

176 TUYỂN TẬP ĐỀ THI VÀO LỚP 10 CHUYÊN TOÁN

2.29. ĐỀ THI TUYỂN SINH VÀO LỚP 10 CHUYÊN TOÁN HÀ


NỘI, NĂM 2023

Câu 1: (2,0 điểm)


1. Giải phương trình √ √
x − 3 − 2x − 7 = 2x − 8

2. Cho a, b, c là các số thực khác 0 thỏa mãn điều kiện a2 − c2 = c, c2 − b2 = b và


b2 − a2 = a. Chứng minh

(a − b)(b − c)(c − a) = 1.

1
A
Lời giải.
7

n
1) Điều kiện xác định x ≥ .
2
á
Sử dụng nhân liên hợp, ta có phương trình ban đầu tương đương với
To
x − 3 − 2x + 7
√ √ = 2x − 8.
x − 3 + 2x − 7

Chuyển vế, rút nhân tử chung ta được


cb

 
1
(x − 4) 2 + √ √ = 0.
x − 3 + 2x − 7
lạ

√ √ 1 7
Ta có x − 3 ≥ 0, 2x − 7 ≥ 0 nên 2 + √ √ > 0 với mọi x ≥ , kéo theo x = 4
x − 3 + 2x − 7 2
u

(thỏa mãn điều kiện xác định).


Vậy phương trình đã cho có nghiệm duy nhất x = 4.

2) Theo đề bài ta có a2 − c2 + c2 − b2 = c + b và a, b, c 6= 0 nên a2 − b2 = c + b. Nếu a + b = 0


thì a = −b. Tuy nhiên khi đó a = b2 − a2 = 0 là trái giả thiết. Do đó, ta phải có a + b 6= 0
c+b
dẫn tới a − b = .
a+b
c+a a+b
Hoàn toàn tương tự ta có b − c = và c − a = .
b+c c+a
Từ đây ta suy ra
c+b c+a a+b
(a − b)(b − c)(c − a) = · · = 1.
a+b b+c c+a
Đây chính là điều phải chứng minh.
CÂU LẠC BỘ TOÁN A1, HOTLINE: 034 761 1986 - 035 290 3286 177

Câu 2: (2,0 điểm)

1) Cho ba số nguyên a, b và c thỏa mãn a2 + b2 + c2 − 2abc chia hết cho 6. Chứng minh
abc chia hết cho 54.

2) Tìm tất cả cặp số nguyên dương (x, y) thỏa mãn x3 y − x2 y − 4x2 + 5xy − y2 = 0.

Lời giải.
1) Nếu cả ba số a, b, c đều lẻ thì a2 + b2 + c2 − 2abc sẽ lẻ và do đó a2 + b2 + c2 − 2abc không
chia hết cho 6, trái giả thiết. Do đó, trong ba số a, b, c phải có ít nhất một số chẵn, nghĩa là
abc chia hết cho 2.
Nếu abc không chia hết cho 3 tức là trong 3 số a, b, c không có số nào chia hết cho 3, dẫn tới

1
a2 ≡ b2 ≡ c2 ≡ 1 (mod 3). Vì a2 + b2 + c2 − 2abc chia hết cho 3 nên −2abc cũng chia hết

A
cho 3, vô lí vì a, b, c đều không chia hết cho 3. Do đó, ta phải có abc chia hết cho 3. Từ đây
ta có ngay a2 + b2 + c2 chia hết cho 3. Vì số chính phương khi chia 3 thì dư chỉ có thể là 0,1
cả 3 số a2 , b2 , c2 phải chia hết cho 3 kéo theo a, b, c đều chia hết cho 3. Khi đó abc chia hết

n
cho 27.

á
Vì (27, 2) = 1 nên abc chia hết cho 27 · 2 = 54. Phép chứng minh hoàn tất.
To
2) Phương trình đã cho được viết lại thành

y2 − (x3 − x2 + 5x)y + 4x2 = 0.


Coi phương trình trên là phương trình bậc hai ẩn y, tính biệt thức
cb

∆ = (x3 − x2 + 5x)2 − 16x2 = x2 (x2 − x + 1)(x2 − x + 9).

Điều kiện cần để phương trình có nghiệm y nguyên là ∆ là số chính phương. Suy ra (x2 − x +
lạ

1)(x2 −x+9) là số chính phương (do x nguyên dương nên x2 > 0). Vì x2 −x+1 = x(x−1)+1
là số lẻ và nếu gọi d = gcd(x2 − x + 1, x2 − x + 9) thì d lẻ và d | (x2 − x + 9) − (x2 − x + 1) = 8
nên ta phải có d = 1. Suy ra x2 − x + 1, x2 − x + 9 đều là các số chính phương. Mà
u

(x − 1)2 < x2 − x + 1 < (x + 1)2

nên x2 − x + 1 = x2 và tìm được x = 1. Thay x = 1 tìm được y = 1, y = 4. Vậy phương trình


đã cho có hai nghiệm nguyên dương (x, y) là (1, 1) và (1, 4).

Câu 3: (2,0 điểm)

1) Tìm tất cả các cặp số nguyên (x, y) sao cho xy là số chính phương và x2 + xy + y2 là số
nguyên tố.

2) Với các số thực không âm a, b, c thoả mãn a + 2b + 3c = 1, tìm giá trị lớn nhất và giá
trị nhỏ nhất của biểu thức P = (a + 6b + 6c)(a + b + c).

Lời giải.
178 TUYỂN TẬP ĐỀ THI VÀO LỚP 10 CHUYÊN TOÁN

1) Đặt xy = z2 với z ∈ N thì x2 + xy + y2 = x2 + y2 + z2 = (x + y)2 − z2 = (x + y − z)(x + y + z).


Chú ý là xy = z2 ≥ 0 nên x, y ở cùng phía với 0. Và nếu cặp (x, y) thoả mãn thì cặp (−x, −y)
cũng thoả mãn, do đó ta chỉ cần xét x, y ≥ 0. Khi đó x + y + z ≥ x + y − z và do x2 + y2 + z2
là số nguyên tố nên ta phải có x + y − z = 1, x2 + y2 + z2 = x + y + z. Do x, y, z ∈ N nên
x2 ≥ x, y2 ≥ y, z2 ≥ z nên để có đẳng thức x2 + y2 + z2 = x + y + z thì x2 = x, y2 = y, z2 = z.
Suy ra x, y ∈ {0, 1}. Thử trực tiếp thì chỉ có x = y = 1 là thoả mãn bài toán.
Vậy, có hai cặp (x, y) là (1, 1), (−1, −1).

2) Theo bất đẳng thức Cauchy − Schwarz, ta có

  r !2
3 1 3
P≥ a + 3b + 3c (3a + 3b + 3c) ≥ a + 3b + 3c ≥ (a + 2b + 3c)2 = 1.
2 2 2

1
A
2 1
Suy ra P ≥ . Dấu bằng xảy ra khi và chỉ khi a = b = 0, c = .
3 3
2

n
Vậy giá trị nhỏ nhất của P là .
3
Lại có, theo bất đẳng thức AM-GM thì á
To
(a + 6b + 6c + 4a + 4b + 4c)2 25(a + 2b + 3c)2 25
4P = (a+6b+6c)(4a+4b+4c) ≤ ≤ = ,
4 4 4

kéo theo
cb

25
P≤ .
16
Dấu bằng xảy ra khi và chỉ khi a + 6b + 6c = 4(a + b + c), c = 0, a + 2b + 3c = 1. Giải ta tìm
lạ

1 3 25
được a = , b = , c = 0. Vậy giá trị lớn nhất của P là .
4 8 16
u

Câu 4: (3,0 điểm)

Cho tam giác ABC có ba góc nhọn (AB < AC), nội tiếp đường tròn (O). Ba đường cao
AD, BE và CF của tam giác ABC cùng đi qua điểm H. Đường thẳng EF cắt đường thẳng
AD tại điểm Q. Gọi M và I lần lượt là trung điểm của các đoạn thẳng BC và AH. Đường
thẳng IM cắt đường thẳng EF tại điểm K.

1) Chứng minh rằng tam giác AEK đồng dạng với tam giác ABM.

2) Đường thẳng EF cắt đường thẳng BC tại điểm S, đường thẳng SI cắt đường thẳng MQ
tại điểm T . Chứng minh rằng bốn điểm A, T, H và M cùng thuộc một đường tròn.

3) Tia T H cắt đường tròn (O) tại điểm P. Chứng minh rằng ba điểm A, K và P thẳng
hàng.
CÂU LẠC BỘ TOÁN A1, HOTLINE: 034 761 1986 - 035 290 3286 179

E
I
K
T Q O

F H

1
S B D M C

A
P ≡ P0

Lời giải. á n
To
1) Xét các tam giác BFC và BEC lần lượt vuông tại F và E với các trung tuyến tương ứng là
1
FM và EM, khi đó ta được FM = EM = BC. Tương tự, xét các tam giác AFH và AEH
2
lần lượt vuông tại F và E với các trung tuyến tương ứng là FI và EI, khi đó ta cũng được

1
FI = EI = AH. Như vậy, MI là đường trung trực của EF, vì thế K là trung điểm của EF.
cb

2
AE AF
Mặt khác, lại chú ý rằng 4AEB ∼ 4AFC (g.g) nên ta được = , kéo theo 4AEF ∼
AB AC
d và AE = EF = 2EK = EK . Do đó, 4AEK ∼
lạ

4ABC (c.g.c). Từ đó ta thu được AEF


d = ABC
AB BC 2BM BM
4ABM (c.g.c).
u

2) Xét 4ISM với ID ⊥ SM và SK ⊥ IM (vì MI là trung trực của EF), vì thế Q là trực tâm của
M = 90◦ = IEM

4ISM. Như vậy, MQ ≡ MT ⊥ SI và từ đó ta được IT d d = IFM.


d Do đó, năm
điểm I, T, E, F, M cùng thuộc một đường tròn và dẫn đến QT · QM = QE · QF. Mặt khác,
lại chú ý rằng tứ giác AEHF là tứ giác nội tiếp, ta cũng có QE · QF = QA · QH. Như vậy,
QT · QM = QA · QH, vì vậy bốn điểm A, T, H, M cùng thuộc một đường tròn.

3) Trên tia T H lấy một điểm P0 sao cho HT · HP0 = HA · HD. Khi đó, ta cũng được HT · HP0 =
HB · HE = HC · HF và do đó các tứ giác T BP0 E và TCP0 F là các tứ giác nội tiếp. Khi đó, ta
[
có BP 0 T = BET
d = HET [ và CP [ 0 T = CFT
[ = HFT
[ . Từ đó, chú ý rằng tứ giác T IEF nội tiếp
nên E
[ T F = EIF
d = 2BAC,d ta thu được

[
BP0C = BP
[ 0 T + CP
[ 0 T = HET [ = 360◦ − EHF
[ + HFT [ −E TF
[
= 360◦ − (180◦ − BAC) d = 180◦ − BAC.
d − 2BAC d

Do đó P0 ∈ (O) và kéo theo P0 ≡ P. Như vậy, HA · HD = HT · HP nên tứ giác AT DP nội


d = DT
tiếp và DAP [ H. Mặt khác, ta có các kết quả quen thuộc BAO [ và AO ⊥ EF,
d = CAH
180 TUYỂN TẬP ĐỀ THI VÀO LỚP 10 CHUYÊN TOÁN

kết hợp với 4AEK ∼ 4ABM, ta thu được OAM [ = BAOd − BAM[ = CAH [ − EAK [ = DAK [ và
IM k AO(⊥ EF). Lại chú ý rằng các tứ giác AT HM và IT DM là các tứ giác nội tiếp, ta được

DT
[ [ − IDT
H = AHT [ − IMT
d = AMT d = AMI
d = OAM
[ = DAK.
[

d = DAK,
Do đó, DAP [ từ đó suy ra A, P, K thẳng hàng.

Câu 5: (1 điểm)

Cho 2023 điểm nằm trong một hình vuông cạnh 1. Một tam giác đều được gọi là phủ
điểm M nếu điểm M nằm trong tam giác hoặc nằm trên cạnh của tam giác.

1
1
1) Chứng minh tồn tại tam giác đều cạnh √ phủ ít nhất 253 điểm trong 2023 điểm đã

A
2
cho.
11

n
2) Chứng minh tồn tại tam giác đều cạnh phủ ít nhất 506 điểm trong 2023 điểm đã
12
cho.
á
To
Lời giải. 1) Chia hình vuông thành 8 phần như sau

cb
lạ

1
Mỗi một phần đều là tam giác vuông cân với độ dài cạnh bên bằng .
2
u

Theo nguyên lý Dirichlet, có 2023 điểm được phân bố vào trong 8 phần như hình vẽ trên nên

tồn tại một phần có chứa ít nhất


 
2023
+ 1 = 253 điểm.
8

1
Nói cách khác, tồn tại một tam giác vuông cân có cạnh bằng chứa ít nhất 253 điểm. Mà
2
1
tam giác vuông cân có cạnh bên bằng thì sẽ chứa trong một tam giác đều có cạnh bằng
2
1
√ , ta có điều phải chứng minh.
2

2) Gọi hình vuông được cho là ABCD với tâm O. Ta sử dụng hai đường vuông góc IF, EG đi qua
O và tạo với các cạnh một góc 60◦ để chia hình vuông thành 4 tứ giác AIOG, BIOE,CEOF,
DFOG như hình vẽ.
CÂU LẠC BỘ TOÁN A1, HOTLINE: 034 761 1986 - 035 290 3286 181

N A H I B
60◦
G

O
E
M

D F C

1
Theo nguyên lý Dirichlet, có 2023 điểm được phân bố vào trong 4 phần như hình vẽ trên nên
tồn tại một phần phủ ít nhất

A
 
2023
+ 1 = 506 điểm.
4

n
Không mất tính tổng quát, giả sử tứ giác AIOG phủ ít nhất 506 điểm. Ta dựng tam giác đều
á
IMN sao cho A ∈ IN, O ∈ IM và G ∈ MN như hình vẽ. Kẻ OH ⊥ AB. Ta có OI =
OH
=
To
√ sin 60◦
1/2 1 1 OG 1/ 3 1
√ = √ . Tương tự OG = √ . Ta lại có OM = = √ = . Suy ra
3/2 3 3 tan 60◦ 3 3

1 1 11
IM = OI + OM = √ + < .
cb

3 3 12

11
Như vậy tam giác đều IMN có cạnh nhỏ hơn , suy ra tồn tại tam giác đều phủ toàn bộ tam
12
lạ

giác đều IMN. Tam giác đều ấy do đó phủ toàn bộ tứ giác AIOG, suy ra nó cũng phủ ít nhất
506 điểm được cho.
u

182 TUYỂN TẬP ĐỀ THI VÀO LỚP 10 CHUYÊN TOÁN

2.30. ĐỀ THI TUYỂN SINH VÀO LỚP 10 CHUYÊN TIN HÀ


NỘI, NĂM 2023

Câu 1: (2,0 điểm)


1) Giải phương trình √
2x + 2 = (5 − x) 3x − 2.
(
x + y + 3xy = 9
2) Giải hệ phương trình
x3 + y3 = 9

1
Lời giải.

A
2
1) Điều kiện xác định x ≥ . Phương trình ban đầu tương đương với
3

n
3x − 2 − x + 5 − (5 − x) 3x − 2 − 1 = 0.

á
Đặt 3x − 2 = a, 5 − x = b. Phương trình trở thành
To
a2 + b − ab − 1 = 0 ⇔ (a − 1)(a + 1 − b) = 0.

Từ đó ta có a = 1 hoặc a = b − 1.

cb

• Với a = 1 thì x = 1.
• Với a = b − 1 thì √
3x − 2 = 4 − x.
lạ

Bình phương hai vế với điều kiện x ≤ 4 ta có

3x − 2 = x2 − 8x + 16 ⇔ x2 − 11x + 18 = 0.
u

Giải phương trình trên ta được x = 2 và x = 9. Kết hợp điều kiện ta còn x = 2.

Vậy phương trình đã cho có hai nghiệm x = 1 và x = 2.

2) Đặt x + y = a, xy = b. Hệ phương trình trở thành


(
a + 3b = 9
a3 − 3ab = 9

Từ phương trình a + 3b = 9 ta có 3b = 9 − a, thay vào phương trình còn lại, ta được:

a3 − a(9 − a) = 9 ⇔ (a + 1)(a − 3)(a + 3) = 0.

Giải phương trình trên ta có a = 3, a = −1 và a = −3.

• Với a = 3 thì b = 2, ta có x = 1, y = 2 và x = 2, y = 1.
CÂU LẠC BỘ TOÁN A1, HOTLINE: 034 761 1986 - 035 290 3286 183

10
• Với a = −1 thì b = , ta không có x, y thỏa mãn.
3
• Với a = −3 thì b = 4, ta không có x, y thỏa mãn.
Vậy hệ phương trình đã cho có đúng hai nghiệm (x; y) là (1; 2), (2; 1).

Câu 2: (2,0 điểm)


2 +2
1) Cho p là số nguyên tố lớn hơn 3. Chứng minh số A = 2 p − 8 chia hết cho 21 .

2) Tìm tất cả các số nguyên x và y thỏa mãn x3 − y3 = 2(x − y)2 + 17.

Lời giải.

1
1) Vì p là số nguyên tố lớn hơn 3 nên p là số lẻ và p không chia hết cho 3.

A
2 2
Do đó p2 + 2 = 2k + 1, k ∈ N, kéo theo 2 p +2 = 2.4k ≡ 2 ≡ 8 (mod 3). Suy ra A = 2 p +2 − 8
chia hết cho 3.
Do p không chia hết cho 3 nên p2 + 2 ≡ 1 + 2 ≡ 0 (mod 3). Đặt p2 + 2 = 3h với h ∈ N∗ .

n
Khi đó, ta có
A = 2p
2 +2
á
− 8 = 23h − 8 = 8h − 8 ≡ 1 − 1 = 0 (mod 7).
To
Từ đây ta suy ra A chia hết cho 3 và 7 mà (3, 7) = 1 nên A chia hết cho 3 · 7 = 21.
Phép chứng minh hoàn tất.

2) Đặt d = x − y thì x = y + d. Chú ý là vế phải lớn hơn 0 nên x > y kéo theo d là số nguyên
dương. Thay vào phương trình, ta được
cb

(y + d)3 − y3 = 2d 2 + 17.
Khai triển và chuyển vế thì
lạ

(3y2 + 3dy + d 2 − 2d)d = 17.


Từ phương trình trên suy ra d | 17 vì d nguyên dương suy ra d ∈ {1, 17}.
u

• Nếu d = 1 thì y(y + 1) = 6 và tìm được y = 2, x = 3 và y = −3, x = −2.


• Nếu d = 17 thì 3y2 + 51y = −254 không có nghiệm nguyên do vế trái chia hết cho 3,
trong khi đó vế phải thì không.
Vậy, có duy nhất một cặp (x, y) thoả mãn là (3, 2), (−2, −3).
Nhận xét. Thực tế thì có thể chuyển 2(x − y)2 rồi rút thừa số chung x − y.

Câu 3: (2,0 điểm)

1) Cho đa thức f (x) = x4 + 2x3 + 3x2 + 2022x + 2023. Chứng minh đa thức f (x) không
có nghiệm hữu tỉ.

2) Với các số thực a, b và c thỏa mãn (a + 1)(b + 1)(c + 1) = (a − 1)(b − 1)(c − 1), tìm
giá trị nhỏ nhất của biểu thức A = |a| + |b| + |c|.
184 TUYỂN TẬP ĐỀ THI VÀO LỚP 10 CHUYÊN TOÁN

Lời giải.
p
1) Giả sử f (x) có nghiệm α hữu tỷ. Viết α = với p, q ∈ Z, q > 0 và (p, q) = 1.
q
Khi đó, ta có

f (α) = 0 ⇔ α 4 + 2α 3 + 3α 2 + 2022α + 2023 =0


⇔ p4 + 2p3 q + 3p2 q2 + 2022pq3 + 2023q4 = 0.

Từ đây do q | 0 ta được q | p4 vì (p4 , q) = (p, q) = 1 nên q = 1. Khi đó, ta có

0 = p4 + 2p3 + 3p2 + 2022p + 2023


= p2 (p2 + 1) + 2p3 + 2p2 + 2022p + 2023.

1
A
Phương trình này vô nghiệm vì vế trái là số lẻ trong khi vế phải là số chẵn. Do đó đa thức
f (x) không có nghiệm hữu tỉ. Phép chứng minh hoàn tất.

n
2) Từ giả thiết ta suy ra ab + bc + ac = −1, có A = |a| + |b| + |c| ≥ 0, xét

á
A2 = a2 + b2 + c2 + 2|ab| + 2|bc| + 2|ac|.
To
Theo bất đẳng thức giá trị tuyệt đối, ta có

A2 = (a + b + c)2 + 2(|ab| + |bc| + |ac|) + 2 ≥ 0 + 2|ab + bc + ac| + 2 = 4.


cb

Từ đây kết hợp A ≥ 0 suy ra A ≥ 2, dấu bằng xảy ra chẳng hạn a = 0, b = −1, c = 1.
Vậy giá trị nhỏ nhất của A = 2.
lạ

Câu 4: (3,0 điểm)


u

Cho hai đường tròn (O; R) và (O0 ; R0 ) cắt nhau tại hai điểm phân biệt A và

B (R < R0 < OO0 ). Gọi PQ là tiếp tuyến chung của hai đường tròn (O) và (O0 ) với P ∈ (O)
và Q ∈ (O0 ). Đường thẳng PQ cắt đường thẳng OO0 tại điểm S. Qua điểm S vẽ một đường
thẳng cắt đường tròn (O) tại hai điểm E, F và cắt đường tròn (O0 ) tại hai điểm G, H sao
cho SE < SF < SG < SH.

1) Chứng minh đường thẳng OE song song với đường thẳng O0 G.

2) Chứng minh SA2 = SP · SQ.

3) Tiếp tuyến tại điểm A của đường tròn (O) cắt đường thẳng OO0 tại điểm M. Tiếp tuyến
tại điểm A của đường tròn (O0 ) cắt đường thẳng OO0 tại điểm N. Đường thẳng ME
EA2 IA
cắt đoạn thẳng AB tại điểm I. Chứng minh = và ba điểm N, I, H là ba điểm
EB2 IB
thẳng hàng.
CÂU LẠC BỘ TOÁN A1, HOTLINE: 034 761 1986 - 035 290 3286 185

P
A
C
O N M O0
S J
I
E
B
F G

1
A
H

n
Lời giải.
á
1) Trên đường thẳng S, E, F lấy các điểm G0 , H 0 sao cho O0 G0 k OE và O0 H 0 k OF. Theo định
To
SE OE SO SF OF SO OP R
lý Thales, ta được SG 0 = O0 G0 = SO0 = SH 0 = O0 H 0 . Mặt khác, ta cũng có SO0 = O0 Q = R0 . Như
vậy,
OE OF R
0 0
= 0 0 = 0.
OG OH R

Kết hợp với OE = OF = R (vì E, F ∈ (O)), ta thu được O0 G0 = O0 H 0 = R0 và vì thế G0 , H 0 ∈


cb

(O0 ) hay S, E, F ∩ (O0 ) = {G0 , H 0 }. Mặt khác, từ SG


SE SF 0 0
0 = SH 0 và SE < SF ta suy ra SG < SH .
0 0 0
Do đó, ta được G ≡ G, H ≡ H và vì thế OE k O G, OF k O H. 0
lạ

2) Trên đường thẳng SA lấy điểm C sao cho OC k O0 A. Khi đó, ta có OOC SO R
0 A = SO0 = R0 . Kết hợp với

O0 A = R0 (vì A ∈ (O0 )) nên OC = R và vì thế C ∈ (O). Hơn nữa, ta cũng có SC SO SP


SA = SO0 = SQ ,
d Từ đó 4SAP ∼ 4SQA (g.g). Do đó, SP = SA .
u

vì thế CP k AQ. Do đó, SQAd = SPC


d = SAP.
SA SQ
và SA2 = SP · SQ.

3) Gọi ME ∩ (O) = {J, E}. Từ tính đối xứng nên ta cũng có MB là một tiếp tuyến của (O). Khi
đó, ta có 4MJA ∼ 4MAE (g.g) và 4MJB ∼ 4MBE (g.g) nên ta được

JA MJ MJ JB
= = = .
EA MA MB EB

EB JB
Từ đó ta thu được EA = JA . Từ đó để ý rằng 4IAE ∼ 4IJB và 4IBE ∼ 4IJA nên ta được

IA IA IE JA EA EA JA EA2
= · = · = · = .
IB IE IB EB JB EB JB EB2

Bây giờ ta sẽ chứng minh EA HA 2 2


EB = HB . Thật vậy, ta có SP = SE · SF và SQ = SG · SH.
SE SG
Do đó, SB4 = SA4 = SP2 · SQ2 = SE · SF · SG · SH. Mặt khác, từ câu a ta có SF = SH hay
186 TUYỂN TẬP ĐỀ THI VÀO LỚP 10 CHUYÊN TOÁN

SE · SH = SG · SF. Như vậy, ta được SA4 = SB4 = (SE · SH)2 hay SA2 = SB2 = SE · SH. Từ
đó ta thu được 4SEA ∼ 4SAH (c.g.c) và 4SEB và 4SBH (c.g.c). Do vậy,
EA SE SE EB
= = = .
HA SA SB HB
Nói cách khác, ta thu được EA HA 0
EB = HB . Đến đây, đặt HN ∩ AB = I . Chứng minh tương tự như
0 2 0
ý trên ta cũng thu được II 0 AB = HA
HB2
IA
. Từ đó suy ra IB = II 0 AB và dẫn đến I ≡ I 0 . Như vậy, N, I, H
thẳng hàng.

Câu 5: (1 điểm)

Trên bàn có hai túi kẹo: túi thứ nhất có 18 viên kẹo, túi thứ hai có 21 viên kẹo. An và

1
Bình cùng chơi một trò chơi như sau: mỗi lượt chơi, một bạn sẽ lấy đi 1 viên kẹo từ một

A
túi bất kỳ hoặc là mỗi túi lấy đi 1 viên kẹo. Hai bạn luân phiên thực hiện lượt chơi của
mình. Người đầu tiên không thể thực hiện được lượt chơi của mình là người thua cuộc,
người còn lại là người thắng cuộc. Nếu An là người lấy kẹo trước, hãy chỉ ra chiến thuật

n
chơi của An để An là người thắng cuộc.
á
To
Lời giải. Đầu tiên An sẽ bốc 1 viên từ túi thứ hai, hai túi lúc này lần lượt có 18 và 20 viên kẹo.
Tại lượt tiếp theo, chiến thuật của An sẽ là nếu Bình bốc như thế nào thì An sẽ bốc y hệt như
vậy. Khi đó ta thấy Bình sẽ phải bắt đầu bốc với hai túi đều có số chẵn viên kẹo, mà Bình thì dù

bốc thế nào cũng sẽ khiến cho túi mà Bình bốc còn lại một số lẻ các viên kẹo, hay nói riêng, là
cb

còn kẹo. Như vậy khi đến lượt An thì An hoàn toàn có thể sao chép cách bốc của Bình, do cứ
túi nào mà Bình bốc thì phải còn kẹo. Hơn nữa sau khi An bốc thì bất cứ túi nào có số lẻ viên
kẹo sẽ quay về còn lại số chẵn viên kẹo. Khi đó đến lượt Bình thì Bình lại phải bốc với hai túi
còn số chẵn viên kẹo, và An vẫn có thể lặp lại chiến thuật như trên. Trong quá trình bốc này, ta
lạ

thấy An luôn có thể bốc kẹo, cho nên An không thể là người thua cuộc, nói cách khác, An sẽ là
người thắng cuộc với chiến thuật này.
u

You might also like